Dermatology

Pataasin ang iyong marka sa homework at exams ngayon gamit ang Quizwiz!

Case A 9-year-old girl developed erythema of the cheeks; subsequently, an erythematous maculopapular rash started spreading from her arms to her trunk and legs. She complains of itching. Her medical history is non-contributory, and the results of physical examination are otherwise unremarkable. Question What is the appropriate therapy?

Correct answer: Supportive antipruritics Explanation The pattern of rash progress indicates erythema infectiosum, a mild, self-limited systemic disease which is caused by a human parvovirus. The usual therapy consists only of supportive measures, such as antipruritics, to relieve itching. Antibiotics are irrelevant; the mild, self-limited nature of the disease makes antiviral therapy unnecessary in immunocompetent patients.

Case A 62-year-old woman well known to you presents with a severely itchy rash. The patient's medical history includes Parkinson's disease, which was diagnosed around 7 years ago. The patient thought that the rash was just eczema initially, but the areas have morphed into severely tense, large blisters; they are extremely pruritic to the patient. On examination you note multiple bullae 1 to 3 cm in size that are tense, and appear to be sitting on an erythematous base. The bullae are located on the patient's lower abdomen in both lower quadrants, as well as in the bilateral axillary and inguinal folds. Question What is most likely diagnosis?

Correct answer: Bullous pemphigoid Explanation The patient scenario is describing bullous pemphigoid (BP). Bullous pemphigoid is an autoimmune blistering disorder that most commonly occurs in older adults (typically those over the age of 60). Neurological disorders have been seen to have an association with development of bullous pemphigoid and Parkinson's disease being identified as an independent risk factor in development of it. Areas affected include the trunk, extremity flexures, and axillary and/or inguinal folds. There may be a prodromal phase where the patient experiences pruritic eczematous, papular, or urticarial-like skin lesions. Classic lesions of BP are 1-3 cm sized tense bulla on an erythematous, urticarial, or non-inflammatory base. The blisters will be plentiful and widespread as well as being severely pruritic in nature. Herpes zoster lesions are tingling and painful; there is an eruption of vesicles in a dermatomal presentation. These vesicles will eventually progress to pustules and then eventually crusts. Pemphigus vulgaris, also known as just simply pemphigus, is another type of blistering pathology and is very rare, making this diagnosis not as likely. This is an intra-epidermal blistering disease that can occur on the skin but also on the mucous membranes. This disease state typically is fatal, with mortality within 5 years after the initial episode. Etiology is unknown. The disease state appears as flaccid bullae, crusts, and erosions in crops or waves that very typically involve the mucous membranes. Nikolsky sign present (this is absent in bullous pemphigoid). Tinea corporis is not the most likely diagnosis. Tinea infections commonly have ring-shaped lesions with advancing, red, itchy, scaly borders and central clearing. This is not consistent with the lesions described above. A varicella rash possesses the following characteristics: pruritic, centrifugal, papular that change to vesicular (described as "dew drops on rose petals"), pustules, and finally progress to crusts.

Case Ico-delete Highlights A 72-year-old man is well known to your practice; he presents with a severely itchy rash. The patient's medical history includes Parkinson's disease for the past 10 years. At first, he thought the rash was just eczema, but the areas have progressed to significant hives, and itching has substantially worsened. On examination, you note multiple bullae that are 1 to 3 cm in size; they are tense, and they appear on an erythematous base. The bullae are noted to be located on the patient's trunk as well as the bilateral axillary and inguinal folds. Question Ico-delete Highlights Considering the most likely pathology, what would be considered the cornerstone of clinical intervention?

Correct answer: Corticosteroids Explanation The patient scenario is describing bullous pemphigoid (BP). Corticosteroids, whether topical or systemic, are considered the cornerstone of therapy. A topical corticosteroid (high potency) is considered the first-line agent. In regards to the elderly population, long-term use of systemic corticosteroids can still be used but with great caution due to the significant adverse reactions that can occur. Bullous pemphigoid is an autoimmune blistering disease that most commonly arises in older adults (typically patients over the age of 60). Neurological disorders have been seen to have an association with development of bullous pemphigoid and Parkinson's disease being identified as an independent risk factor in development of it. Areas affected include the trunk, extremity flexures, and axillary and/or inguinal folds. There may be a prodromal phase where the patient experiences pruritic eczematous, papular, or urticarial-like skin lesions. Classic lesions of BP are 1 - 3 cm sized tense bulla on an erythematous, urticarial, or non-inflammatory base. The blisters will be plentiful, widespread, as well as being severely pruritic in nature. The general approach to treatment of BP consists of 3 major goals: decrease blister formation and pruritus, promoting healing of blisters and erosions, and improve quality of life. Patients and their caregivers/support system should be educated on general skin care. Daily rupturing of the tense blisters has been found useful in reduction of lateral extension of the blister edges that seem to occur. Patients are instructed to cleanse blisters with alcohol prior to rupturing them with a sterile needle, leaving the actual epithelial layer in place once fluid escapes the blister. Any areas of erosions should be covered with non-stick dressings to reduce pain and infection. Appropriate education to stay alert for infection of the areas should also be conveyed. Agents such as azathioprine, dapsone, and mycophenolate mofetil have very few benefits in patients with BP; studies confirm this conclusion. No studies currently exist that support the use of methotrexate in the treatment of BP.

Case A 21-year-old woman presents with painful bumps on her shins. The patient stated they appeared when she "got the flu last week". She has been using acetaminophen for the discomfort and to manage her flu symptoms. No other medications have been taken. On clinical exam of the anterior aspect of the patient's legs, the nodules appear symmetrical, red, and shiny; they are 3 cm in diameter. Palpation elicits pain. Question What is the most likely diagnosis?

Correct answer: Erythema nodosa Explanation Erythema nodosa is an acute inflammatory condition characterized by painful nodules on the anterior aspect of the legs. It is often symptomatic of a bacterial, viral, or fungal disease or drug eruption. This occurs most often in women and between the ages of 20 - 30 years. Clinical features include acute fever, malaise, and joint pain. Lesions are nodular, painful, red, and shiny. The symptoms last 2 weeks and heal without scarring. Urticaria is chronic or acute and is characterized by wheals and papules. Itching and prickling sensations are constant. Both sexes are affected equally and it's often seen in childhood or teen years. The presenting areas are the arms, legs, thighs, and waist. The most common cause is an allergy to medications, foods, or physical agents. The symptoms usually disappear within 6 months (acute) or can last longer (chronic). Erythema multiforme is characterized by macules, papules, vesicles, and bullae. It occurs secondary to a toxic influence. There may or may not be a prodromal period with a sore throat, diarrhea, and fever. The lesions are red macules or papules and are seen on the sides of the neck, face, legs, genitalia, and mucosa membranes. A typical lesion is a 'target' or 'iris' lesion. The course is generally 3 - 4 weeks. Erythema Ab Igne is often called toasted skin syndrome; it occurs secondary to exposure to heat from flames or heating appliances. The course is generally benign, but may have a potential for malignant changes. The lesions are red, mottled skin with hypo- or hyperpigmentation. After many years of constant exposure, hyperkeratotic papules, plaques, and ulcers may occur. Nummular eczema has round, coin-like (nummular) lesions; there is a distribution on the extensor surface of the extremities as well as the posterior portion of the trunk, buttock, and legs. Purulent drainage is not uncommon. Treatment includes topical steroids, systemic antibiotics, and antihistamines; exposure to water should be decreased, and drying agents are used when oozing is present.

Case A 22-year-old woman presents with a 3-day history of a rash on her legs. On physical examination, several extremely tender indurated nodules are noted, as well as red spots on her anterior lower extremities that are distal to her knees. She states that she cannot sleep with a blanket on her bed due to the discomfort it causes when the blanket rubs against her legs. Question What is the most likely diagnosis?

Correct answer: Erythema nodosum Explanation The clinical picture is suggestive of erythema nodosum. Common findings include indurated and very tender nodules on the anterior lower legs that tend to be bilateral and symmetric. Erythema multiforme lesions may develop over 10 days. Lesions progress from macules to papules to vesicles and bullae. Target-like lesions are typical. Erysipelas and cellulitis are acute spreading infections characterized by a hot red tender area of the skin; they often begin at a site of bacterial entry. Psoriasis lesions appear as silvery-white scales on an erythematous base; they are usually on the extensor surfaces.

Case A 20-year-old Caucasian college student comes in with a complaint of hypopigmented patches which appeared gradually during the summer. He reports no history of unprotected sex. Scaly patches are present, but no pruritus. On examination, he has hypopigmented patches over the face and chest, but no vesicles or pustules. Question What is the most likely diagnosis?

Correct answer: Tinea versicolor Explanation Tinea versicolor (Pityriasis versicolor) is a mild superficial infection of the skin. It is characterized by multiple, usually asymptomatic, scaly patches varying from white to brown in color. They are most frequently seen on the chest, neck, and abdomen and occasionally on the face. The condition is usually seen in young adults. Diagnosis of this condition is made on the basis of clinical findings. On microscopic examination of scraping from the lesion under a Woods lamp, yeast and short plump golden hyphae are seen. Treatment of T. versicolor involves topical therapy with selenium sulfide, imidazoles, and zinc pyrithione. Hypopigmented patches can be distinguished from vitiligo on the basis of appearance. Vitiligo usually presents as a loss and not just a lessening of pigmentation (as in T. versicolor), most commonly in the perioral region, wrists, and hands. Tinea cruris is confined to the groin region and the gluteal cleft and is associated with severe itching and a rash. The margins of the rash are sharp, with cleared centers. The area is hyperpigmented on resolution. Treatment involves the use of drying powders, ketoconazole cream, and in severe cases, systemic ketoconazole. Leprosy is associated with pale anesthetic macular skin lesions. There is associated nerve thickening with the associated anesthesia. There is also accompanying history of living in an endemic area in childhood. Tinea capitis is a disease caused by superficial fungal infection of the skin of the scalp, eyebrows, and eyelashes, with a propensity for attacking hair shafts and follicles. The disease is considered to be a form of superficial mycosis or dermatophytosis. Several synonyms are used, including ringworm of the scalp and tinea tonsurans.

Case A 15-year-old girl with a recent history of epilepsy presents 1 week following the evaluation by a neurologist for prophylactic anti-seizure medication. She was placed on phenytoin by the neurologist at that time. Today, she complains of a 3-day history of fever, myalgias, nonproductive cough, sore throat, and several episodes of nonbilious vomiting. She also states that a nonpruritic rash has developed, first at her hands and feet, but has now begun to "move up her arms and legs." The rash is pictured in the supporting image. Question What is an additional expected clinical manifestation in this patient?

Correct answer: Mucosal erosions Explanation This patient's most likely diagnosis is erythema multiforme. The initial lesion is a dull-red, purpuric macule or urticarial plaque that expands slightly to a maximum of 2 cm over 24-48 hours. In the center, a small papule, vesicle, or bulla develops, flattens, and then may clear. An intermediate ring develops and becomes raised, pale, and edematous. The periphery gradually changes to become cyanotic or violaceous and forms a typical concentric, "target" lesion. The Nikolsky sign is negative; the top layers of the skin do not slip away from the lower layers when slightly rubbed. The lesions are symmetrical, predominantly on the acral extensor surfaces of the extremities, and they spread centripetally to involve the abdomen and back. Lesions may also coalesce and become generalized. The palms, neck, and face are frequently involved. Mucosal involvement is present in as many as 70% of patients with erythema multiforme. Erosions of the oral mucosa may result in difficulty in eating, drinking, or opening the mouth. The degree is usually mild, limited to one mucosal surface. The most common sites of mucous membrane involvement in order of frequency are the oropharynx (lips, palate, and gingiva most often affected), conjunctivae, genitalia, anus, tracheobronchial tree, esophagus, and bowel. Eye involvement (10%) is usually mild and may manifest as red conjunctivae, chemosis, and lacrimation. The genital areas may have painful, hemorrhagic bullae and erosions. More severe erosions of at least 2 mucosal surfaces are seen in erythema multiforme major and are characterized by hemorrhagic crusting of the lips and ulceration of the nonkeratinized mucosa. Constipation is not an anticipated finding. Rather, profuse diarrhea occurs from involvement of bowel mucosa. Shortness of breath and hyperventilation with mild hypoxia may result from anxiety or tracheobronchial epithelial involvement. Manifestations of fluid overload are not expected findings. Dehydration occurs and may range from mild to massive as a result of evaporation through open skin lesions, poor oral intake secondary to oropharyngeal mucous membrane involvement, profuse diarrhea, and insensible losses secondary to hyperthermia.

Case A 63-year-old male Caucasian farmer presents to your office for a routine physical exam. His past medical history is positive for hypertension and he has been well maintained on a regimen of a beta-blocker for many years. He continues to feel well and is very active on his farm, working outside for hours each day. Examination of his skin reveals a 4-mm papule on the forehead; it is pearly white and rather flat. There are small telangiectatic vessels seen in the lesion, with a small amount of scale in the center. The patient believes the lesion has been there for a few months, but he is not sure. Question What is the most likely diagnosis?

Correct answer: Nodular basal cell carcinoma Explanation The correct response is nodular basal cell carcinoma. Basal cell carcinoma is the most common skin cancer, and can develop at any age, although most patients are over 40 years of age. Basal cell carcinoma most often develops on the face, and it is very uncommon in African-American individuals. Basal cell carcinoma generally does not metastasize, although it can be very locally invasive, advancing by direct extension, both peripherally and deeper. If a basal cell carcinoma is not treated, or is inadequately treated, it can destroy the normal tissue of the face, even destroying underlying bone and brain tissue over time. Nodular basal cell carcinoma is the most common form of basal cell carcinoma. These lesions begin as a papule with a pearly white border. Telangiectatic vessels can be seen, especially as the lesion grows. The centers of nodular basal cell carcinomas will often go through cycles where they ulcerate, bleed, crust, and scale over; the patient may mistakenly assume that the lesion is healing. The treatment of nodular basal cell carcinoma depends on the size and location of the cancer. Nodular basal cell carcinomas on the forehead or cheeks that are smaller than 2 cm in diameter are usually treated with electrodesiccation and curettage with good results. Larger cancers in these areas are treated with excision, because electrodesiccation of large lesions can cause unfortunate scarring. Basal cell cancers around the nose, eyes, ear, and mouth require referral to a dermatologic surgeon for Mohs micrographic surgery. All patients with basal cell carcinoma should be followed for at least 5 years, since approximately 1/3 of patients will develop a 2nd basal cell carcinoma within 5 years. There are other types of basal cell carcinoma tumors. Pigmented basal cell carcinoma is a nodular basal cell cancer with melanin. These lesions have a brown, black, or bluish hue. Because of their pigment, these lesions may be mistaken for a melanoma, although a pigmented basal cell carcinoma will have the characteristic pearly white border as with the other basal cell cancers; biopsy will confirm the diagnosis. Superficial basal cell carcinoma is the least aggressive type of basal cell carcinoma. This type often occurs on the extremities or trunk and spreads superficially, with deeper extension occurring only after several years. These lesions present with a red, scaling plaque that may be mistaken for eczema or psoriasis, but examination will reveal the characteristic pearly white border of a basal cell cancer. Cystic basal cell carcinoma presents as a round, smooth cystic mass. Morpheaform (sclerosing) basal cell carcinoma presents with a waxy, firm yellowish or white mass on the surface that blends with the surrounding skin, but it often has a deep involvement with wide extension. Nevoid basal cell carcinoma syndrome is a rare genetic disease; it has an autosomal dominant mode of inheritance with high penetrance, but variable expressivity. Patients with this syndrome present with multiple basal cell carcinomas at birth or in early childhood, a variety of skeletal abnormalities, small pits on the palmar and plantar skin surfaces, jaw cysts, and ectopic calcification of the falx cerebri. 70% of patients with the nevoid basal cell nevus syndrome will display characteristic facial abnormalities. This patient's lesion is not a sebaceous hyperplasia, although both lesions can develop small blood vessels on the surface. The telangiectasias seen with basal cell carcinoma are distributed in a haphazard fashion, while those vessels in senile sebaceous hyperplasia are seen only between the small yellow lobules. The pearly border around this patient's lesion is diagnostic for basal cell carcinoma. Actinic keratoses are sun-induced premalignant lesions that present with adherent scale, but they would not have the pearly border characteristic of basal cell carcinoma.

Question A 73-year-old male presents with a complaint that his toenails are thick, hard to cut, discolored, and dystrophic. A KOH culture confirmed a fungal infection. What is the most likely diagnosis?

Correct answer: Onychomycosis Explanation The term used for ingrown nail is onychocryptosis. Onychauxis is used to define a thickened, overgrown nail. A hooked or incurvated nail is defined by onychogryphosis. Onycholysis is the loosening or separation of all or part of the nail from the nail bed. Onychomycosis is a disease of the nail caused by a fungal infection

Case A 62-year-old man who is well known to you presents with a severely itchy rash. The patient's medical history includes Parkinson's disease for the past 10 years. The patient initially believed the rash to be eczema, but the areas have progressed to significant hives, and the itching has become drastically worse. On examination, you note multiple bullae that are 1 to 3 cm in size, tense, and appear on an erythematous base. The bullae are located on the patient's trunk as well as the bilateral axillary and inguinal folds. Question Although you have an idea of the pathology, what would be considered the gold standard diagnostic study to make the definitive diagnosis?

Correct answer: Perilesional skin biopsy for direct immunofluorescence Explanation The patient scenario is describing bullous pemphigoid (BP). Bullous pemphigoid is an autoimmune blistering disease that most commonly arises in older adults (typically those over the age of 60). Neurological disorders have been seen to have an association with development of bullous pemphigoid and Parkinson's disease being identified as an independent risk factor in development of it. Areas affected include the trunk, extremity flexures, and axillary and/or inguinal folds. There may be a prodromal phase where the patient experiences pruritic eczematous, papular, or urticarial-like skin lesions. Classic lesions of BP are 1 -3 cm sized tense bulla on an erythematous, urticarial, or non-inflammatory base. The blisters will be plentiful, widespread, as well as being severely pruritic in nature. Diagnosis is aided with what is considered the gold standard, a perilesional skin biopsy for direct immunofluorescence (DIF). Lesional skin biopsy for hematoxylin and eosin staining is also a laboratory considered to aid in confirming diagnosis of BP; however, it is not considered the gold standard. Serum specimens for indirect immunofluorescence and serum specimen for enzyme-linked immunosorbent assay are used when suspecting BP, but only to detect circulating basement zone antibodies and again are not considered the gold standard test. Examining a specimen sample for antibodies is also not the most appropriate answer.

Case A 21-year-old man presents with itchy skin changes. He works as a lifeguard. About a week ago, he noticed red, round patch on his belly that spread to his trunk and legs. He denies recent infections, allergies, and illnesses; he does not take any medications, and he admits that he occasionally smokes marijuana. The rest of his personal and family history is noncontributing. On examination, you find round and annular, scaly, pruritic, papulosquamous changes on his torso and legs. There are no changes on his mucosa, and the rest of physical examination is within normal limits. Question In order to make a diagnosis, what should be your next step?

Correct answer: Potassium hydroxide preparation Explanation The clinical picture of skin changes and the patient's occupational history suggest tinea corporis. You should confirm your diagnosis with potassium hydroxide preparation of skin scrapings that will demonstrate the presence of fungal hyphae. If the potassium hydroxide test is negative, then you should consider a skin biopsy to identify the pathohistological patterns of the other dermatoses. The Venereal Disease Research Laboratory test (VDRL) is a non-treponemal screening test for syphilis. Secondary syphilis often appears as symmetric pink, reddish, or brown papules on the trunk and proximal extremities, including the palms and soles. However, they are non-itchy, and mucosal structures and other parts of the body are also involved. Secondary syphilis occurs 2 to 10 weeks after the primary chancre. Prick test with intradermal injections are done to assess allergies to drugs or bee venom. Your patient has no such a history. Tzanck smear is sometimes used to the diagnosis of chickenpox (highly contagious viral disease caused by varicella zoster). Diagnosis is usually clinical, but there is no reason to think about chickenpox in this patient. Chickenpox usually starts 10 - 21 days after a contact with infected person; it begins as an itchy vesicular rash on the body and head. Typically, you will find crops and lesions at various stages of healing.

Case A 36-year-old man presents with an itchy rash that has developed on his left hand. You examine the patient's hand. The rash appears to be on the sides of the fingers and on the palm. The lesions are vesicular in nature and have a tapioca-like color to them. Question What is appropriate treatment for your patient?

Correct answer: Short course of tapered prednisolone Explanation The clinical picture is suggestive of pompholyx: vesiculobullous hand eczema, formerly known as dyshidrotic eczema. Numerous treatment options are available, including topical and systemic corticosteroids, intralesional triamcinolone, or tapered prednisolone. Antibiotics (e.g., erythromycin) should be used only if there is suspected secondary infection; they are not indicated in this patient. Terbinafine is used in the treatment of fungal infections. Dyshidrotic eczema is not caused by a fungus. Coal tar and salicylates are not indicated for treatment in this case.

Question An 8-month-old male infant presents with rashes over the scalp and eyebrows. Physical examination shows a dry, scaly, and crusting lesion over the scalp, eyebrows, and nape area. He is comfortable, so his mother presumed that it was not itchy at all. He has been breastfed up to the present, and he started solid food at about 5 months old. There are no other signs and symptoms noted. Bowel movement and urination are normal. Developmental milestones are consistent with age. What is the most likely diagnosis?

Correct answer: Seborrheic dermatitis Explanation The clinical picture is suggestive of seborrheic dermatitis, which is a chronic inflammatory disease common in the pediatric age group especially during infancy and adolescent. The etiology is unknown, but it has been attributed to stressful situations, poor hygiene, and excessive perspiration. Malassezia furfur has also been implicated as a causative agent. It is clinically manifested as a diffused or focal scaling and crusting lesion seen over the scalp (called cradle cap), face, eyebrows, neck, axilla, retroauricular, and diaper areas. It is non-pruritic and extremely common among HIV-infected infants and adolescents. Treatment is the use of an anti-seborrheic shampoo, such as selenium sulfide, sulfuric acid, zinc pyrithione, and tar for scalp lesion. Topical antifungals are effective against M. Furfur. Inflamed lesions usually respond to topical corticosteroids. Seborrhea has a shorter course and responds much more rapidly to treatment than atopic dermatitis. Atopic dermatitis is primarily an IgE mediated allergic disorder; it is characterized by erythematous, weepy patches on the cheeks, followed by subsequent extension to the face, neck, wrists, hands, abdomen, and the extensors surface of the extremities. They are extremely pruritic. Secondary infection is common and may be extensive. 80% of patients have serum IgE concentrations elevated 5 - 10 fold above normal. The disease most often begins in infancy but has the tendency to undergo remission at 3-5 years of age. Diagnosis is mainly clinical. A family history of asthma, hay fever, or atopic dermatitis; increased IgE level; eosinophilia; and being Caucasian all support the diagnosis. Treatment requires control of environmental precipitants. Local therapy is the mainstay of management; therapy includes wet dressings (Burrow's solution), topical corticosteroids creams, or lotion. Infection is treated with systemic antibiotics. Psoriasis is a common skin disorder during the initial 2 years of life. Girls are more frequently affected, and 50% have a family history. It is characterized by erythematous papules that coalesce to form plagues with irregular borders that appear as silvery or yellow-white scale. Their removal may induce a pinpoint bleeding that is called Auspitz sign. Lesions may be seen anywhere, but they are most often found in the scalp, knees, elbows, umbilicus, and genitalia. Diagnosis is mainly clinical. Treatment depends on the age of the child, sites of involvement, and extent of the disease. Tar preparations may be added to the daily bath. Occasionally, sunlight may have an adverse rather than beneficial effect. Salicylic acid may help remove the scales. Topical corticosteroids may be effective, but they should be used with caution. Candidiasis (Moniliasis) is caused by Candida albicans, which is a dimorphic yeast; it is not part of the normal skin flora. Certain conditions, such as cold temperature or humidity as well as alteration of the normal flora by the use of antibiotics, may promote overgrowth of the yeast. It is clinically manifested by intensely erythematous, confluent plaques; they have scalloped borders. The hallmark of candidal infection is the numerous papules and vesicopustules, which are satellite pustules that stud the contiguous skin. Diagnosis is by KOH preparation or culture. Treatment is the application of anti-candidal agents (nystatin, miconazole, clotrimazole, and/or ketoconazole). Lichen simplex chronicus is characterized by chronic, pruritic, eczematous, circumscribed solitary plaques that are usually hyperpigmented and lichenified. It is most commonly seen in the posterior aspect of the neck, dorsum of the feet, wrists, and ankles. Topical fluorinated corticosteroids are helpful, and constant irritation of the skin should be avoided.

Case A 47-year-old Caucasian man presents because he is worried that he has cancer on his back. He had his shirt off at a company picnic and someone suggested that he should get a spot on his back checked. On physical exam, there is a single lesion about 3 mm in size on his posterior (back); it is brown/black in color and has a "stuck-on" appearance. It is sent to pathology. Question What is your initial diagnosis?

Correct answer: Seborrheic keratosis Explanation The clinical picture is suggestive of seborrheic keratosis, which consists of benign plaques that are 3 to 20 mm in diameter; their color can range from beige to brown, or even black. They feel greasy and have a "stuck on" appearance. Onset is rare before the age of 30 years. Acrochordons (skin tags) are small benign tumors that form primarily in skin creases. They are often raised from the surface of the skin on a fleshy stalk known as a peduncle. Dermatosis papulosa nigra is a condition of multiple seborrheic keratosis lesions on the face of dark-skinned individuals. Actinic keratosis consists of single or multiple, discrete, dry, rough, adherent scaly lesions that occur on any habitually sun-exposed skin of adults. Dermatofibromas are benign skin growths; they are typically found on the legs and range in size from about 0.5-2 cm. They can be described as hard papules that may appear in a variety of colors, but they are usually brownish to tan.

Case A 32-year-old woman with vitiligo presents for detailed diagnostic evaluation 1 month after the last episode of fever, productive cough, and infiltrate seen on chest X-ray that is consistent with pneumonia; she was successfully treated with fluoroquinolones. Currently, she feels healthy, and she does not take any medication. Her past medical history is significant for recurrent otitis media, respiratory infections, episodes of upset stomach, diarrhea, and bloating with sulfurous burping. Routine laboratory examinations show only mild macrocytic anemia. Question What should be your next step in the diagnostic evaluation of her condition?

Correct answer: Serum immunoelectrophoresis Explanation Your patient has autoimmune disease (vitiligo) and frequent infections suggestive of a defect in humoral immunity. Therefore, the 1st step should be to evaluate immunoglobulins by serum immunoelectrophoresis. Your patient currently shows no signs of systemic infection, and a blood culture would be useless. Schilling test may be used in selected cases of pernicious anemia; it is used to demonstrate the absence of intrinsic factor when other causes are excluded. Your patient has no signs and symptoms of neurological deficits: macrocytosis is more likely caused by problems with intestinal absorption of vitamin B12. Diagnosis of CVID is based on the exclusion of other causes of humoral immune deficiencies. You may search for the particular genetic defects when a patient has low total serum concentration of IgG; poor or absent response to immunization; and when other primary immunodeficiencies are excluded (e.g., X-linked agammaglobulinemia, IgA deficiency, DiGeorge syndrome, ataxia telangiectasia, or other T-cell defect syndromes). Your 1st step, however, is to detect the presence of an immune deficiency. A sputum culture in an asymptomatic patient is not indicated. Even in healthy lungs, sputum cultures demonstrate both normal and pathogenic flora. Sputum culture is performed in cases of persistent productive cough, fever, hemoptysis, and/or radiological indications of pulmonary infection; it is also used to diagnose the causes of pneumonia, bronchiectasis, pulmonary abscess, and other respiratory infections.

Case A 32-year-old man presents with symptoms of a skin rash. He has had the rash on his left arm for about a month and has tried only over-the-counter hydrocortisone cream without change. He denies previous skin lesions and states that this one is asymptomatic. The physician assistant (PA) who sees the patient immediately tells him that he has tinea corporis. Question How should the lesions be described in his office visit note?

Correct answer: Sharply defined annular papules with central clearing Explanation Sharply defined annular papules with central clearing is an accurate description of a skin lesion caused by tinea corporis, otherwise known as "ringworm." Tinea corporis is a type of dermatophyte skin infection, and the most common etiologic organisms are Trichophyton mentagrophytes and Microsporum canis. The skin lesions are annular, well-demarcated, and have a central clearing. They may appear scaly and be pruritic, or patients may be asymptomatic. Pustules or vesicles can also form at the edges of the lesion. Diagnosis can be confirmed by scraping skin scales from the border of the lesion, dissolve them in 20% KOH, and examine under the microscope for presence of hyphae. Treatment is always with a topical antifungal. 'Well-marginated, erythematous half-moon-shaped plaques' is not the correct answer. This description is accurate for lesions caused by tinea cruris. Tinea cruris occurs in the crural folds and can spread to the upper thighs. 'Polycyclic hypopigmented macules with fine scaling' is not the correct answer. This type of lesion is seen in patients with tinea versicolor, which is caused by a yeast-like fungus known as Pityrosporum orbiculare. 'Umbilicated white papules formed in groups' is not the correct answer. This type of description is used for Molluscum contagiosum, which is a poxvirus that causes pale papules to form. Molluscum is commonly seen in infants, young children, and sexually active teenagers. 'Erosions covered by honey-colored crusts' is not the correct answer. This type of description is used for impetigo. Staphylococci and group A streptococci are the most common etiologic agents in impetigo, invading the upper epidermis and forming pustules.

Case Ico-delete Highlights An 82-year-old man presents for evaluation of an itchy hive-like rash on his abdomen. He has had it for months. Over the last few days, the patient reports that the character of the rash has changed: it now resembles blisters. Other than some skin irritation, he feels healthy. He has tried multiple over-the-counter topical treatments, but they have been unsuccessful in treating the rash. He denies any changes in soaps, lotions, laundry detergents, or anything else that may have come in contact with his skin. He also denies unusual travel, pets, or hobbies. No close contacts have reported a similar condition. He takes no medications, and he does have any chronic illnesses. On physical exam, multiple clusters of bullae are noted across the trunk bilaterally, with some distribution on both anterior and posterior surfaces. There are still some remaining pink-red lesions; they are scattered among the bullae. The bullae are 1-3 cm in size; they are tense and do not easily rupture. They do not extend into normal skin with pressure. Pressure on the normal skin does not produce a blister. The remainder of his physical exam is normal. Question What test would confirm the most likely diagnosis?

Correct answer: Skin biopsy with immunofluorescence testing Explanation This patient presents with bullous pemphigoid (BP), a benign autoimmune skin disorder characterized by the presence of bullae (or blisters). BP is relatively rare and tends to affect the elderly. The preceding rash is typically pruritic, and it appears as urticarial plaques, possibly resembling erythema multiforme. This stage may last for months, then the skin blisters. When the blisters rupture (typically within 1 week), the skin heals. Unless the BP is extensive and generalized, the prognosis is good. Skin biopsy with direct and indirect immunofluorescence and light microscopy is the best test for confirming BP. Serology for parvovirus B19 is the appropriate confirmatory test for patients suspected to have fifth disease (erythema infectiosum), which presents with the characteristic "slapped cheek" rash. It is not helpful in cases of BP. A skin scraping with potassium hydroxide (KOH) would be helpful in diagnosing a fungal skin condition. While pruritus can be associated with fungal infections, this patient's presentation with bullae does not suggest a fungal etiology. A viral culture of the fluid inside the bullae would not reveal any virus, as BP is an autoimmune condition. Herpes family infections could be on the differential, especially with a blister-like appearance, but herpes vesicles are typically much smaller, and none of the herpes infections fit the presentation and distribution of this patient's "rash." A Wood lamp examination can be helpful in evaluating several dermatologic conditions, such as vitiligo, as well as some bacterial and fungal infections. It uses ultraviolet light. A Wood lamp exam can also be performed on the eyes when checking for corneal abrasions, but it is not helpful in cases of BP.

Case A 69-year-old Caucasian woman presents with a painful lesion on her lip that has been rapidly increasing in size and bleeding for the past month. She gives a long history of sun exposure and several blistering sunburns in her adolescence. On examination, there is a tender, pink papule 2 cm in diameter on her lower lip. Question What is the most likely diagnosis?

Correct answer: Squamous cell carcinoma Explanation The most likely diagnosis in this patient is squamous cell carcinoma. Squamous cell carcinomas (SCC) arise from epithelial Malpighian cells. Patients may present with a red papule or crusted plaque with ill-defined margins on the lips, ears, neck, hands, or arms. It may ulcerate, invade the underlying tissue, and metastasize. Treatment includes surgical excision, curettage and electrodesiccation, cryosurgery, and Mohs surgery. Patients with basal cell carcinomas (BCC) can present with a shiny pearly nodule (noduloulcerative BCC), or with erythematous scaling plaques (superficial BCC), or with a solitary flat yellowish plaque (morpheaform BCC). There are several types of malignant melanomas, including: Superficial spreading melanomas—plaques with blue-black spots Nodular melanomas—rapidly growing dark papules Lentigo maligna melanomas—brown macules with scattered darker spots on sun-exposed areas, such as the face Acral lentiginous melanomas—most common form in African American patients; they develop on the palms, sole, subungual skin, and mucous membranes There are several forms of Kaposi's sarcoma (KS), including: Classic KS—purple plaques or nodules on the lower limbs of elderly men of Mediterranean origin without AIDS AIDS-related form—widely disseminated purple papules or plaques on the skin, mucous membranes, and viscera Lymphadenopathic form—involves lymph nodes and skin In Paget's disease of the breast, patients present with a unilateral, sharply marginated, red, scaly rash or erythematous crusting affecting the nipple and areola. There may also be an underlying mammary duct carcinoma.

Case A 45-year-old woman presents with a lesion on her calf. She states that she has had it for a while and that it has not gotten bigger, but it does not heal and it bleeds occasionally. On exam, the lesion is a pink, sharply demarcated, scaling plaque. Question What is the most likely diagnosis?

Correct answer: Squamous cell carcinoma Explanation This is a clinical picture of squamous cell carcinoma (SCC). Most SCC appears as a sharply demarcated, scaling, or hyperkeratotic macule, papule, or plaques. Most often they are asymptomatic, but may bleed. SCC is most often caused by UV radiation (sun exposure) or HPV infection. Actinic keratosis consists of single or multiple, discrete, dry, rough, adherent scaly lesions occurring on the habitually sun-exposed skin of adults. They are described as adherent hyperkeratotic scale, skin-colored, yellow-brown, or brown. Often, there is a reddish tinge and it is rough, like coarse sandpaper, "better felt than seen" on palpation with a finger. Malignant melanoma is usually described using the ABCDE pneumonic: Asymmetry, Border, Color, Diameter, and Evolution. Melanomas will be asymmetric, have irregular borders, be multi-colored (brown, black, gray, or pink), have a diameter > 6.0mm, and be elevated. Verruca wart is known as a common wart, and is described as a firm papule, 1 to 10 mm or (rarely) larger, hyperkeratotic, with clefted surface, with vegetations, and skin colored with "red or brown dots." The reddish-brown dots are a diagnostic marker for verruca warts.

Question A 68-year-old Irish farmer presents for his annual physical examination. He continues to smoke 10 cigarettes per day. He has no complaints, but an erythematous, scaly, nontender nodule measuring 0.5 centimeters is noted on his left lower lip. There are no surrounding telangiectasias. The nodule is firm, ill-defined, and fixed to the underlying tissue. It does not blanch with pressure. What is the most likely diagnosis of this lesion?

Correct answer: Squamous cell carcinoma Explanation The lesion described is very characteristic of squamous cell carcinoma (SCC). The patient is of Celtic descent with an outdoor profession, which puts him at increased risk for developing this type of lesion. Hemangiomata blanch under pressure. Benign nevi typically have a well-defined border. Actinic keratoses (AK) are precursors to SCC and are typically hyperkeratotic. It can be difficult to distinguish between AK and SCC clinically, but this lesion is firm and fixed to underlying structures. While basal cell carcinomas are also more common on sun exposed areas, they often have surrounding telangiectasias and a rolled border.

Case A 70-year-old man presents for a routine physical examination. He has been in a good state of health, but he is concerned about a growth on his right leg. He states that it has been present for about 1 year; it disappears once in a while, but then it returns, sometimes covered with white scales that then dropped off. On further questioning, he gives a history of spending countless hours at the beach body surfing as a youth; he rarely used sunscreen. On examination of the skin, you note a round pinkish spot measuring about 0.5 cm in diameter on the posterolateral aspect right leg. The lesion is raised; it feels dry and rough to touch. You suspect skin cancer, and you refer the patient to a dermatologist who decides to cauterize the lesion with liquid nitrogen. Within 3 weeks, the area becomes crusted, shrinks, and falls off. 2 weeks thereafter, the spot returned, and after a month, it develops into an open sore that will not heal. Question What is the most likely diagnosis?

Correct answer: Squamous cell carcinoma (SCC) Explanation The hallmark of a squamous cell carcinoma is an open sore that will not heal. It is the 2nd most prevalent form of skin cancer, with a yearly incidence in the United States of about 200,000 cases per year. Most occur in elderly subjects on areas exposed to the sun, and they are more prevalent in light-skinned individuals with an early history of time spent outdoors as a youth. Most of them are slow-growing, but if left untreated, they will eventually penetrate the dermis and invade underlying tissue. Actinic keratosis is incorrect. AK is a benign reddening and roughening of the skin found in sun-exposed areas, most commonly in fair-skinned elderly persons. It may present as a single spot or it may be spread over a wider area, giving the affected person a blotchy appearance. Some 15% will transform into a cancer, which is almost always a squamous cell carcinoma. Basal cell carcinoma is incorrect. BCC is the most common of all the skin cancers. It has been estimated that some 800,000 Americans are affected at any given time. It too appears to be sun-induced and is most commonly found on the exposed areas of skin in lightly pigmented persons. They have several modes of presentation, the most common being a shiny, pearly, translucent nodule. Basal cells carcinomas arise from the basal cells of the epidermis, and they almost never metastasize. Nodular melanoma is incorrect. Nodular melanoma is 1 of the 4 types of melanoma. It is the most aggressive of all the melanomas. Initially, it is typically recognized as a pigmented (usually dark brown or black) bump or node. Unlike most melanomas, it does not initially spread superficially, but it rapidly grows into deeper tissues, infiltrating the subcutaneous lymphatics and metastasizing to the lymph nodes. The prognosis is understandably poor. Contrary to common opinion, nodular and other types of melanomas (with the exception of lentigo melanoma) are not usually induced by exposure to the sun, and more melanomas are found on the trunk or limbs than on the face or other exposed areas. Herpes infection is incorrect. Herpes may exist as an open sore, but it is rarely an isolated lesion. It is never preceded by a rough, pinkish lesion; instead, it is preceded by a vesicular lesion.

Case A 40-year-old woman presents with a 6-month history of itchy lesions on her left ankle. On examination, her left leg is erythematous with scaly patches and an ulcer over the medial malleolus; she also has bilateral varicose veins. Question What is the most likely diagnosis?

Correct answer: Stasis dermatitis Explanation Stasis dermatitis is characterized by erythematous, pruritic scales with oozing patches; they are usually found over the medial aspect of the ankles. Hyperpigmentation and stasis ulcers may also be present. The dermatitis develops on the lower limbs due to chronic venous incompetence. Management includes using topical glucocorticoids and emollients, as well as avoiding irritants. Patients should also be encouraged to wear compression stockings and elevate the affected limb as much as possible. Lichen planus is characterized by pruritic, polygonal papules; they are flat-topped and violaceous in color, and they have a network of gray lines (Wickham's striae) on the surface. They commonly develop on the wrists, legs, and genitalia. Management includes using topical glucocorticoids, though many patients have spontaneous remissions. Asteatotic eczema is characterized by fine cracks that may be erythematous. It is usually found on the pretibial surfaces of elderly people during winter; it may also be pruritic. Management includes application of topical emollients and avoidance of irritants. Contact dermatitis in the acute phase is characterized by sharply demarcated, weeping, edematous, eczematous plaques. There is usually a history of contact with an allergen. Management includes avoidance of irritants and application of topical glucocorticoids. Lichen simplex chronicus consists of a well-circumscribed, scaly plaque with lichenification and hyperpigmentation due to chronic scratching or rubbing (which can also occur during sleep). It commonly involves the occiput, back of the neck, arms, dorsum of the feet, and ankles.

Case A 50-year-old woman presents with a 4-month history of well-defined white patches distributed symmetrically over her face, hands, and neck. The patches of discoloration easily burn when exposed to sun. Question If you perform a biopsy of the lesion, in what structure would you primarily expect to find melanocytes?

Correct answer: Stratum germinativum Explanation Your patient has vitiligo, which is the loss of the melanocytes. The stratum germinativum is the deepest layer of the epidermis; this is where melanocytes are primarily found. Basal cells in stratum germinativum can be considered the stem cells of the epidermis. Melanocytes are not a part of the stratum corneum. The stratum corneum is the outermost layer of the epidermis and consists of dead cells (corneocytes). The stratum lucidum is a layer of dead skin cells in the epidermis that is found only in areas of thick skin (i.e., the palms of the hands and the soles of the feet). Melanocytes determine the darkness of the stratum lucidum, but they are not a part of it. Melanocytes are not a part of the stratum granulosum. The stratum granulosum is a thin layer of cells in the epidermis that consists of granular cells (i.e., the keratinocytes that migrate from the stratum spinosum). The stratum spinosum is a layer of the epidermis where keratinization begins. Melanocytes are not a part of the stratum spinosum.

Case A 17-year-old girl presents with a painful, swollen, red left forearm. Symptoms began about 2 days prior to presentation; they followed a bug bite and initially presented as a small red area. She does not recall any other injuries to the arm. She denies fever, chill, nausea, and vomiting, but she states that she has had increased pain and swelling over the last day or so. She has no significant past medical history, and she had been well until this recent illness. Physical examination reveals a well-developed, well-nourished patient in mild distress. Her exam is significant for an erythematous, warm, shiny plaque area measuring approximately 5 cm in diameter with a well-defined border on her left forearm. Vital signs are as follows: blood pressure 110/72 mm Hg, pulse 78 beats per minute, temperature 99.6°F, and respiratory rate 14 breaths/min. Question What is the most common cause of this condition in the United States?

Correct answer: Streptococcus pyogenes Explanation The patient has erysipelas, and its most common cause in the US is Streptococcus pyogenes, which is a group A beta-hemolytic strep organism. Erysipelas may affect children or adults, and it was once known as St Anthony's Fire due to its appearance. The disease is usually the result of bacteria entering a break in the skin, and it can also occur in areas where there is an obstruction of the lymphatic system. The area becomes red, swollen, warm, and very painful, with a raised, clearly defined border. Other symptoms include fever, chills, and lymphadenopathy. It is diagnosed by the appearance of the skin, which includes extreme edema of the skin and vascular dilatation. It may also be diagnosed by blood tests and skin biopsies. Treatment is with antibiotics. The first line treatment is penicillin. Other treatments include dicloxacillin, cephalosporins, clindamycin, and erythromycin.

Question A 29-year-old woman presents to your general practice for a check up after noticing a mole on her back that is getting larger and darker. She has been a member of a tanning salon for 8 years. Normally, what is the destination of melanin pigment after being produced?

Correct answer: Taken up by keratinocytes Explanation The keratinocytes deposit the melanin granules on the superficial side of their nuclei and act as melanin stores. This is thought to protect the nuclei from UV radiation by absorbing it before it can reach the nuclei. The melanin is injected from the melanocytes into the keratinocytes by cytokine secretion. Langerhans cells are antigen-presenting cells in the dermis. They are not involved in pigmentation of the skin. Melanin is released by melanocytes. These are derived from the neural crest and are distributed among the keratocytes. The number of cells per unit area is the same in all races, and the difference lies in the rate of synthesis, accumulation, and degradation. Merkel cells are pressure receptors in the skin. Dendritic cells are antigen-presenting cells and are not involved in pigmentation.

Question A 36-year-old man presents with a foot rash. He states he has had it for a few weeks, and it usually begins to itch after he takes off his socks. You examine his foot and note maceration, peeling, and fissuring of the toe webs. What is appropriate management in this case?

Correct answer: Terbinafine Explanation The clinical picture is suggestive of acute tinea pedis, interdigital type. Treatment includes antifungals (i.e., terbinafine, itraconazole, etc). The others listed are not indicated for treatment. If left untreated, the infection could progress and get worse, become secondarily infected with bacteria, or can cause a spreading infection like cellulitis. Burow's solution is no longer indicated in the treatment of Tinea pedis. Selenium sulfide (or Selsun Blue) is not indicated for the treatment of tinea pedis. Washing with soap and water would not kill the fungal infection. Coal tar is not indicated for treatment in this case.

Case A retired investment banker on holiday in the United States presents for what appears to be suspicious skin lesion on his cheek. He has had a long-standing discolored patch; it has recently enlarged in size, and there is crusting. An examination of the lesion is highly suggestive of squamous cell carcinoma. After undergoing treatment, he is keen to undertake measures to prevent a recurrence. Question What would be the most appropriate advice regarding its prevention?

Correct answer: The use of protective clothing and avoidance of midday sun Explanation The use of protective clothing and avoidance of midday sun is the correct answer. This is the single best measure to avoid exposure to UV rays. In particular, darker clothing, wide-brimmed hats, and full sleeves are helpful. Suggesting the use of indoor tanning salons rather than exposure to direct sunlight is an incorrect response. Indoor tanning salons are associated with developing cancer, particularly if exposure begins at a younger age. They are to be discouraged in all forms. Strict avoidance of damaging UV-B rays is an incorrect response. While avoidance of UV-B rays is certainly helpful, UV-A rays are strongly associated with cancer as well. Use of SPF 50 and higher sunscreen is an incorrect response. SPF 15 is the recommended level sunscreen for use. Using zinc oxide paste is an incorrect response. Zinc oxide is an important additive to prevent UV-A rays. Other compounds, such as aminobenzoic acid, may also be used. Broad-spectrum sunscreens contain compounds that block both types of rays.

Case A 10-year-old boy presents with a 1-week history of a rash on his scalp. His mother states that the boy has been scratching his head often, and she notes that there are areas where his hair appears to have fallen out. She attempted to treat it with over-the-counter preparations, but his condition has not improved. The boy is active and otherwise healthy. The rash appears as erythematous, circular, scaly patches. There are areas where the hairs have become brittle and broken off. A scraping of one of the patches is placed in potassium hydroxide solution and shows hyphae. Question What is the most likely diagnosis?

Correct answer: Tinea capitis Explanation Tinea capitis, also known as ringworm, is a fungal skin infection that is more common in children. It is contagious and can be difficult to rid the patient of the infection. It is called ringworm due to the appearance of the infection on the skin; the lesions produced are ring-shaped and were once thought to be caused by a worm. It is caused by a fungus from the species Trichophyton and Microsporum. The lesions usually begin as round, often reddened, papules on the scalp. Over the course of a few days, the papules become scaly and coalesce to form a ring-like shape. The hair often breaks off in the affected area, making the infection more noticeable. There can be itching, swelling, and occasionally a purulent discharge. Diagnosis can be confirmed by microscopic analysis. A scraping of the area placed in potassium hydroxide solution will show hyphae if a fungal infection is present. Once this has been established, treatment can begin. Tinea capitis is treated with oral antifungal medications; topical preparations are usually not very effective. Griseofulvin, itraconazole, and terbinafine are most commonly used. Vitiligo causes a loss of pigmentation. Tinea versicolor is common in adults and adolescents. The most commonly affected areas include the chest, back, and shoulders. Psoriasis causes raised, scaly patches that do not fluoresce under UV light. Seborrheic dermatitis causes patches of thick, scaly, crusty skin; it is found mainly on the scalp. It can also appear in areas with oil glands (e.g., folds of skin, like those of the face, groin, and armpits).

Case An 11-year-old boy is seen for a patch of itchy skin on his forehead. He has just returned from a vacation in Florida where he visited with friends who have several pets. His mother states that he has not been sick lately. The lesion appeared 1 week ago overnight and has been growing slowly since then. The lesion is the size of a half dollar, annular with sharp margins, and lightly scaly. KOH examination of the scale shows hyphae. Refer to the image. Question What is the most likely diagnosis?

Correct answer: Tinea faciei Explanation Tinea faciei (corporis) is a fungal skin infection that can be caused by Microsporum, Trichophyton, or Epidermophyton. Other localizations of infection with those fungi are hair-bearing areas (tinea capitis), toes (tinea pedis - athlete's foot), and nails (tinea unguium). Applying KOH microscopic examination of untreated tinea often reveals hyphae. Some of the organism makes affected skin fluoresce bright bluish-green under a Wood's lamp. On non-hair-bearing skin, tinea can have the typical annular ("ringworm") appearance or present as deep inflammatory nodules or granulomas. Fungus-infected pets can be the source of infection, especially in tinea faciei. Due to pruritus, scratching can lead to contamination of other body parts as well as bacterial superinfection. Tinea faciei is treated with topical antifungals like imidazole, triazolo, haloprogin, tolnaftate, and ciclopirox olamine. If nails of hair-bearing areas are involved, systemic therapy with griseofulvin is necessary. Pityriasis rosea is a papular squamates dermatitis of unknown etiology, though viral infection is discussed. It usually begins with a medallion-like primary plaque on the trunk, which can be red or pink with a faded center and trailing scale. After a few days or weeks, annular or papular lesions 2 to 6 cm that are oval with the long axis parallel to the skin fold lines develop predominately on the trunk. The eruptions are moderately pruritic and look similar to secondary syphilis, but soles and palms are usually not involved. Lesions disappear after 3 to 8 weeks spontaneously. Treatment is symptomatic; oral antihistamines and local glucocorticoids are given to alleviate pruritus. Scabies is an infestation by the human itch mite, Sarcoptes scabiei, causing papulovesicular lesions usually sparing face, scalp, neck, palms, and soles (exception: infants). Burrows appear as dark, wavy lines, which are 3 to 15 mm long. Gravid female mites burrow beneath the stratum corneum and deposit 2 to 3 eggs per day. Larvae hatch from these eggs after about 2 weeks; then emerge to the surface of the skin, where they mate; and then invade the skin again. The first infestation is usually asymptomatic for several weeks. At re-infestation, there is an immediate hypersensitivity reaction. Patients usually complain about intense itching, which worsens when they are in bed or after a hot shower. Transfer from person to person requires close personal contact; the mites cannot survive outside the host for more than a day. To diagnose infestation, a needle is put into the skin parallel to the burrow. The mite is stuck on the needle and can be extracted for microscopical examination. Treatment consists of 5% permethrin cream, which is applied from neck down after a bath and left on for 8 hours. Possible bacterial superinfections caused by scratching need to be treated with antibiotics. Seborrheic dermatitis affects infants (cradle cap), as well as patients with Parkinson's disease, after cerebrovascular accidents, and with HIV infections. The lesions are itching, greasy scales overlying erythematous patches or plaques and usually affect face and scalp. On occasion, it can appear on the groin, axilla, submammary folds, and gluteal cleft. Treatment consists of topical glucocorticoids as well as shampoos containing coal tar and salicylic acid. Nummular eczema is a skin disease of unknown etiology affecting mostly middle-aged men. The lesions appear on the trunk as well the extensor surfaces of extremities (pretibial and dorsum manus) and are coin-shaped edematous papules that become crusted and scaly. Nummular eczema does not respond well to treatment with antihistamines and glucocorticoids. Sometimes antibiotic therapy (tetracycline or erythromycin) can be useful, even if there are no signs of bacterial superinfection.

Case A 15-year-old boy is at his dermatologist after returning from a trip to Mexico for 2 weeks during spring break. He presents with some concerns over discoloration on his skin. Physical exam reveals macules with a hypopigmented light brown appearance, which fluoresce yellow-green under Woods lamp. KOH prep displays hyphae and spores that appear with "spaghetti and meatball" appearance. Question What is the most likely diagnosis?

Correct answer: Tinea versicolor Explanation Tinea versicolor is the correct answer, as the patient presents with macules that are hypopigmented. Additionally, a KOH prep confirms the diagnosis with hyphae and spores present. It is a fungal infection which is why the hyphae and spores fluoresce. It is often more noticeable when one tans as this patient would have while just on spring break; this occurs because the yeast prevents the skin from tanning so it appears hypopigmented on tanned skin. Lastly, the condition is often found in tropical and subtropical areas in the world, such as Mexico. Roseola is not the correct answer as the patient does not have rose-pink macules that blanch. Additionally, spores and hyphae would not be present. Tinea pedis is not the correct answer as the patient does not present with a scaly itchy rash between the toes. Additionally, the patient would not have a hypopigmented rash on his extremities. Pityriasis alba is not the correct answer as this rash appears most commonly as a pale pink rash or plaques on the cheeks, which this patient does not have. Photodermatitis is incorrect because the patient does not present with any erythema on the body or any vesicles/bullae; therefore, the photodermatitis would be ruled out as a diagnosis.

Case A 24-year-old woman presents with a rash. She has never had a rash like it prior to presentation. You ask about symptoms, and the only thing she states is that it mildly itches. On physical examination, you note scattered multiple lesions with marked vesicles, pustules, sharply marginated plaques with central clearing, and crusting at the margins that produces annular configurations; the rash appears only on the arms, neck, and trunk. Question What is an appropriate treatment for this patient?

Correct answer: Topical clotrimazole applied BID and continued for 1-2 more weeks after lesions have cleared and applied at least 3 cm beyond the lesions' margin Explanation The appropriate treatment for this patient's Tinea Corporis is a topical azole, twice a day. Applications should continue for 1-2 weeks after the lesions have cleared. Applying 3 cm beyond the lesion will assure eradication of the fungus Soaking lesions and then removing loose tissue and applying steroids is a treatment for psoriasis. Coal tar is not effective for fungal infections. Wet dressing in Burow's solution is no longer a common treatment for acute tinea pedis, macerated interdigital type. PUVA photochemotherapy is not an indicated treatment for fungal infections.

Case A 35-year-old woman presents with intensely pruritic red papules over the anterior wrists. On close examination, the papules are shiny, they have a flat surface, and there is occasional central umbilication. A red plaque is seen along the scratch line at the anterior forearm. Question What is the best initial therapy for the patient?

Correct answer: Topical corticosteroid Explanation Lichen planus is characterized by flat-topped, pruritic, red-to-violaceous papules or plaques, mostly on the wrists. Lesions are polygonal with occasional central umbilication. As with psoriasis, the classic Koebner phenomenon also may be present in lichen planus. Most cases are idiopathic, but studies have shown an association with hepatitis C infection. The best initial therapy for localized lichen planus is a potent topical corticosteroid.

Case Ico-delete Highlights A 62-year-old man who is well known to you presents due to an itchy rash. The patient's medical history includes Parkinson's disease for the past decade. The patient believed the rash was just a mosquito bite at first due to the fact that it stayed small and confined to one space. This area has spread and now has characteristics of hives and intense itching. On examination, you note three bullae that are 1 to 3 cm in size, tense, and appear on an erythematous base. The bullae are noted to be located on the right trunk in a region that measures 5 centimeters in diameter. This patient states he has come to you for treatment of this issue before, around 5 years ago. Question What would be the best course of action to alleviate the effects of this patient's diagnosis?

Correct answer: Topical corticosteroids Explanation The patient scenario is describing bullous pemphigoid (BP). Bullous pemphigoid is an autoimmune blistering disease that most commonly arises in older adults (typically those over the age of 60). Neurological disorders have been seen to have an association with development of bullous pemphigoid; Parkinson's disease has been identified as an independent risk factor in development of it. Areas affected include the trunk, extremity flexures, and axillary and/or inguinal folds. There may be a prodromal phase where the patient experiences pruritic eczematous, papular, or urticarial-like skin lesions. Classic lesions of BP are 1-3 cm sized tense bulla on an erythematous, urticarial, or non-inflammatory base. The blisters will be plentiful and widespread as well as being severely pruritic in nature. Treatment plans for BP are to decrease the troublesome symptoms that are associated with the blistering lesions, resolve the present lesions, and then prevent any recurrences. A duration of 5 years since his last episode is a reasonable amount of time since this last episode of BP. Initial management is usually high-potency topical corticosteroids; these are in fact considered first-line treatment for moderate to severe generalized disease. Clobetasol is a great choice in this scenario. Studies have shown that both disease control and overall survival rates are better with topical corticosteroids versus the oral corticosteroids. Oral corticosteroids may be considered in the future as a possible treatment plan, as well as potentially adjuvant antibiotic therapy. Methotrexate may also be considered as treatment, but would not be the appropriate choice in this case scenario. In disease states that are resistant to these more common treatment plans, intravenous immunoglobulin (IVIG) may be considered, as well as plasmapheresis. Recently specific medications such as rituximab, etanercept, or omalizumab have been utilized in refractory cases. Antihistamines may be utilized for the pruritus; however, this choice will not help resolve the actual disease state.

Case A 28-year-old man presents with a 1-week history of an itchy, scaly rash on his elbows. When he scratches it hard, scales come off and the rash bleeds. Examination reveals the elbows are affected bilaterally. The lesions appear as 3-4 cm annular whitish scales on an erythematous base that is irregular and well-demarcated. The antecubital fossae are unaffected. Question What is an appropriate treatment?

Correct answer: Topical fluorinated glucocorticoids Explanation The clinical picture is suggestive of localized plaque psoriasis. Since the lesions are localized to the elbows with <5% skin involvement, the treatment includes topical fluorinated glucocorticoids. The appropriate treatment is applying glucocorticoids and covering the area with plastic wrap. Mineral oil is not indicated in the treatment of psoriasis, but it is indicated in skin diseases such as seborrheic dermatitis. Benzoyl peroxide gel is not indicated in the treatment of psoriasis, but it is indicated in conditions such as acne vulgaris. Topical metronidazole is not indicated in the treatment of psoriasis, but it is indicated in conditions such as perioral dermatitis and rosacea. Aluminum chloride hexahydrate is not indicated in the treatment of psoriasis, but it is indicated in conditions such as tinea pedis.

Case A patient presents with a rash that involves the hands and wrists. On examination of the rash, you note flat-topped, sharply defined purple papules measuring 1 to 10 mm in size. You apply mineral oil and examine the lesions with a hand lens and note white lines within the lesions. You also notice that he has nail dystrophy and a white lacy pattern on his buccal mucosa. Question What would be an appropriate treatment?

Correct answer: Topical glucocorticoids with intralesional triamcinolone Explanation The clinical picture is suggestive of lichen planus. Treatment includes topical glucocorticoids for cutaneous lesions. Intralesional triamcinolone is helpful for symptomatic cutaneous or oral mucosal lesions and lips. Topical ketoconazole is recommended for cutaneous fungal infections. Lichen planus is not caused by a fungus. Topical pimecrolimus is an effective treatment for eczema. Hydroxyzine would help with the pruritus associated with eczema. PUVA photochemotherapy is a recommended treatment for cases resistant to topical therapy or generalized Lichen planus. In this case, the patient has concurrent oral Lichen planus, which requires triamcinolone. Lugol's iodine, also known as Lugol's solution, is used as an indicator test for the presence of starches in organic compounds. It can be used to better visualize the mucogingival junction in the oral cavity. Similar to the method of staining oral mucosa, it may be applied to the vagina and cervix during colposcopy. Lugol's iodine, however, does not treat Lichen planus.

Case A 49-year-old woman is evaluated by dermatology for a lesion below her left eye; the image can be seen in the image. Its presence has been notable for 1 year, and it has been slowly enlarging. A diagnosis of basal cell carcinoma (BCC) is made, and her treatment options are explained to her. She absolutely refuses surgery, saying surgery was responsible for killing her father. Question What is a reasonable treatment option in this case?

Correct answer: Topical imiquimod Explanation Topical imiquimod is the correct answer. It acts as an immune modulator, enhancing the effect of the host immune system in fighting the malignant cells. When applied daily for 6 - 8 weeks, it results in a complete cure in between 70 - 90% of patients. It remains a viable option for low-risk superficial basal carcinoma, particularly in younger people. Micrographic surgery is an incorrect response. Mohs' micrographic surgery, given as a distractor option, is the standard of care for basal cell carcinoma. In this technique, the base and edges are microscopically examined to verify sufficient margins before the surgical repair of the site. If the margins are insufficient, more is removed from the patient until the margins are sufficient. Because a surgical option is not favored by the patient, it is contraindicated. Topical betamethasone is an incorrect response. It is not utilized in BCC. Intravenous 5-fluorouracil is an incorrect response. Topical 5-fluorouracil is used as a 2nd-line topical drug, but imiquimod is preferred. Systemic 5-FU is never used. External beam radiation is an incorrect response. Radiotherapy is generally used in older patients who are not candidates for surgery. It is also used in cases where surgical excision will be disfiguring or difficult to reconstruct (especially on the tip of the nose, and the nostril rims).

Case A 61-year-old woman presents with a 3-week history of breakouts on her face. She presented with a similar issue 6 months earlier, but she did not receive treatment. Physical examination reveals erythema and dilated vessels on the cheeks. Question Ico-delete Highlights What is an appropriate treatment?

Correct answer: Topical metronidazole cream Explanation The correct response is topical metronidazole cream. This patient has rosacea; in its mildest form, erythema and dilated vessels are seen on the cheeks. The initial management of rosacea is topical metronidazole gel or cream twice daily. Other treatment options include sodium sulfacetamide, topical erythromycin cream, and oral antibiotics (e.g., minocycline, doxycycline, or tetracycline). Oral isotretinoin is recommended for individuals with severe disease.

Question An 11-year-old boy is seen for an itching skin alteration on his fore head. He just returned from a vacation in Florida where he visited with friends who have several pets. His mother states that he has not been sick lately. The lesion appeared over night 1 week ago, and it has been growing slowly since then. The lesion is ½-dollar size, annular with sharp margins, and lightly scaly. KOH examination of scale shows hyphae. Refer to the image. What is the appropriate treatment?

Correct answer: Topical miconazole Explanation The skin alteration described above is most likely tinea faciei, a dermatophyte infection. Since there is only 1 lesion, it can be successfully treated by topical antifungal agents (topical miconazole, ketoconazole, or econazole). The pharmacologic mode of action is unknown. Oral griseofulvin is the drug of choice if systemic therapy is required. That is the case if the infection is widespread, the patient suffers from a disease with decreased cellular immunity, or if nails and hair-bearing areas are involved. Side effects of griseofulvin are gastrointestinal distress and headache. It is administered at a daily dose of 350 to 500 mg and it should be taken with a fatty meal to ensure best absorption. Dosage can be doubled if infection is unresponsive. Topical glucocorticoids would be treatment of choice in seborrheic dermatitis or pityriasis rosea. There is no indication for oral glucocorticoids in a single skin lesion. Topical podophyllin is a treatment that can be used for genital warts. It is an antimitotic drug and its use results in necrosis of visible warts. Side effects can be burning and pain.

Question A 7-month-old child presents to the emergency room exhibiting toxic epidermal necrolysis and large, flaccid bullae that are forming and rupturing under the epidermis, causing desquamation in the diaper area. This spectrum of disease describes a condition usually caused by which one of the following?

Correct answer: Toxin producing Staphylococcus aureus. Explanation Scalded skin syndrome consists of three separate entities: toxic epidermal necrolysis, scarlatiniform erythema, and bullous impetigo, which appear to be a spectrum of disease. All three conditions are attributed to the action of the exfoliating toxin of S. aureus . Interestingly, S. aureus is often not recovered from the bullae, apparently because the toxin is produced at another site of infection. In bullous impetigo, also a disease of children, the bullae are localized and are filled with staphylococci and polymorphonuclear leukocytes. A thin crust forms over the sites where the bullae have ruptured.S. pyogenes may rarely cause a necrotizing fasciitis but not the typical scalded skin syndrome. C. diphtheriae may also cause dermal lesions but they are often found in patients such as the homeless, who often live in unsanitary condition. E. coli and C. Perfringens are not usually involved in skin lesions.

Case A woman presents with her adopted 6-year-old daughter due to excessive scratching of the scalp and ears. The woman states that she washes the girl's hair frequently, but it hasn't been helpful; the woman is very frustrated for her daughter. On examination of the scalp, excessive excoriations are noted on the posterior neck and postauricular regions bilaterally. No cervical lymphadenopathy is noted. Nits are also observed on the shaft of the hair. Question What is the appropriate treatment for this patient, who has been diagnosed with lice?

Correct answer: Treat with permethrin and a fine tooth comb Explanation Permethrin is the most commonly used treatment for lice. The use of a fine tooth comb at least on a weekly basis helps with the removal of nits. Lice are human parasites and do not survive off of humans for more than 10 days to 3 weeks. Pets do not spread lice, so getting rid of pets would not treat the problem. Lice do not carry or transmit diseases; therefore, there is no testing that should be done. Oral antibiotics should be used only if a secondary infection is suspected. Patient has no cervical lymphadenopathy so no antibiotic is indicated in this case. Patient was diagnosed with head lice not ringworm or fungal infection of the scalp. No oral antifungals are needed.

Case A 32-year-old man with a past medical history of allergic rhinitis and asthma that is well-controlled complains of recurrent pruritus associated with an erythematous rash in the flexural areas of his elbows and knees. The lesions seem to become worse when he scratches them and when he is under stress. He denies any recent insect bites, travel, fever, chills, new clothing, or detergent use. Physical examination reveals rough-appearing erythematous plaques in the bilateral antecubital and popliteal fossae, with areas of excoriations within the lesions. Question What intervention is most appropriate at this time?

Correct answer: Triamcinolone 0.1% applied to the lesions once or twice daily Explanation The dermatologic lesions described in this patient, along with his coexisting past medical history, are most indicative of an eczematous eruption. Corticosteroids should be applied sparingly to the dermatitis once or twice daily and rubbed in well. In general, one should begin with triamcinolone 0.1% or a stronger corticosteroid and then taper to hydrocortisone or another slightly stronger mild corticosteroid. The presence of fissures, crusts, erosions, or pustules may indicate staphylococcal infection. Antistaphylococcal antibiotics should be given systemically if an infection is suspected. Eczema herpeticum, a generalized herpes simplex infection manifested by monomorphic vesicles, crusts, or scalloped erosions superimposed on atopic dermatitis or other extensive eczematous processes, is treated successfully with oral acyclovir. Immunosuppressant therapies, such as tacrolimus, can be effective in managing atopic dermatitis when applied twice daily. The agents should be used sparingly and only in locations where less expensive corticosteroids cannot be used. Tacrolimus and pimecrolimus both have black-box warnings for patients at high risk for lymphoma (those with HIV, iatrogenic immunosuppression, prior lymphoma). Systemic corticosteroids, such as prednisone, are indicated only for severe acute exacerbations.

Case A 28-year-old woman presents with dark spots on her nose. She states that she was diagnosed with acne vulgaris, but the treatment was not successful. On examination, small black papules less than 1 mm are found on her nose, but they are not surrounded by inflammation. On removal with forceps, the patient does not feel any discomfort. Examination of the lesion with hand lens shows vellus hairs in the keratinous plug. Question What is the most likely diagnosis?

Correct answer: Trichostasis spinulosa Explanation The correct answer is trichostasis spinulosa, based on the on the clinical examination, which shows dark black lesions with no surrounding inflammation. Lesions can be pulled out with no discomfort, and they show vellus hairs in keratinous plugs. Acne comedones, or closed acne lesions, do not contain multiple vellus hairs. Keratosis pilaris is characterized by skin-colored or slightly darker rough lesions (gooseflesh appearance). Folliculitis is an inflammatory disease. Trichostasis spinulosa is a disease characterized by aggregated vellus hairs in hair follicles. They are often found at the nose, but other body sites are possible. At young ages, the cosmetic disfigurement is the most concerning factor. It is mostly confused with open comedones in acne vulgaris. It occurs equally in men and women, especially at older ages.

Case A 38-year-old woman presents with a skin rash. She indicates that she has a history of psoriasis, but she has not had a serious outbreak for several years. You note salmon-pink papular lesions involving the flexor surfaces of the patient's wrist. The patient has no other lesions. The lesions appear to follow a linear pattern of distribution. She indicates that the lesion has been present about 3 days. You ask her if she was exposed to any type of agent, and she tells you that she scratched the area and then the lesions appeared the next days upon awakening. Question What is this lesion called?

hide Correct answer: Koebner's phenomenon Explanation The clinical picture is suggestive of the Koebner phenomenon. The Koebner phenomenon refers to skin lesions appearing on lines of trauma due to certain causative agents or by existing skin diseases, like psoriasis. The irritation of trauma itself can act as a co-factor in producing linear patterns. Auspitz sign is the appearance of minute blood droplets after the removal of psoriasis scales. Gottron's papules are purple papules on the fingers seen in dermatomyositis. Gorlin's sign refers to the ability of a patient to touch the tip of the nose with their tongue. It is seen in Ehlers-Danlos syndrome. Darier's sign appears in allergic patients; they develop urticaria when the skin is rubbed.

Question A 3-year-old girl presents with lesions on the flexural aspect of her arms. She is not taking any medications. Her mother states that symptoms appeared when they were traveling to the mountains for a winter trip less than 1 week prior to presentation. On clinical exam, well rounded, erythematous, papular lesions are present; there is weeping and scaling. What is the most likely diagnosis?

orrect answer: Atopic dermatitis Explanation Atopic dermatitis may have a genetic factor. The lesions are small papules. Scratching tends to cause erythema, weeping, scaling, and lichenification. Exacerbating factors include sudden changes in temperature/humidity, exercise, and stress. In infancy, the lesions may appear on the face; in childhood, they appear in flexural areas; in adulthood, they can appear anywhere. Contact dermatitis is a common inflammation due to external agents that can appear anywhere on the body. The lesions can be papules, blisters, and pustules and appear red, scaly, and dry. Treatment includes a detailed history to remove the causative agent, antihistamines, cold compresses, and or topical/systemic steroids. Nummular eczema has round, coin-like (nummular) lesions with a distribution on the extensor surface of the extremities as well as the posterior portion of the trunk, buttock, and legs. Purulent drainage is not uncommon. Treatment includes topical steroids, systemic antibiotics, antihistamines; decreased exposure to water and drying agents are used when oozing is present. Lichen simplex is rare in children; it presents with lesions that are circumscribed, scaling patches resulting from chronic scratching. The margins appear well-defined. The most common occurrences are on the ankles, anterior tibia, and nuchal areas. Erythema multiforme is characterized by macules, papules, vesicles, and bullae. It occurs secondary to a toxic influence. There may or may not be a prodromal period with a sore throat, diarrhea, and fever. The lesions are red macules or papules and are seen on the sides of the neck, face, legs, genitalia, and mucosa membranes. A typical lesion is a "target" or "iris" lesion. The course is generally 3 - 4 weeks.

Case A 23-year-old woman presents with itchy, scaly patches on both her knees and lower legs. The patches were initially penny-sized, but have recently increased in size and are mildly itchy now. There is no history of infection, drug intake, or insect bites. However, she says that she has been under extreme stress over the last month due to family issues. On examination, you note whitish, irregular, slightly raised plaques on the knees and legs. Scraping the scales leads to multiple bleeding points. Question What clinical finding have you elicited?

orrect answer: Auspitz sign Explanation The correct answer is Auspitz sign. The diagnosis in this woman is chronic plaque psoriasis. When the scales over a plaque are scraped, it leads to appearance of multiple bleeding points. It appears as a moist red surface through which dilated capillaries are torn at the tip of the elongated dermal papillae. This sign is secondary to parakeratosis, elongation of the dermal papillae, thinning of stratum malphighii, and dilatation and tortuosity of papillary capillaries. Auspitz sign is neither sensitive nor specific for psoriasis. This sign may not be elicited in inverse, pustular, erythrodermic, or guttate psoriasis. It may additionally be seen in nonpsoriatic conditions such as Darier's disease and actinic keratosis. Carpet tack sign refers to the presence of horny plugs on the undersurface of follicles, which are revealed when scales are removed from erythematous plaques in discoid lupus erythematosus, though it is not diagnostic for it. It may be seen in certain other conditions, such as seborrheic dermatitis and pemphigus foliaceous, but these conditions are associated with loose scales. Scratch sign refers to the loosening and appearance of a barely perceptible scale in pityriasis versicolor when scratched with a fingernail. This sign may be negative if the person has recently washed the area, or if the lesion has been recently treated. The Koebner phenomenon refers to the appearance of new lesions on lines of trauma, which are often linear. It is seen in psoriasis, lichen planus, and vitiligo, apart from other conditions. Rovsing sign is not a dermatological sign. It refers to increased pain experienced in the right lower quadrant when the left lower quadrant of the abdomen is palpated. It may be seen in appendicitis.

Case A 59-year-old Caucasian woman presents with a 1-month history of a lesion on her face. She reports a tendency to sunburn ever since her youth. She is not taking any medications and she exercises regularly. On examination, there is a brown irregularly-shaped macule 3 cm in diameter on her right cheek. It has darker brown spots scattered irregularly within it. Question What is the most likely diagnosis?

orrect answer: Malignant melanoma Explanation The most probable diagnosis in this patient is malignant melanoma. Malignant melanomas arise from melanocytes. The main types are: Superficial spreading melanomas—irregular tan plaques with blue-black spots or nodules on any site Nodular melanomas—develop anywhere on the body as dark papules or plaques that grow rapidly Lentigo maligna melanomas—develop on sun-exposed areas (e.g., the face) as brown macules with darker spots scattered on its surface Acral lentiginous melanomas—most common form in African American patients; they develop on the palms, soles, subungual skin, and mucous membranes Surgical excision is the definitive treatment for early-stage melanoma. Chemotherapy, radiotherapy, and immunotherapy are used for advanced melanoma. Patients with basal cell carcinomas (BCC) can present with a shiny pearly nodule (noduloulcerative BCC), or with erythematous scaling plaques (superficial BCC), or with a solitary flat yellowish plaque (morpheaform BCC). Patients with squamous cell carcinomas (SCC) may present with a red papule or crusted plaque on the lips, ears, neck or hands that may ulcerate, invade the underlying tissue, and metastasize. There are several forms of Kaposi's sarcoma (KS), including: Classic KS—purple plaques or nodules on the lower limbs of elderly men of Mediterranean origin without AIDS AIDS-related form—widely disseminated purple papules or plaques on the skin, mucous membranes, and viscera Lymphadenopathic form—involves lymph nodes and skin In Paget's disease of the breast, patients present with a unilateral sharply marginated, red, scaly rash or erythematous crusting affecting the nipple and areola. There may also be an underlying mammary duct carcinoma.

Question A 59-year-old woman presents for second- and third-degree burns on her head and neck, the anterior surfaces of her upper extremities, anterior right leg, and anterior trunk, including her genital area. What is the estimate of the extent of the patient's burns, according to the Rule of Nines?

Correct answer: 46% Explanation The correct answer is 46%. This estimation is based on the Rule of Nines. Body surface area is estimated at 9% for the following areas: head and neck anterior surface of upper torso anterior surface of lower torso posterior surface of upper torso posterior surface of lower torso anterior surfaces of each leg posterior surfaces of each leg and an additional 1% for the groin area for a total of 100%. In this case, 9% for her head and neck, 9% for the anterior surface of both arms, 9% for the anterior surface of her right leg, 9% for her anterior upper torso, 9% for her anterior lower torso, and 1% for the genital area for a total of 46%.

Case A 16-year-old boy presents to his primary care physician with skin lesions. The lesions are scattered over his forehead, nose, and chin. He denies facial flushing and pruritus. The following presentation is seen on examination (see image). Question What is the likely diagnosis?

Correct answer: Acne vulgaris Explanation Acne vulgaris is correct. The patient has the presence of comedones, which are characteristic of acne vulgaris. Acne vulgaris is commonly seen in adolescent males. Rosacea is incorrect. While the lesions associated with rosacea resemble acne, it also presents with facial flushing and erythema, eye irritation, and occasionally an enlarged nose. It more commonly presents in middle-aged women. Contact dermatitis is incorrect. Contact dermatitis usually presents with erythematous pruritic lesions. Erythema multiforme is incorrect. Erythema multiforme presents with target lesions, usually on the distal extremities. Impetigo is incorrect. Impetigo presents with honey colored crusted lesions. It is more common in children.

Case A 15-year-old boy is being treated at the dermatologist for lesions on his cheeks, forehead, chin, upper chest, and upper back with a history that has been progressively getting worse. Upon physical exam, open and closed comedones are present on the patient. Additionally, inflamed papules and some scarring are present. All labs are within normal limits. Question What is the most likely diagnosis?

Correct answer: Acne vulgaris Explanation The clinical picture is suggestive of acne vulgaris, due to the physical presentation of open and closed comedones. Additionally, the puberty age, location of comedones, and appearance of inflamed papules with some scarring are typical features of acne vulgaris. Contact dermatitis is not associated with comedones; it is associated with a rash. Folliculitis is an infection of a hair follicle and is not associated with comedones; it is therefore an incorrect answer. Furthermore, it appears as blistering and rash, with pus, skin burning sensation, and itching, all of which is not described in the stem of the problem. Miliaria is an obstruction of sweat glands, or also described as a heat rash. It can appear as blisters and red lumps. Itching and flushing can also occur. Furthermore, it is not associated with comedones, and therefore is an incorrect answer. Rosacea is not the correct answer, because it is not associated with comedones and is associated with swelling and redness.

Case A 45-year-old woman has a history of laying out in the sun at her home in Florida since she was a teenager. She states she noticed a lesion on her cheek for the past few weeks and it will not go away. Upon physical exam, the patient has a wart that appears to be elevated and pink with a sandpaper texture. Question What is the most likely explanation of these findings?

Correct answer: Actinic keratosis Explanation The clinical picture is suggestive of actinic keratosis,as the patient presents clinically in a sun exposed as sandpaper textured, elevated, pink, wart-appearing lesions. Melanoma is not the correct answer. It presents instead with the following ABCD's: asymmetry, border irregularity, color that is not uniform throughout, and a diameter greater than 6 mm. Basal cell cancer is not the correct answer because this patient does not present with a pink or flesh colored papule, therefore this is an incorrect answer to this problem. Basal cell carcinoma appears as a pink or flesh colored papule. It can also have an ulceration. Squamous cell skin carcinoma is not the correct answer because the patient would have to present with a white waxy lump or a brown scaly patch, which is not present in this stem of the problem and therefore the incorrect answer. Melasma is not the correct answer because the patient would appear with brown patches on the face instead of a sandpaper-appearing lesion, and therefore is the incorrect answer to this problem. Furthermore, melasma is more commonly found in pregnant women due to hormonal changes.

Case A 28-year-old man presents with a rash. The mildly pruritic lesions are located on his arms and legs. They have been present for about 3 days, without change or resolution. He has tried over-the-counter anti-itch creams, but they have been ineffective. The patient reports that he was seen approximately 1 week ago for some blister-like lesions on his penis. He was given an antiviral medicine and those lesions resolved. He is wondering if he was misdiagnosed and if the 2 "rashes" are related. He admits to feeling some malaise over the last 2 weeks, but he is otherwise healthy. He denies fevers, unusual travel, medication use (except for as listed above), and known allergies. He has no known chronic conditions. On physical exam, vitals are normal; the patient is in no apparent distress. A pink-to-red papular rash is observed on the backs of the hands and feet and extensor surfaces of the arms and legs. The individual lesions are quite distinct; they have a red center, and they are surrounded by a pale ring and then another outer ring of red, inflamed tissue. The remainder of his physical exam is normal. Question The patient is at risk of recurrence for both "rashes." What medication would best prevent recurrence of this patient's second rash (the one presented today)?

Correct answer: Acyclovir Explanation The correct response is acyclovir. This patient is presenting with a target-lesion rash; it is typical of erythema multiforme, which is a relatively common type IV hypersensitivity reaction. In about 50% of the cases of erythema multiforme, no cause is identified. Other causes are infection and medications. When a cause is identified, recent herpes simplex virus (HSV) infection is the most common. In this patient's case, he likely was appropriately diagnosed and treated for his HSV infection, then developed the erythema multiforme rash. Erythema multiforme is an acute, self-limited condition; it can range in severity from a rash, mild malaise, and pruritus (erythema multiforme minor) to a much more severe condition, involving mucosal surfaces and possible desquamation with erythema multiforme major. For individuals who have experienced erythema multiforme in response to HSV, acyclovir is recommended to prevent recurrence. Diphenhydramine is an antihistamine and is often used in the acute treatment of the erythema multiforme, but it is not recommended for health maintenance as a way of preventing recurrence. Doxycycline is a tetracycline antibiotic. It is useful for several bacterial infections and several dermatologic conditions, ranging from acne to anthrax. It does not have a role in the treatment or prevention of erythema multiforme. Methotrexate is an immunosuppressant medication used for a variety of conditions, such as rheumatoid arthritis and severe psoriasis. This patient's rash is not suggestive of psoriasis, so methotrexate would not be appropriate. Prednisone is a steroid medication, often used during the acute stage of more severe erythema multiforme. Due to significant side effects associated with long-term use of systemic steroids, prednisone would not be recommended as a prophylactic treatment for this patient.

Question A 27-year-old woman presents with painful, itchy blisters on her genitals. She is G0/Ab0/P0, has a regular monthly cycle (29/5); menarche was at 13, and she has had a steady partner for 2 years. She does not use any method of birth control, including condoms. She states that both her partner and she are monogamous, and he does not have any symptoms. 2 days earlier, she had been feeling sick. She was taking OTC aspirin, but still has an elevated temperature of 37.5°C. On exam, you palpate enlarged lymph nodes in the inguinal area and see multiple vesicles on her labia and perineum; some are ruptured and some are crusted over. There is no vaginal discharge, and the rest of the pelvic exam is inconspicuous. What is the proper treatment for this condition?

Correct answer: Acyclovir Explanation The symptoms described above are typical for genital herpes, which is caused by the herpes simplex virus. There is no cure for herpes infections; however, oral acyclovir, an antiviral drug, can reduce the number of recurrences. Penicillin would be treatment of choice for syphilis, caused by treponema pallidum. Fluconazole, an antifungal drug, would be treatment of choice in candida vulva-vaginitis, caused by candida species, mostly candida albicans. Metronidazole, an antiprotozoal and antibiotic drug, is first-line therapy for bacterial vaginosis, which is caused by Gardnerella vaginalis; it is also the first-line therapy for trichomoniasis, which is caused by trichomonas vaginalis. Doxycycline, a tetracycline, is often used in combination with other antibiotic drugs to treat pelvic inflammatory disease, which is usually caused by chlamydia trachomatis and neisseria gonorrhoeae.

Case A 20-year-old man has had a 1-month duration second-line acne therapy; he now presents with skin that looks worse and has more inflammation and cystic development. This patient has now progressed to severe acne. Question What is the most appropriate next step?

Correct answer: Add oral isotretinoin after stopping medication for 2 weeks. Explanation Systemic treatment with isotretinoin is indicated for patients with cystic acne or with acne that has not responded to conventional therapy. Systemic retinoids are teratogenic and should be avoided in pregnant women or women planning to conceive. Informed consent must be obtained before its use, and patients must be enrolled in a monitoring program (iPledge). For severe acne, the medications are continued, not discontinued. Skin biopsies and blood cultures are not indicated for acne treatment. Emollient creams are used to soften skin. Sulfur-based cleansers are indicated for treating acne but are not indicated for inflammatory or cystic-type acne.

Question A surgeon performs an exploratory laparotomy, producing a large incision in the patient's abdomen. Poor blood supply to what area is most likely to cause problems during the healing process?

Correct answer: Adipose tissue Explanation The correct response is adipose tissue. Surgeons worry about their obese patients more than their underweight patients because a thick layer of relatively poorly vascularized subcutaneous fatty tissue is both mechanically unstable (it holds stitches poorly) and heals very slowly. These patients have a frequent rate of dehiscence (tearing open of the incisional site) with subsequent difficult-to-control infection (access by antibiotics, leukocytes, and serum antibodies are all hampered by the poor blood supply). Aponeuroses are strong thickenings of muscle sheath that usually suture and heal well after surgery. Loose connective tissue is well vascularized and surgeons do not usually worry much about it during the healing process. Muscle usually heals well after surgery. Skin usually heals well, unless it becomes infected.

Case An 8-year-old girl presents with a 3-month history of hair loss. Except for usual minor childhood illnesses, such as colds and ear infections, the girl has been in good health since birth. 3 months ago, the mother noted small bald areas developing on the child's scalp when she brushed the child's hair. The child denies any pain or itching of the scalp, and she also denies pulling at her hair. There have been no other symptoms. She has not taken any medications, and she has no known allergies. On exam, you find 3 round silver dollar sized areas of complete alopecia. The scalp is normal. Question What is the most likely diagnosis?

Correct answer: Alopecia areata Explanation The correct answer is alopecia areata. Alopecia areata is characterized by hair loss in round or oval patches on the scalp. The skin within the plaques of hair loss is normal. Alopecia areata occurs in 1% of the population; 60% of patients are less than 20 years old. It is associated with atopy and autoimmune diseases. The course of alopecia areata is unpredictable, but may resolve spontaneously in 6 to 12 months. Occasionally, high potency topical steroid preparations are prescribed. Toxic alopecia, also known as anagen effluvium, is inhibition of the anagen follicle that results in loss of 80-90% of scalp hair. It occurs acutely and diffusely, as a result of chemotherapy, radiation, medications (thallium, thiouracil, heparin) and hypervitaminosis A. Traction alopecia is due to trauma to the hair follicles that occurs as a result of tight braids, ponytails, curlers, rollers, or headbands. Broken hairs and inflammatory follicular papules are seen at the scalp margin, and lymphadenopathy may be present. Trichotillomania is compulsive pulling, twisting, and breaking of hair. The result is irregular areas of incomplete hair loss, usually on the crown, and in the parietal or occipital areas of the scalp. The scalp is normal as in alopecia areata. Trichotillomania is associated with obsessive-compulsive disorder in some children; in others it is a benign habit. Tinea capitis is a dermatophyte infection of the scalp caused by Trichophyton tonsurans or Microsporum canis. The lesion starts as a small papule at the base of the hair follicle, spreading into a circular plaque (often known as ringworm). The infected hairs break off and create a pattern known as "black-dot ringworm" where the patches of alopecia have small hairs broken off at the hair follicle. Occasionally, patients have a severe inflammatory response, causing development of a large, boggy mass known as a kerion. Treatment is with oral antifungal agents, such as griseofulvin.

Case A 19-year-old man presents with hair loss described as localized oval patches for the past month. Upon examination, the patches are sharply demarcated without tenderness, erythema, or scaling noted. Question What is the most likely diagnosis?

Correct answer: Alopecia areata Explanation The most likely diagnosis is alopecia areata, as it typically occurs in children and young adults and presents as oval or round sharply demarcated patches of hair loss. Secondary syphilis typically presents with a moth-eaten appearance of hair loss. Trichotillomania (hair pulling) occurs most often in children and the hair loss is irregular and sporadic. Tinea capitis may present with hair that is broken off and often with erythematous scaly patches of the scalp that are sometimes tender. Male-pattern baldness does not present with sharply demarcated patches of hair loss.

Case A 3-year old boy is brought to the pediatrician with a 2-week history of symptoms of an itchy, red "wound" on the right knee. The mother ignored the wound initially, assuming that the boy got injured while playing on the street, but it did not heal and seems to have worsened. On examination, there are a few intact vesicles and a few ruptured vesicles covered with honey-colored crusts. You suspect non-bullous impetigo. Question What treatment will you prescribe?

Correct answer: Antibiotic ointment Explanation Impetigo is a contagious pyogenic infection classified into bullous and non-bullous types. The non-bullous type is caused by Staphylococcus aureus or Streptococcus pyogenes and is more common on the exposed parts of the body. An initial breach in skin barrier by trauma, insect bite, etc., occurs, followed by infection. It is most common in the age group 2-5 years and in those living in poor hygiene or overcrowded settings. It is a self-limiting condition and treatment may aid in reducing transmission and relieving discomfort. Treatment is with proper wound care and topical antibiotics like mupirocin or retapamulin. The bullous type is characterized by large bullae, along with fever, diarrhea, weakness, etc. Topical or oral steroids are not indicated in skin infections. Oral antibiotics are usually not necessary in non-bullous impetigo, although they may be indicated in extensive bullous lesions. Emollients are not necessary in impetigo.

Case A 75-year-old woman presents with a 2-week history of dryness and itchiness of her legs. She reports that it has been recurrent for the past 2 winters. On examination, you note pretibial erythematous fine cracks. Question What is the most likely diagnosis?

Correct answer: Asteatotic eczema Explanation Asteatotic eczema is characterized by fine cracks, which may be erythematous, usually on the pretibial surfaces of elderly people during winter. They may also be pruritic. Management includes application of topical emollients and avoidance of irritants. Lichen planus is characterized by pruritic, polygonal papules which are flat-topped and violaceous in color and have a network of gray lines (Wickham's striae) on the surface. They commonly develop on the wrists, legs, and genitalia. Management includes using topical glucocorticoids though many patients have spontaneous remissions. Stasis dermatitis is characterized by erythematous, pruritic scales with oozing patches usually over the medial aspect of the ankles. Hyperpigmentation and stasis ulcers may also be present. It develops on the lower limbs due to chronic venous incompetence. Management includes using topical glucocorticoids and emollients as well as avoiding irritants. Contact dermatitis in the acute phase is characterized by sharply demarcated, weeping, edematous, eczematous plaques. There is usually history of contact with an allergen. Management includes avoidance of irritants and application of topical glucocorticoids. Lichen simplex chronicus consists of a well-circumscribed scaly plaque with lichenification and hyperpigmentation due to chronic scratching or rubbing (which can also occur during sleep). It commonly involves the occiput, back of the neck, arms, dorsum of the feet, and ankles.

Case A patient comes to see you for a growth on his forearm. He has had it for 7 years and is concerned that it may be enlarging. You observe a well-circumscribed, pink, slightly pigmented, raised nodule about 1cm diameter. The center appears slightly ulcerated. Question What is the most likely diagnosis?

Correct answer: Basal cell carcinoma Explanation Basal cell carcinoma usually begins as a small indented nodule localized around hair follicles, typically in sun-exposed regions of the skin. It typically begins to form an ulcerated center as it enlarges. This type of tumor rarely metastasizes but is locally invasive. It can be quite disfiguring. Excision is necessary to prevent its spread. Melanoma is typically a highly pigmented tumor of the skin, often seen in sun-exposed regions. This tumor metastasizes widely and early on. The lesion may be a constellation of pigment colors, ranging from black to blue to red. The borders are frequently uneven. Tumors may arise from other organs including the meninges, throat (pharynx and larynx), and eye. Satellite nodules may also be seen distant from the main tumor site. Due to the aggressive character of this tumor, wide and deep incisions are necessary, along with tumor staging if there is the possibility of metastasis. Molluscum contagiosum is caused by a large DNA pox virus. This disease is typically seen in children and sexually active young adults. The lesion presents as a pearly, flesh-colored papule approximately 2-10mm in diameter, often with a central umbilication. These lesions are often seen on the skin or mucous membranes. Treatment includes curettage and desiccation or liquid nitrogen. Diagnosis is supported by the presence of intracytoplasmic inclusions in a KOH preparation of crushed tissue. Squamous cell carcinoma is also a slow growing tumor often found on sun-exposed surfaces of the skin. This cancer does eventually metastasize. The lesion appears as a raised plaque that tends to ulcerate. Unlike basal cell carcinomas, this tumor originates in the skin and squamous mucosa, which can result in lesions involving the skin, cervix, tongue, esophagus, and lips. Histologically, this tumor may range from a more indolent type of tumor containing keratin pearls of concentric lamellated keratin to a very anaplastic nature where no pearls are evident. Squamous cell carcinoma metastasizes via lymphatics and then to the rest of the body. Excisional biopsy is required to determine the type of skin cancer and treatment necessary. Lipoma is a subcutaneous, soft, round to oval mass of fatty connective tissue. The lesion is approximately 3-5mm in diameter and surrounded by a delicate capsule. The lipoma is normally a benign growth, but constant enlargement may cause compressive effects on surrounding tissues. Local excision is the usual mode of treatment.

Case A 55-year-old man presents with a non-healing, inflamed 'pimple' on his cheek. It has been present for 5 months, and it has recently started to bleed when he shaves. On physical examination, you note that he has fair skin, blue eyes, and a ruddy complexion. There is a raised lesion on the left cheek measuring 2.5 cm in diameter that appears rolled, with pearly pink borders and telangiectasias. Image 6-1 (courtesy John Hendrix, MD) depicts the lesion. Question What is your preliminary diagnosis?

Correct answer: Basal cell carcinoma Explanation This case is a classic presentation of a basal cell carcinoma (BCC). It is found most commonly on the head and neck. People with fair skin and light hair and eyes are at particular risk. BCC is the most common of all the skin cancers. It has been estimated that some 800,000 Americans are affected at any given time. It appears to be sun induced, and it is most commonly found on the exposed areas of skin in lightly pigmented persons. The lesions have several modes of presentation, the most common being a shiny, pearly, translucent nodule with telangiectasias. Basal cells carcinomas arise from the basal cells of the epidermis and almost never metastasize. Metastases from solid organ malignancies are subcutaneous, firm nodules, although they may ulcerate. Merkel cell carcinoma is a rare tumor on the head and/or neck that usually appears as an ill-defined, violaceous (violet color) nodule or plaque. Sebaceous hyperplasia is usually ivory or yellow in appearance and has a central pore. It is typically well-demarcated; it is often pigmented and rough on the surface, without inflammatory signs. Squamous cell carcinoma is usually a hyperkeratotic or ulcerated papule or plaque.

Case A 67-year-old man was cleaning out his garage and noticed a "bug" crawling on his leg. The "bug" bit him and was then killed by the man. He discarded it and went about his business. Two days later, he presents with pain, itching, and swelling of the affected leg. The "bug" had a violin-shaped pattern on its back. Question Based on the patient's symptoms and description, was this the result of an insect or arachnid bite?

Correct answer: Brown recluse spider Explanation Based on the description and the symptoms, the referred to "bug" is in actuality, a brown recluse spider. The brown recluse is an arachnid and is one of only about 4 spiders capable of being dangerous to humans. They are native to the Midwestern and Southeastern sections of the United States. The bite from one can result in death if treatment is not given. The brown recluse spider can be recognized by the "violin" pattern found on its back. They are usually dark brown in color and have dark brown to black colored legs. They usually do not attack unless frightened or when they touch the skin of an animal or human. They can be found in closets, basements, sleeping bags, woodpiles, or other warm, dry places. The venom of a brown recluse spider is extremely poisonous, and its toxin causes damage to cells and tissues. This is done through the use of enzymes that can destroy cell membranes. This can lead to the local destruction of the skin. It can also spread to the blood vessels and fat. The skin becomes necrotic in the area of the bite, which is characteristic for this spider. The immune system becomes activated as with other insect bites. Brown recluse spider bites can lead to necrosis of the skin, destruction of red blood cells, blood clot formation, acute renal failure, coma, and death. Symptoms of a brown recluse spider bite can be similar to a bee sting. There can be mild erythema, pain, itching, and myalgias. These symptoms usually develop within a few hours. Within a few days, the area will heal. In severe cases, the patient may develop fever, nausea, vomiting, blistering, and eventually necrosis of the affected skin. The diagnosis can be made through the patient's history and by examining the affected area. Laboratory analysis should include a complete blood count, urinalysis, electrolyte, and renal function tests. Usually, no specific tests are done. Treatment usually consists of tetanus immunization, antihistamines, and antibiotics. In some cases, steroid treatments are used. This is usually reserved for patients who have an underlying illness such as anemia or renal failure.

Case A 16-year-old boy presents to a local urgent care center for treatment of a painful right arm. The patient states that he was camping with his family in Alabama and was bitten by an insect that he did not see. The wound was cleaned and the boy's camping trip continued without any further interruption. Approximately 5 hours after being bitten, he began feeling intense pain, itching, fever, and nausea. The affected area was red initially, but then became blistered and bluish in color. At the time of presentation, the area showed signs of necrosis. Question What is the insect that most likely caused this patient's symptoms?

Correct answer: Brown recluse spider Explanation The brown recluse spider is one of only about 4 spiders capable of being dangerous to humans. They are native to the Midwestern and Southeastern sections of the United States. The bite from one can cause death if treatment is not given. The brown recluse spider can be recognized by the "violin" pattern found on its back. They are usually dark brown in color and have dark brown to black-colored legs. They usually do not attack unless frightened or when they touch the skin of an animal or human. They can be found in closets, basements, sleeping bags, woodpiles, or other warm, dry places. The venom of a brown recluse spider is extremely poisonous, and its toxin causes damage to cells and tissues. This is done through the use of enzymes that can destroy cell membranes. This can lead to the local destruction of the skin. It can also spread to the blood vessels and fat. The skin becomes necrotic in the area of the bite, which is characteristic for this spider. The immune system also becomes activated, as with other insect bites. Brown recluse spider bites can lead to necrosis of the skin, destruction of red blood cells, blood clot formation, acute renal failure, coma, and death. Symptoms of a brown recluse spider bite can be similar to a bee sting. There can be mild erythema, pain, itching, and myalgias. These symptoms usually develop within a few hours. Within a few days, the area will heal. In severe cases the patient may develop fever, nausea, vomiting, blistering, and eventually necrosis of the affected skin. The diagnosis can be made through the patient's history and by examining the affected area. Laboratory analysis should include a complete blood count, urinalysis, electrolyte, and renal function tests. Usually no specific tests are done. Treatment usually consists of tetanus immunization, antihistamines, and antibiotics. In some cases, steroid treatments are used. This is usually reserved for patients who have an underlying illness such as anemia or renal failure.

Case A 4-year-old boy presents with a 1-month history of a red area on his cheek that has gradually enlarged. The rash is slightly pruritic but is nontender. The patient had an upper respiratory infection 3 weeks ago that lasted 5 days. He had a low-grade subjective fever for the first 2 days of that illness. There is no recent travel history. The patient was given a new kitten for his birthday 2 months ago. Past medical history and family history are unremarkable. On examination, the patient is well-appearing and afebrile. He has a 3-cm annular erythematous lesion on his right cheek. It is slightly scaly with central clearing and a papulovesicular border. The rash is nontender to touch. There is no cervical adenopathy. Eyes, ears, and pharynx are normal to examination. Wood lamp examination reveals no fluorescence. Question What is the most appropriate treatment for this patient?

Correct answer: Clotrimazole 1% cream topically tid for 3 weeks Explanation The correct response is clotrimazole 1% cream topically tid for 3 weeks. Tinea corporis (ringworm) is a common skin lesion found in children. It is caused by superficial infection of glabrous (non-hairy) skin with a variety of fungi. In children, the causative agent is usually Microsporum canis, Microsporum audouini, or Trichophyton mentagrophytes. Adults are more likely to be infected with Trichophyton tonsurans, Trichophyton mentagrophytes, Trichophyton rubra, or Trichophyton verrucosum. The lesion has a characteristic appearance that is annular and scaly with a well-demarcated vesicular, pustular, or papular border. If the lesion has been present for any significant length of time, there will be central clearing of the lesion. The infection is often contracted from young kittens or puppies. Dermatophyte infections of the hair with Microsporum audouinii and canis produce a brilliant green fluorescence under Wood lamp, while Trichophyton schoenleinii produces a pale green fluorescence. Trichophyton tonsurans and violaceum do not fluoresce. It should be noted that only the hair fluoresces in the case of dermatophyte infection, not the skin. Cellulitis due to Streptococcus pyogenes or Staphylococcus aureus is characterized by erythema, induration, and tenderness of the skin. It often follows some trauma to the skin. The infection may be superficial in the case of erysipelas, caused by group A streptococcus; it also may be deeper, involving subcutaneous skin. The skin surface is usually smooth, although Staphylococcus aureus may cause exfoliation, which is seen in toxic epidermal necrolysis due to an exfoliative toxin. Candidal skin infection (moniliasis) has a predilection for intertriginous areas, and it is uncommon on the face. It has a macerated, shiny red appearance with small "satellite" pustules at the border. Nummular eczema is a discoid, or coin-shaped, plaque caused by dry skin and possible atopy. There is no central clearing or "active" border to the lesion. Localized dermatophyte infections may be treated topically with clotrimazole. Griseofulvin is associated with liver toxicity and aplastic anemia, and it should be reserved for dermatophyte infections of the hair (tinea capitis and tinea barbae) and nails (onychomycosis) or chronic, persistent infection of the feet (tinea pedis). Nystatin is an effective topical treatment for candidal infections, but it is ineffective in treating dermatophytosis. Topical steroids are an appropriate treatment for nummular eczema, but they only mask the clinical features of tinea corporis (tinea incognito) without treating the infection. Amoxicillin and cephalexin are appropriate therapy for uncomplicated cellulitis due to Streptococcus pyogenes and Staphylococcus aureus, respectively.

Case A 16-year-old female presents with a red, itchy rash on her feet. The patient reports the rash started when she wore her new cheerleading shoes. PMH negative. The patient takes no medication. There is a stocking pattern of vesicles and some crusting. The surrounding area is erythematous. Question What is the most likely diagnosis?

Correct answer: Contact Dermatitis Explanation Pompholyx or dyshidrotic eczema is a vesicular eruption with intense itching involving the hands and soles. This is generally secondary to stress. The vesicles are prone to involute and desquamate. Scales are a late finding. Treatment involves a high potency steroid. Tinea pedis occur in two forms: chronic, scaly, ruborous form and an acute bullous type. The dry scaly tinea pedis is often caused by T. rubrum whereas the bullous type is caused by T. mentagrophytes (isolated from the roof of the bullous lesion). A dry tinea contains only dermatophytes and normal flora. Once macerated, gram-negative rods (ie. Pseudomonas) plus a lower amount of dermatophytes are involved. Treatment consists of antifungals (ie. imidazoles). However, the gram-negative rods do not respond to the imidazoles and must be treated with a non-imidazole (ie. ciclopirox olamine). Contact dermatitis may present with minor redness to itchy patches, vesicles, and ulcers.It is usually well demarcated, localized, and found in the area of contact with the offending substance. The patches may blister, open and ooze, scale, or crust. The lesions may be self-limiting if the offending substance is avoided. Treatment includes avoidance of the substance, cool compresses, oatmeal baths. The bullous pemphigoid eruptions are self-limiting and subepidermal. They are dense, thick bullae seen frequently in the elderly. IgG's are seen along the basement membrane as this is associated with the immune system. Treatment consists of astringent soaks, systemic steroids, and immunosuppressants. Bullous diabeticorum involve spontaneous eruptions in diabetic patients. These may be epidermal or subepidermal and are self-limiting. The etiology is unknown. Treatment consists of draining and local wound care

Case A 15-year-old boy presents with a complaint of small red itchy blisters on his feet and ankles. The blisters began early in the summer when he was attending band camp. He denies taking medication. On clinical exam, there are red papules with some dry scales. The area affected is limited to the foot and ankle area. Question What is the most likely diagnosis?

Correct answer: Contact dermatitis Explanation Contact dermatitis is a common inflammation due to external agents that can appear anywhere on the body. The lesions can be papules, blisters, and pustules and appear red, scaly, and dry. Treatment includes a detailed history to remove the causative agent, antihistamines, cold compresses, and or topical/systemic steroids. Atopic dermatitis may have a genetic factor. The lesions are small papules. Scratching tends to cause erythema, weeping, scaling, and lichenification. Exacerbating factors include sudden changes in temperature/humidity, exercise, and stress. In infancy, the lesions may appear on the face; in childhood, they appear in flexural areas; in adulthood, they can appear anywhere. Nummular eczema has round, coin-like (nummular) lesions with a distribution on the extensor surface of the extremities as well as the posterior portion of the trunk, buttock, and legs. Purulent drainage is not uncommon. Treatment includes topical steroids, systemic antibiotics, antihistamines; decreased exposure to water and drying agents are used when oozing is present. Lichen simplex is rare in children; it presents with lesions that are circumscribed, scaling patches resulting from chronic scratching. The margins appear well-defined. The most common occurrences are on the ankles, anterior tibia, and nuchal areas. Erythema multiforme is characterized by macules, papules, vesicles, and bullae. It occurs secondary to a toxic influence. There may or may not be a prodromal period with a sore throat, diarrhea, and fever. The lesions are red macules or papules and are seen on the sides of the neck, face, legs, genitalia, and mucosal membranes. A typical lesion is a "target" or "iris" lesion. The course is generally 3-4 weeks.

Case A 12-year-old boy presents with itching and redness between his toes. The mother notes that this is his second visit in 2 weeks, and she adds that her son had the same symptoms previously. She states that the child was prescribed an antifungal cream on the previous visit. They still have cream left. On examination, the interdigital spaces are macerated and erythematous. Question What is the most appropriate management for this case?

Correct answer: Continue the antifungal cream and give health education advice. Explanation A tinea pedis fungal infection of the keratin of soles and interdigital spaces is due to a dermatophyte fungus (most commonly Trichophyton rubrum). Dermatophytes use enzymes called keratinases that help them invade the superficial keratin of skin, so the infection remains limited to this layer. The dermatophyte cell wall also contains mannans (especially T. rubrum) that reduce keratinocyte proliferation; this results in a decreased rate of keratin sloughing and a chronic infection state. Continuing the antifungal cream and giving health education advice about the nature of the disease and how laundering the occlusive footwear will prevent relapse. Also, tinea pedis often recurs because the patient stops the medication as soon as the symptoms stop. Educate the patient to keep using the medication until the entire tube is empty. Factors that lead to moist skin (e.g., prolonged sweating in hot, humid, tropical environments; prolonged use of closed shoes) result in complications (e.g., hyperhidrosis, maceration) that make the skin subject to fungal infection. The mode of transmission is person-to-person, and activities such as swimming and communal bathing (keeping feet wet for a long time) increase the risk of infection. An antibiotic cream will not affect the dermatophytes, and a systemic antibiotic will not add any advantage to the treatment. Continuing the antifungal cream with the addition of a systemic antifungal will not prevent a relapse; it is used in patients with a systemic disease (e.g., diabetes mellitus). Prescribing a steroid cream for itching will certainly alleviate the itching symptoms, but it will worsen the condition due to the continued progression of the disease.

Case A 67-year-old man presents concerned with 2 asymptomatic skin lesions on the underside of his penis; the lesions are shown in the image. He has a 20-year history of hypertension; it is well-controlled with diet and hydrochlorothiazide (HCTZ). He is a retired automobile sales manager, and he has been faithfully married for 40 years. There is no history of sexually transmitted infections, and he has not been sexually active for the past 2 years. He has no history of skin cancer; he does not smoke tobacco, drink alcohol, or have a history of illicit drug use. Question What does appropriate management include?

Correct answer: Cryospray and curettage Explanation The correct response is Cryospray and curettage. The most likely diagnosis is Seborrheic keratosis (SK). Seborrheic keratoses are common skin lesions that affect the elderly. They tend to run in families. SK is typically well-demarcated; it is most often pigmented and rough on the surface, without inflammatory signs. The lesions have been described as waxy and "stuck on" in appearance, as if they can be peeled off. Darkly pigmented lesions can be hard to distinguish from nodular melanomas; therefore, if in doubt, a skin biopsy should be performed straightaway. Seborrheic keratoses are common on sun-exposed areas, such as the back, arms, face, scalp and neck; thus, in addition to genetics, ultraviolet light has been implicated in their etiology. However, unlike SCC, BCC, and MM, they are also found on skin that has not been exposed to the sun such as the genital area, in this case, on the shaft of penis. SK lesions are benign and do not require treatment except for cosmetic reasons. Lesions in the genital area might negatively influence the sexual life of the patient and thus removal may be preferred. Electrocautery (electrosurgery) permits the whole lesion to be easily rubbed off. This, however, precludes histopathologic verification of diagnosis. Cryosurgery with liquid nitrogen spray works only in flat lesions, and recurrences are possibly more frequent. The best approach is curettage after slight freezing with cryospray, which also permits histopathologic examination.

Case A 57-year-old man presents for a routine checkup; he is concerned about a lesion on his head. He appears healthy and has a rustic, tanned complexion. The patient has been a 1-pack/day smoker for more than 30 years, and he consumes alcohol on a daily basis. He has a crusted, rough, yellow-brown solitary lesion on the middle forehead at the hairline. He does not use any type of SPF protection and has had major sunburns throughout life; he spends the weekends on his sailboat whenever possible. Question What would be an appropriate treatment for this patient?

Correct answer: Cryosurgery Explanation This patient presents with actinic keratosis; the appropriate treatment is cryosurgery. Application of liquid nitrogen is a rapid and effective method of eradication. Because actinic keratosis is considered premalignant, not treating it could lead to the development of squamous cell carcinoma. Coal tar, topical steroids, and mupirocin ointment are not indicated for the treatment of actinic keratosis.

Case The nurse taking care of a 76-year-old woman at a nursing home notices an ulcer on the left lateral malleolus. The woman is mostly confined to bed due to severe disabling bilateral hip osteoarthritis. She has been a diabetic for the past 10 years, is on insulin, and has been recently diagnosed with depression. Examination reveals a 2x3 cm ulcer over the left lateral malleolus. There is full-thickness loss of tissue with exposure of subcutaneous fat. There is a large amount of slough in the ulcer, but there is no undermining. Sensations are intact in all dermatomes of the lower extremities. Dorsalis pedis, popliteal, and femoral arteries are bilaterally palpable. Examination of the back and lower limbs reveals another similar ulcer on the left greater trochanter of the femur. Question What is the most likely diagnosis?

Correct answer: Decubitus ulcer Explanation The correct diagnosis is decubitus ulcer. A decubitus ulcer, or pressure sore/ulcer, is a localized injury resulting from continuous pressure on the skin, soft tissue, muscle, and bone by the weight of an individual against a surface. These ulcers most commonly occur on the hip and buttocks, but may also be seen on the heels and lateral malleoli. More than 70% ulcers occur over the age of 65 years. Several risk factors exist, including prolonged immobility, diabetes, poor nutrition, chronic renal, cardiac, or lung disease, and immunosuppression. Venous ulcers are generally irregular and shallow, presenting over bony prominences. Accompanying features, such as varicosities, venous dermatitis, edema, and lipodermatosclerosis, are usually present. Neuropathic ulcers are painless ulcers typically seen in diabetics on the sole, including the tip of the toes. Surrounding callus may be present. They are commonly seen on the heel in persons with sensory deficit. Foot pulses are usually well felt. Ischemic ulcers are located on the lateral aspect of the ankle or distal end of the toes. They are painful and have a deep punched-out appearance. They are a result of reduced blood supply to the capillary bed, secondary to peripheral vascular disease. Arterial pulses in the lower limbs, especially the foot, are absent. Actinomycotic ulcers appear as cutaneous and subcutaneous swelling, which suppurate and drain through sinuses. It is endemic in tropical regions and rarely encountered in the US.

Case A 67-year-old woman presents to a dermatologist with painful vesicles on an erythematous base. He diagnoses her with herpes zoster. Question In what type of configuration would her lesions be expected to be found?

Correct answer: Dermatomal grouping Explanation Many skin lesions are grouped in specific patterns. Lymphangitis and poison ivy are typically in linear groups, though poison ivy can also be irregular. Erythema ab igne, livedo reticularis, and x-ray dermatitis have a retiform, or network-like, grouping. Erythema ab igne is a distinctive mottled, reticulated pattern of pigmented, brownish red patches with telengiectases, seen in individuals who constantly apply hot water bottle or heating pad to the body. Herpes zoster lesions follow dermatomal patterns. Erythema multiforme typically appears in an iris grouping (bull's eye), though it can also be annular (circular). The nodular lesions of late syphilis form a serpiginous grouping.

Case A mother brings her 5-year-old son to your facility presenting with several blisters on his cheek and around his nose. On examination, you note numerous vesicles and bullae containing light yellow fluid on both cheeks with minimal involvement around the nose and several outcrops of vesicles and bullae on his wrists. Question What treatment is most appropriate?

Correct answer: Dicloxacillin orally Explanation The clinical picture is suggestive of impetigo. Impetigo should be treated with an antimicrobial agent effective against Staphylococcus aureus (β-lactamase-resistant penicillins or cephalosporins, clindamycin, amoxicillin-clavulanate) for 7-10 days. Of the presented options, dicloxacillin would be most effective. Bacitracin ointment and mupirocin cream would be indicated if there was only a small area infected. Penicillin V and amoxicillin would not be effective for penicillin-resistant S. aureus.

Question A 33-year-old female patient presents to the office with multiple vesicles with associated intense itching on her hands and feet. Her past medical history is negative. The patient is a waitress who takes no medication, and does not work with chemicals. The vesicles are tense with some scales. There is no erythema or initial incidence of itching. The vesicles have responded well to a limited treatment with a high steroid cream. What is this patient's diagnosis?

Correct answer: Dyshidrotic Eczema Explanation Pompholyx or dyshidrotic eczema is a vesicular eruption with intense itching involving the hands and soles. This is generally secondary to stress. The vesicles are prone to involute and desquamate. Scales are a late finding. Treatment involves a high potency steroid. Tinea pedis occur in two forms: chronic, scaly, ruborous form and an acute bullous type. The dry scaly tinea pedis is often caused by T. rubrum whereas the bullous type is caused by T. mentagrophytes (isolated from the roof of the bullous lesion). A dry tinea contains only dermatophytes and normal flora. Once macerated, gram-negative rods (ie. Pseudomonas) plus a lower amount of dermatophytes are involved. Treatment consists of antifungals (ie. imidazoles). However, the gram-negative rods do not respond to the imidazoles and must be treated with a non-imidazole (ie. ciclopirox olamine). Contact dermatitis presents with red, itchy patches in the area of contact with the offending substance. The patches may blister, open and ooze, scale, or crust. The lesions may be self-limiting if the offending substance is avoided. Treatment includes avoidance of the substance, cool compresses, oatmeal baths. The bullous pemphigoid eruptions are self-limiting and subepidermal. They are dense, thick bullae seen frequently in the elderly. IgG's are seen along the basement membrane as this is associated with the immune system. Treatment consists of astringent soaks, systemic steroids, and immunosuppressants. Bullous diabeticorum involve spontaneous eruptions in diabetic patients. These may be epidermal or subepidermal and are self-limiting. The etiology is unknown. Treatment consists of draining and local wound care.

Case A 62-year-old man who is well known to you presents with a severely itchy rash. The patient's medical history includes Parkinson's disease for the past decade. The patient felt the rash was just eczema at first, but the areas have progressed to significant hives; the itching has become far worse. On examination, you note multiple bullae that are 1 to 3 cm in size; they are tense and appear on an erythematous base. The bullae are noted to be located on the patient's trunk as well as the bilateral axillary and inguinal folds. Question Although you have an idea of the pathology, what diagnostic study would confirm the most likely diagnosis

Correct answer: Enzyme-linked immunosorbent assay (ELISA) Explanation The patient scenario is describing bullous pemphigoid (BP). Bullous pemphigoid is an autoimmune blistering disease that most commonly arises in older adults (typically those over the age of 60). Neurological disorders have been seen to have an association with development of bullous pemphigoid and Parkinson's disease being identified as an independent risk factor in development of it. Areas affected include the trunk, extremity flexures, and axillary and/or inguinal folds. There may be a prodromal phase where the patient experiences pruritic eczematous, papular, or urticarial-like skin lesions. Classic lesions of BP are 1 - 3 cm sized tense bulla on an erythematous, urticarial, or non-inflammatory base. The blisters will be plentiful and widespread; they are also severely pruritic in nature. The correct choice above is the enzyme-linked immunosorbent assay (ELISA), which requires a collection of at least 5 ml of venous blood to be performed. Diagnosis is also aided by what is considered the gold standard, which is a skin biopsy for direct immunofluorescence (DIF). The hematoxylin and eosin staining are also conducted via a skin biopsy. Detection of viral antibodies is not appropriate in the case of BP, as this is an autoimmune process.

Case A 19-year-old woman presents with a 2-week history of rash. Other than a sore throat that she had last week, she reports being in good health; her sore throat was treated with penicillin. She does not believe that she has come in contact with any type of irritants or any individuals who are sick. On physical examination, you note several target-like lesions on the palms of her hands that are bilateral and symmetric. She indicates that she is on birth control medication. Question What is the most likely diagnosis?

Correct answer: Erythema multiforme Explanation The clinical picture is suggestive of erythema multiforme. Erythema multiforme is a cutaneous reaction to various antigenic stimuli. Examples include viral infections (herpes simplex is the most common cause) or drugs (i.e., penicillin, sulfonamides). 50% of cases are idiopathic and about 50% of patients can have flu-like symptoms up to 2 weeks before the rash develops. The lesions progress from macules to papules with a zone of central clearing, appearing as target-like lesions. Lesions can occur on skin, and/or mucosa, and severity of disease can vary widely from patient to patient. Treatment should be individualized and can include local care to skin or eyes, oral antihistamines, oral rinsing with saline, careful wound care, analgesics, and topical steroids. Any drug that is thought to cause the reaction (and any non-necessary drugs) should be discontinued. Patients with severe reactions (e.g., denuded skin, inability to eat or drink due to mucosal lesions) may need hospitalization. Erysipelas is an acute group A strep skin infection; it extends into underlying fat tissue and affects lymphatics that favor the lower extremities. Skin lesions enlarge rapidly; they usually appear as sharply demarcated, red, swollen, warm, and indurated. Red streaks are usually also present. Atopic dermatitis is characterized mainly by dry skin and pruritus, primarily on the flexural surfaces. Acute reactions appear as erythema progressing to papules and plaques with flaking, xerosis, cracks, fissures, and excoriations. Erythema nodosum appears as tender red nodules on the anterior surfaces of the lower leg. Steven-Johnson syndrome is a mucocutaneous drug-induced or idiopathic reaction pattern characterized by skin tenderness and erythema of skin and mucosa; this is followed by extensive cutaneous and mucosal epidermal necrosis and sloughing.

Case A 7-year-old boy presents with an "itchy scalp". His mother reports the child has been scratching his head frequently for the last week. She cannot see a rash; he denies fever, chills or other symptoms of viral illness. The child has not had any dermatologic problems in the past, and he has been healthy overall. The child just says his head "itches a lot", so he scratches it. The teacher has noticed his behavior, which has become disruptive in class. The child's past medical history is unremarkable; he has no medical conditions. He has not had any surgeries. He takes no medications, and he does not have any allergies. Family history is negative for atopy and psoriasis. He lives at home with his 2 younger siblings, mother and father. He is in 1st grade and has met normal developmental milestones. On physical exam, the child appears restless and fidgety. On exam of his scalp, excoriations and small buds are noted on the hair shaft, close to the scalp. The rest of his exam is normal. Question What is the best intervention for this patient's condition?

Correct answer: Education about the condition and topical permethrin Explanation This child is presenting with head lice (pediculosis). The small buds on the scalp may be eggs or nits (empty egg casings). This condition is common in overcrowded areas and with sharing of hats and combs. Education about the condition and topical permethrin is the best intervention. The child and parent should be educated about disposing of or sanitizing the infested clothing and bedding and treatment. Topical rinses of permethrin are available over-the-counter, and they are a common first-line treatment. It may have to be repeated, and resistance can occur. Fungal infections, such as ringworm, may cause pruritus of the scalp. However, education about the condition and a systemic antifungal is not appropriate treatment for pediculosis. This patient should not be simply dismissed as having inadequate hygiene, so it would be inappropriate to recommend improved hygiene (including better shampooing). The lice would persist, despite rigorous shampooing with normal hair products, and could spread to others in the household. When a clear cause of pruritus is not identified, it may be acceptable to offer reassurance and a systemic antihistamine as needed for itching. That would not be adequate for this patient. If no lice was identified, and this child's other behaviors had recently changed both at home and in the classroom, psychiatric disorders should be considered. However, a referral to child psychiatrist is not necessary in this case. His disruptive classroom behaviors should resolve with treatment of his lice.

Case A 3-year-old girl is brought to the emergency department by her parents. They noticed the abrupt onset of a rash that began on her trunk and extended to her extremities symmetrically. The rash has been accompanied by fever, and her parents note that she napped longer than usual today. She has no chronic medical conditions, but her mother states that she did have a cold sore recently. On examination, the following is seen. Refer to the image. Question What is the most likely diagnosis?

Correct answer: Erythema multiforme Explanation The patient had sudden onset of target lesions as seen above. Target lesions are classic for erythema multiforme. The diagnosis is further suggested by her recent cold sore, which is consistent with herpes simplex, one of the triggers for erythema multiforme. Impetigo presents with honey-colored crusted lesions. Patients with erysipelas present with a sharply demarcated erythematous plaque. Erythema infectiosum presents with red cheeks with circumoral pallor and macropapules on the extremities. Patients with varicella develop a progression of macules to papules, vesicles, and crusts.

Case A 56-year-old woman has had multiple skin issues over the years. She has been on long-term treatment with methotrexate for psoriasis vulgaris, with occasional topical steroid application. She also has increased sensitivity to sunlight. She has a questionable lesion pictured below. The lesion is shown in the image. Question What is the next best step in management?

Correct answer: Excisional biopsy Explanation Excisional biopsy is the correct answer. Given the obvious hyperpigmentation, irregularity, asymmetry, and size of the lesion, melanoma must be the diagnosis unless proven otherwise. Urgent further evaluation is necessary, and an excisional biopsy is the first step. Shave biopsy is incorrect. Although current guidelines suggest that it may be an option for very early melanoma in situ, shave biopsies have been criticized for not providing adequate information regarding the tumor invasion. Cryotherapy is incorrect. Cryotherapy is absolutely contraindicated in a suspected melanoma. Furthermore, it does not make a diagnosis. Close observation and follow up is incorrect. If melanoma is suspected, it must be evaluated urgently. Given the nature of the lesion (hyperpigmentation, irregularity, asymmetry, and size of the lesion), a diagnostic test is essential. Fine needle aspiration is incorrect. Needle aspiration is typically done for lesions below the level of the dermis. It is not used in superficial lesions.

Case A 50-year-old man presents because of a mole his hairdresser noticed on the back of his neck. The patient says that he had felt some itchiness in the area of this mole over the past few weeks. He also has multiple other moles on his back, arms, and legs. He spends most of the work week indoors, but he typically spends his weekends gardening. The rest of his personal and family history is not contributing. Physical examination is within normal limits, except for a presence of 1.7 x 2.3 cm pigmented mole on the back of his neck. Question What will be your next step in the diagnosis?

Correct answer: Excisional biopsy with narrow margins Explanation A lesion on the neck is suspicious of malignant melanoma. You will choose the excisional biopsy to remove the entire lesion with narrow margins and depth through the subcutaneous tissue because some other lesions may present with similar clinical picture: pigmented basal cell carcinoma, seborrheic keratoses, atypical nevi, etc. In those cases, wide excision is not necessary. Besides, if you confirm the diagnosis of malignant melanoma, the depth of the lesion will guide your next step: complete removal with 1 cm tumor free margins or sentinel biopsy. Shave biopsy removes only a part of the lesion, so that the depth of the lesion cannot be estimated (the depth is one of the most important factors in the evaluation of the malignant melanoma). Wide margin excision is not recommended until the diagnosis of malignancy is confirmed. Besides, it can prevent the identification of sentinel nodes by disrupting the lymph flow. Before you decide about the sentinel lymph node mapping and biopsy, you should confirm the presence of the malignancy. Core biopsy is not indicated in this case because it cannot remove the entire lesion.

Case A 4-year-old girl is brought by her mother to the ED for swelling and redness of the left elbow. The mother tells you that the child fell onto the elbow 4 days prior and had sustained a small abrasion. The child scratched and picked at the wound for 2 days and subsequently developed redness around the site and purulent drainage from the wound. Yesterday the elbow became quite swollen and the child had a fever of 103° F. The mother states that the area of redness has increased rapidly over the past 24 hours. A photograph of the involved area is shown. Question What organism is most likely to cause these findings?

Correct answer: Group A Streptococcus Explanation The patient has a left elbow cellulitis most likely caused by Group A Streptococcus. Group A Streptococcus is the causative agent for many pediatric infections, most commonly acute pharyngitis, impetigo, pneumonia, bacteremia, and vaginitis. Trauma to the skin, such as a laceration, puncture wound, or as in this case, an abrasion, predisposes to the formation of cellulitis. Another common organism causing cellulitis is Staphylococcus aureus. The classic findings of cellulitis are local tenderness, erythema, and pain. Often fever, chills, and malaise may develop. The lesion is usually red, warm, and swollen. Toxic shock syndrome and septic arthritis are 2 other conditions that can be caused by both staphylococcus and streptococcus. The other organisms listed are less common causes of cellulitis. Pseudomonas aeruginosa, a gram-negative bacillus, usually causes a gangrenous cellulitis characterized by extensive necrosis of the skin and subcutaneous tissue. It is usually associated with disease in those patients who are immunocompromised. Clostridium perfringens (gram-positive anaerobic bacilli) and Peptostreptococcus (also anaerobic) cause necrotizing infections of devitalized tissue. They are most often seen in wounds that are dirty or inadequately debrided. Enterococci (gram-negative bacilli) rarely cause cellulitis by themselves; they frequently are found mixed with gram-positive bacilli and anaerobes in surgical wound infections, diabetic foot ulcers, and decubitus ulcers.

Question A 27-year-old female comes to your clinic complaining about painful, itchy blisters on her private parts. She is G0P0, has a regular monthly cycle (29/5), menarche at 13, and has had a steady partner for two years. She does not use any method of birth control, including condoms. She states that both, her partner and she are monogamous and he does not have any symptoms. Two days earlier, she had been feeling sick. She was taking OTC aspirin, but still has an elevated temperature of 37.5° C. Upon examination you palpate enlarged lymph nodes in the inguinal area and see multiple vesicles on her labia and perineum, some ruptured, and some crusted over. There is no vaginal discharge and the rest of the pelvic exam is unremarkable. Which test will confirm your clinical diagnosis?

Correct answer: HSV culture Explanation The symptoms described above are highly suspicious of herpes genitalis, an infection caused by the herpes simplex virus (HSV). The painful vesicular and ulcerated lesions are 1-3 mm in diameter, occur 3-7 days after exposure, and resolve over 7 days. The disease can cause severe morbidity and can reoccur. It is highly contagious and the virus can be shed without visible evidence of active disease. There is no cure, but the number of recurrences can be reduced by antiviral treatment like acyclovir, famciclovir, or valacyclovir. Infection with the human immunodeficiency virus (HIV) does not cause any skin changes in the urogenital area. Chlamydia infection is characterized by a thin, whitish, non-odorous discharge, but can also be asymptomatic. It is dangerous to the effect that it can lead to salpingitis and subsequently infertility. RPR, rapid plasma reagin test, is used to diagnose syphilis. In the first stage, there is regional adenopathy and the chancre that shows on the site of inoculation is painless and disappears after 1-5 weeks. The second stage is characterized by general lymphadenopathy, malaise, fever, and mucocutaneous lesions, which resolve after a few months. After a period of latency, which can last for many years, tertiary syphilis affects internal organs, vascular and central nervous systems. A wet mount prep can help you diagnose candida (cheesy white discharge, itchy burning erythema), bacterial vaginosis (white, gray, or yellowish discharge, fishy odor, maybe pruritus), and trichomoniasis (green, frothy discharge, rancid odor, severe itching, maybe inflammation with small, strawberry-like dots).

Case A 40-year-old man presents with a 3-day history of right-sided chest pain. The patient reports that the pain is gradually increasing in intensity, burning in character, and like a band across his mid-chest and mid-back on the right side. Today, he has noticed a light rash in this area. He has no fever, but he is experiencing fatigue and malaise. His past medical history includes GERD (for which he is on lansoprazole daily), asthma (for which he uses an albuterol inhaler daily), and knee arthritis (for which he uses over-the-counter low-dose naproxen prn). Family history is noncontributory. On examination, he is afebrile with a BP of 128/80 mm Hg and a pulse of 70 bpm. Oral exam is unremarkable. Lungs are clear, and the abdomen is benign. On the right side of his chest, there are small vesicles with surrounding erythema in the mid-region. The skin is excruciatingly tender in a dermatomal fashion along the affected area. Question What is the most likely diagnosis?

Correct answer: Herpes zoster infection Explanation Chest pain can have multiple etiologies. Right-sided chest pain is usually noncardiac. It could be from the gastrointestinal tract, lungs and pleura, musculoskeletal, neurological, or referred pain. In this instance, the dermatomal distribution of burning pain associated with a vesicular rash indicates herpes zoster infection, which is a frequent condition seen in the office. All patients who have had chickenpox earlier are susceptible to get this from reactivation of the same virus hiding in the neural cells. Stress of any kind plays an important role. The activated virus then causes severe neuralgia and vesicular eruption in a dermatomal distribution. The most common sites are thorax, neck, face, and lumbosacral area. Diagnosis is clinical. Treatment includes general measures such as relative isolation until the crusts disappear, pain control usually with opioids and membrane stabilizers (e.g., carbamazepine, gabapentin), antihistamines for pruritus, and antipyretics. Antiviral therapy includes acyclovir, famciclovir, or valacyclovir for 7-10 days. Zoster usually resolves in 2-6 weeks, but it may be accompanied not only by pre-herpetic and herpetic neuralgia, but also post-herpetic neuralgia for an indefinite amount of time. NSAID-induced esophagitis, gastritis, or GERD are common causes of chest pain of the burning variety, but they are not associated with a rash and there is no local cutaneous tenderness. Costochondritis is also a common and frequent cause of non-cardiac atypical chest pain; the pain is due to inflammation of the costal cartilages. The pain is reproducible at the junctions of the ribs and cartilages, without any rash, erythema, or warmth. Candida intertrigo is a superficial infection with candida that is seen in opposed surfaces and folds of skin, such as under the breast, axilla, inguinal areas, and abdominal folds. There is an absence of pain, and the rash is typically erythematous plaques or maculopustular in appearance. Herpes simplex is a cutaneous infection, unlike herpes zoster, which is a neuronal infection. Type I virus causes herpes simplex labialis or cold sores on the lips, and type II causes genital infections. There is no dermatomal distribution. HSV1 is considered more responsible for producing oral lesions, and HSV2 is responsible for genital lesions.

Case A 21-year-old woman presents with recurrent painful nodules that form in her armpits. On physical examination, you note red inflammatory nodules that are very tender to palpation. Also noted are open comedones that seem to be paired. The patient indicates that these areas ultimately break down and drain a foul-smelling, purulent material. Question What is the most likely diagnosis?

Correct answer: Hidradenitis suppurativa Explanation The clinical picture is suggestive of hidradenitis suppurativa. The age of onset begins at puberty and it affects girls/women more than boys/men. The initial lesions appear as red inflammatory nodules and/or abscesses that are very tender. They may resolve, or they may drain purulent material. The lesions may reoccur in the same area. Their distribution is localized to the axillae, breasts, anogenital area, and groin. They may have a unique "double comedone" appearance that is highly characteristic of this disorder. Roth spots are retinal hemorrhages. Sebaceous gland hyperplasia is a common lesion seen in older individuals. They appear as 1-3 mm in diameter and have central umbilication and telangiectasia, similar to basal cell carcinoma. Cellulitis is an acute, spreading infection of the skin; it is characterized by red, hot, tender areas of the skin. Acanthosis nigricans is a velvety, dark brown thickening of the skin.

Case A 50-year-old Hispanic man presents with an oval skin growth on his forehead. He noticed the growth 2 weeks ago, and it has gradually grew in size and is now bleeding spontaneously. He has been a farmer for the past 30 years and often works in the fields. He gives a history of cocaine abuse in his 20s. He is a diabetic and hypertensive on treatment with Metformin and Atenolol, respectively. On exam, there is an erythematous papule about 3x3 cm in size, which bleeds on probing. His BMI is 30. Biopsy of the lesion reveals squamous cell carcinoma (SCC) of the skin. Question What is a risk factor for SCC in this man?

Correct answer: His gender Explanation Squamous cell carcinoma (SCC) of the skin is more common in men than women in the ratio 2:1 due to greater cumulative sun exposure during their lifetime. SCC of the skin is more common in Caucasians than other races. Blue or green eyes and light-colored hair are also considered risk factors. Type I and type II skin, history of sunburn or indoor tanning, and immunosuppression are other risk factors. Smoking, and not cocaine abuse, is a risk factor for SCC skin. Obesity and atenolol, as such, are unrelated to SCC of the skin.

Case A 3-year-old boy presents with a 6-day history of an itchy, red wound on the right knee. The mother ignored it initially, assuming that the boy was injured while playing; however, the wound did not heal and seems to have worsened. On examination, the child is afebrile. There are a few intact vesicles and a few ruptured vesicles covered with honey-colored crusts on the right knee. You send a swab from one of the lesions, and Staphylococcus aureus is detected. Question What is the most probable diagnosis?

Correct answer: Impetigo Explanation Impetigo is a contagious, pyogenic infection classified into bullous and non-bullous types. The non-bullous type is caused by Staphylococcus aureus or Streptococcus pyogenes and is more common on the exposed parts of the body. An initial breach in the skin barrier (e.g., trauma, insect bite) occurs, followed by an infection. It is most common between the ages of 2 to 5 years, and in those with poor hygiene or living in overcrowded settings. It is a self-limiting condition and heals without scarring in a few weeks. Treatment aids in reducing transmission and relieving discomfort. Treatment consists of proper wound care and topical antibiotics (e.g., mupirocin or retapamulin). The bullous type is characterized by large bullae, along with fever, diarrhea, and weakness. A carbuncle is a large abscess on the skin, also caused by Staphylococcus aureus; there is pus drainage onto the skin. Cellulitis is a diffuse infection of the skin presenting with fever, tight, warm inflamed skin, and necrotizing fasciitis involving the deep fascia, along with necrosis of subcutaneous tissues. A furuncle is a type of folliculitis involving the hair follicle. The vesicles and honey-colored crusts are more indicative of impetigo and not the other conditions.

Case A 7-year-old boy presents with a rash on both legs. The rash started several days ago while he was summer camping. He tells you that some of his friends had similar changes on their skin, but they were not as extensive as he has now. His past medical history is significant for atopic dermatitis that was never put under complete control. On examination, you find several vesicles on his ankles, a few honey brown crusted lesions with an erythematous bases, and several other lesions in various stages of crusting and oozing. Some of them itch. The rest of the examination, including local lymph nodes status, is normal. Question What is the most likely diagnosis?

Correct answer: Impetigo Explanation Your patient most probably has impetigo, a superficial skin infection typically caused by Streptococcus pyogenes group A. It is associated with hot weather; it can be transmitted from person to person under the condition of poor hygiene, and it can be secondary to his pre-existing atopic dermatitis (superinfection). It presents as honey brown crusted lesions with erythematous bases. This classical description means that you do not need to perform further investigation (cultures); you should simply give him a topical antibiotic. Scabies is characterized by intensive pruritic lesions; they appear more frequently during the night and are usually localized interdigitally, in axillary folds, areolas, and wrist flexors. It appears as red papules with excoriations, crusting, and scaling; it is threadlike (like burrows under the skin). Superinfection is also possible. Dyshidrosis is a non-contagious acute, chronic, or recurrent likely allergic dermatosis of the fingers, palms, and soles; it is characterized by a sudden onset of many deep pruritic, clear vesicles. In the later stages of scaling, fissures and lichenification can occur. The description of skin changes in your patient does not fit dyshidrosis. Atopic dermatitis is a chronically relapsing, non-contagious, and pruritic inflammation of the skin. It is a familiar chronic disease that can occur together with other atopic diseases (asthma, hay fever, conjunctivitis, etc.). Abnormal skin reaction to irritants, food, and other allergens manifests as red, flaky, and very itchy lesions, usually on flexural surfaces of the joints. The skin also becomes prone to surface bacterial infections. The description of skin changes in your patient does not fit atopic dermatitis. Varicella-zoster will manifest as diffuse crops of pruritic papules, vesicles, ulcers, and crusts in different stages of development. It is transmitted via respiratory secretions and is not localized.

Question An 18-year-old college student presents with a bright red rash on her left cheek area that has worsened since yesterday when it first appeared. It is now becoming more tender and she developed a temperature elevation of 100.2°F taken at home. She denies any new soaps or facial creams, and wears occasional make-up. She denies any ill contacts. Her current temperature is 101.2 and she feels ill. Your most likely diagnosis is erysipelas. She denies any medication allergies. The most important next step in the management of this patient is

Correct answer: Intravenous penicillin G for 48 hours, then Penicillin VK 250 mg 4 times daily for 7 days Explanation Erysipelas is a cellulitis caused by beta-hemolytic streptococci and causes a painful, bright red area that is typically seen on the cheek often near the angle of the nose. If this disease is not treated promptly, it can become systemically toxic and may result in death, especially in the very young and very old. In most cases, IV administration of an antibiotic active against beta-hemolytic streptococci and staphylococcus for the first 48 hours is indicated followed by oral administration of similar drug for 1 week. If it is a very mild case, IV antibiotics are not always indicated, but in this case, a fever of 101.2°F would constitute a more severe infection warranting IV antibiotics first. Blood cultures and CBC are not correct as prompt treatment is necessary in this case and your diagnosis has already been made. These tests will not add much to your management, especially as blood cultures are not always positive. AST and ALT are not useful here. Erythromycin is not the first-line drug of choice but may be used if the patient is allergic to penicillin.

Case A 79-year-old Italian man presents with a 6-month history of lesions on his legs. He is not taking any medications. On examination, you note that he has multiple painless purple papules on his lower limbs. Question What is the most likely diagnosis?

Correct answer: Kaposi's sarcoma Explanation The patient's Mediterranean origin and typical lesions on the legs make the diagnosis of Kaposi's sarcoma likely. Kaposi's sarcoma (KS) is caused by human herpesvirus type 8 and is thought to arise from endothelial cells. There are several forms, including: Classic KS—an indolent form, usually occurs in elderly men of Mediterranean origin without AIDS, such as this patient. Typically, it presents as purple plaques or nodules on the lower limbs. Treatment is by cryotherapy, electrocoagulation, or radiotherapy. AIDS-related form—the most aggressive form, with widely disseminated purple papules or plaques on the skin, mucous membranes, and viscera. Treatment is by chemotherapy. Lymphadenopathic form—involves lymph nodes and skin. There are several types of malignant melanomas, including: Superficial spreading melanomas— plaques with blue-black spots Nodular melanomas—rapidly growing dark papules or plaques Lentigo maligna melanomas—brown macules with scattered darker spots on sun-exposed areas, such as the face Acral lentiginous melanomas—most common form in African-American patients; they develop on the palms, soles, subungual skin, and mucous membranes Patients with basal cell carcinomas (BCC) can present with a shiny pearly nodule (noduloulcerative BCC), or with erythematous scaling plaques (superficial BCC), or with a solitary flat yellowish plaque (morpheaform BCC). Patients with squamous cell carcinomas (SCC) may present with a red papule or crusted plaque on the lips, ears, neck, or hands; it may ulcerate, invade the underlying tissue, and metastasize. In Paget's disease of the breast, patients present with a unilateral, sharply marginated, red, scaly rash affecting the nipple and areola. There may also be an underlying mammary duct carcinoma.

Case A 30-year-old woman presents with a 2-week history of itchy pimples on her wrist. On examination, there are violaceous papules with a network of grey lines on their surface; they are found on the medial aspect of her right wrist. Question What is the most likely diagnosis?

Correct answer: Lichen planus Explanation Lichen planus is characterized by pruritic, polygonal papules which are flat-topped and violaceous in color and have a network of gray lines (Wickham's striae) on the surface. They commonly develop on the wrists, legs, and genitalia. Management includes using topical glucocorticoids, though many patients have spontaneous remissions. Asteatotic eczema is characterized by fine cracks which may be erythematous, usually on the pretibial surfaces of elderly people during winter; they may also be pruritic. Management includes application of topical emollients and avoidance of irritants. Stasis dermatitis is characterized by erythematous, pruritic scales with oozing patches, usually over the medial aspect of the ankles. Hyperpigmentation and stasis ulcers may also be present. It develops on the lower limbs due to chronic venous incompetence. Management includes using topical glucocorticoids and emollients, as well as avoiding irritants. Contact dermatitis in the acute phase is characterized by sharply demarcated, weeping, edematous, eczematous plaques. There is usually a history of contact with an allergen. Management includes avoidance of irritants and application of topical glucocorticoids. Lichen simplex chronicus consists of a well-circumscribed, scaly plaque with lichenification and hyperpigmentation due to chronic scratching or rubbing (which can also occur during sleep). It commonly involves the occiput, back of the neck, arms, dorsum of the feet, and ankles.

Case A 25-year-old woman presents with a 1-year history of an itchy lesion on the back of her neck. She is a medical student, and she says the itchiness increases when she is studying for exams; her roommate has even told her that she scratches it when sleeping. On examination, she has a scaly, well-circumscribed, lichenified plaque on her posterior nuchal region. Question What is the most likely diagnosis?

Correct answer: Lichen simplex chronicus Explanation Lichen simplex chronicus consists of a well-circumscribed, scaly plaque with lichenification and hyperpigmentation due to chronic scratching or rubbing (which can also occur during sleep). It commonly involves the occiput, back of the neck, arms, dorsum of the feet, and ankles. Management includes the use of oral antihistamines, topical, or intralesional injection of, glucocorticoids, as well as breaking the itch-scratch cycle. Stasis dermatitis is characterized by erythematous, pruritic scales with oozing patches, usually over the medial aspect of the ankles. Hyperpigmentation and stasis ulcers may also be present. It develops on the lower limbs due to chronic venous incompetence. Management includes using topical glucocorticoids and emollients, as well as avoiding irritants. Patients should also be encouraged to wear compression stockings and elevate the affected limb as much as possible. Asteatotic eczema is characterized by fine cracks which may be erythematous, usually on the pretibial surfaces of elderly people during winter; they may also be pruritic. Management includes the application of topical emollients and avoidance of irritants. Contact dermatitis in the acute phase is characterized by sharply demarcated, weeping, edematous, eczematous plaques. There is usually a history of contact with an allergen. Management includes avoidance of irritants and application of topical glucocorticoids. Lichen planus is characterized by pruritic, polygonal papules; they are flat-topped and violaceous in color, and they have a network of gray lines (Wickham's striae) on the surface. They commonly develop on the wrists, legs, and genitalia. Management includes using topical glucocorticoids, though many patients have spontaneous remissions.

Case A diagnosis of malignant melanoma is made on an otherwise healthy 45-year-old man. The lesion is on his right big toe. In close consultation with his dermatologist and oncologist, a decision is made to try isolated limb perfusion to the affected limb. Question What is the most appropriate chemotherapeutic agent for infusion?

Correct answer: Melphalan Explanation Melphalan is the correct answer. Melphalan is the drug of choice in treatment of malignant melanoma when using isolated limb perfusion. It belongs to the category of nitrogen mustard alkylating agents. Other uses include ovarian cancer and multiple myeloma. Interferon gamma is an incorrect response. While used as a component of some chemotherapeutic regimens, it has been associated with mixed results as well as severe side effects. Vincristine is an incorrect response. Vincristine is a vinca alkaloid; it is commonly used in lymphomas and leukemias. Vinblastine is an incorrect response. Vinblastine is a vinca alkaloid; it is used in cases of Hodgkin's lymphoma, non-small cell lung cancer, breast cancer, head and neck cancer, and testicular cancer. Cyclophosphamide is an incorrect response. It is a nitrogen mustard alkylating agent used in lymphomas, some forms of brain cancer, leukemia, and some solid tumors.

Question Refer to the image. The 26-year-old male seen in the image is HIV positive with a CD4 count less than 200. While he was in the hospital for the treatment of his miliary Tb, he developed smooth, skin-colored, umbilicated papules on his face. The lesions are asymptomatic but spreading gradually to the other parts of the body and causing cosmetic problems to the patient. What is the name of this skin infection?

Correct answer: Molluscum contagiosum Explanation Molluscum contagiosum is a poxvirus infection characterized by skin-colored, smooth, waxy, umbilicated papules 2 to 10 mm in diameter. Transmission can be direct or venereal. Its contagiousness to others varies. The lesions may be seen anywhere on the skin, face, body, or genital area. They are usually asymptomatic unless secondarily infected. This disease is quite common in children, and the lesions can be widespread in patients with reduced cellular immunity. Verrucae vulgaris is the other name for common warts. They are sharply demarcated, rough-surfaced, round, irregular, light gray, yellow, brown, or grayish black lesions. Filiform warts are long, narrow, small growths of skin, usually seen on the eyelids, face, neck, or lips. Condyloma acuminata is the name for genital warts. They are moist, soft, pink, or red swellings that grow rapidly and become pedunculated. They most commonly occur on warm, moist surfaces in the subpreputial area, perineum, vaginal wall, cervix, penile shaft in men, and urethral meatus. Kaposi's sarcoma is a vascular skin tumor associated with AIDS. They may be the first notable manifestation of AIDS. The lesions start as pink or red papules, round or oval, and appear first on the upper body and mucosa. They may become widely disseminated on the skin and are associated with organ and lymph node involvement. This patient does not have the typical Kaposi's sarcoma lesions.

Case A 9-year-old boy has 11 irregularly shaped, 2 X 3 cm, hyperpigmented lesions on the skin of his trunk. He has freckles in his axilla. He recently developed several cutaneous and subcutaneous masses and some visual disturbances. Question What is the most likely diagnosis?

Correct answer: Neurofibromatosis Explanation This child has neurofibromatosis. The irregularly shaped, 2 X 3 cm, hyperpigmented lesions on his skin are café au lait spots. He has freckles in his axilla. The cutaneous and subcutaneous masses are neurofibromas. They frequently do not develop until late in childhood or adolescence. This patient has also developed a glioma on the optic nerve, resulting in visual disturbances. Neurofibromatosis is caused by autosomal dominant mutations. The condition has no cure. There are 2 major varieties: NF-1 (incidence 1/3000) and NF-2 (1/25,000). This disease is characterized by tumors forming on nerve fibers. In addition, some patients have hamartomas of the iris (Lisch nodules). Tuberous sclerosis is manifest by adenoma sebaceum, small bright red or brownish nodules in a butterfly distribution over the nose and cheeks. These children have CNS lesions leading to seizures and intellectual disability. Large, often unilateral cafe au lait spots characterize McCune-Albright syndrome. There are associated bony and endocrine disorders. Encephalotrigeminal angiomatosis (Sturge-Weber syndrome) is characterized by a congenital capillary hemangioma on the skin, often in the trigeminal distribution on the face, and meningeal hemangioma that may cause seizures. Spider angiomas consist of a central artery with enlarged vessels radiating outwards and a surrounding erythematous flush.

Case A 23-year-old woman with non-insulin dependent diabetes presents with a chief complaint that the "sore on the bottom of my foot doesn't heal and gets my socks wet". The patient states the "sore" presented gradually and has slowly worsened. The patient's past medical history (PMH) is negative with the exception of Type II diabetes. Physical exam of the right foot shows a 1cm, partial thickness ulcer present on the plantar aspect of the 5th MTPJ. The borders are well defined, with white hyperkeratosis and clear serous drainage. There is no pain upon palpation. Question Ico-delete Highlights What is the most likely type of ulcer?

Correct answer: Neurotrophic Explanation Neurotrophic ulcers occur on the weight-bearing area of the foot. The depth varies from superficial to deep. The size varies and pain is absent because of the loss of sensation to the lower extremity. A vasospastic ulcer presents on the toes. It generally presents with severe pain, is small in size, and has indolent borders. Skin changes, such as mottling, occur secondary to vasospasticity. Ulcers associated with venous insufficiency occur on the medial aspect of the lower leg. They have an irregular shape and vary in size. The ulcers are superficial in depth and are associated with mild pain. A hypertensive ulcer generally presents on the lower legs (i.e., posterior-lateral aspect). The shape is irregular and the depth is superficial. It is accompanied with severe pain. Hematologic ulcers generally occur on the malleolar area (laterally) and present as a punched-out lesion that is generally deep-set. Mild pain may be an associated factor.

Case A 23-year-old Caucasian woman complains of severe itching and burning pain in her earlobes. She has a 3-day history of wearing new ear studs. On examination, there is redness and inflammation of both ear lobules with a few blisters. You suspect allergic contact dermatitis to 1 of the metals in the ear studs. Question Ico-delete Highlights What metal is the most likely cause?

Correct answer: Nickel Explanation Nickel is the most common cause of allergic contact dermatitis among metals. The prevalence of nickel allergy varies across countries and ranges from 4 - 13%. Women are more commonly affected; the most common causes are ear piercing and fashion jewelry containing free nickel. Men may experience a nickel allergy through occupational exposure. Hairdressers are at a higher risk; the prevalence rate is 27 - 38% in this population. An allergy to copper jewelry is due to its nickel content. Other metals (e.g., gold, silver, and aluminum) may also cause allergy, but are not as common as nickel allergy. An aluminum allergy may occur after vaccinations, exposure to deodorants, cans, etc.

Case A 4-year-old Caucasian boy is seen in the outpatient clinic with a 3-week history of generalized rash. Pruritus was minimal, but has resolved and he has not had a fever. The mother noted that the rash began as a single lesion on the abdomen that grew in size as the rash progressed elsewhere and is now the largest lesion present. Examination revealed an otherwise well, afebrile child with a generalized rash consisting of scaly, reddish-pink plaques concentrated on the trunk in a Christmas tree pattern with a single lesion, greatest in size, at the abdomen. Question What is the optimal treatment at this time?

Correct answer: Observation Explanation Pityriasis rosea is a benign, self-limited rash frequently seen in childhood. Typically, it begins with a single oval, reddish pink plaque on the torso that becomes scaly (herald patch), followed by similar lesions concentrated on the trunk in a pine, or Christmas tree, distribution. Treatment is aimed at pruritus, if present. Coal tar is an antiquated remedy for a number of dermatologic conditions. There is no necessity for oral prednisone in asymptomatic pityriasis rosea. Pityriasis rosea is not a dermatophytosis; therefore, antifungals are of no value. Topical hydrocortisone could be used for significant pruritus, which is not present here.

Case An 82-year-old man presents for evaluation of an itchy, hive-like rash on his abdomen. He has had it for months. Over the last few days, the patient reports the character of the rash has changed; it now resembles blisters. Other than some skin irritation, he feels healthy. He has tried multiple over-the-counter topical treatments, but they have been unsuccessful in treating the rash. He denies any changes in soaps, lotions, laundry detergents, or anything else that may have come in contact with his skin. He also denies unusual travel, pets, or hobbies. No close contacts have reported a similar condition. He takes no medications, and he does not have any chronic illnesses. On physical exam, multiple clusters of bullae are noted across the trunk bilaterally, with some distribution on both anterior and posterior surfaces. There are still some remaining pink-red lesions; they are scattered among the bullae. The bullae are 1 - 3 cm in size; they are tense and do not easily rupture. They do not extend into normal skin with pressure. Pressure on the normal skin does not produce a blister. The remainder of his physical exam is normal. Question Once you confirm the diagnosis, what intervention is most appropriate for this patient's condition?

Correct answer: Oral prednisone Explanation This patient is presenting with bullous pemphigoid (BP), a benign autoimmune skin disorder, characterized by the presence of bullae (or blisters). BP is relatively rare and tends to affect the elderly. The preceding rash is typically pruritic and appears as urticarial plaques, possibly resembling erythema multiforme. This stage may last for months, and then the skin blisters. When the blisters rupture (typically within 1 week), the skin heals. Unless the BP is extensive and generalized, the prognosis is good. Oral steroids, such as prednisone, have been the primary treatment; however, other approaches, such as antihistamines for pruritus, topical steroids, antibiotics, methotrexate, and azathioprine may all have a role in treating BP. The use of trichloroacetic acid (TCA) or bichloroacetic acid (BCA) is most appropriate for condyloma acuminata (genital warts). These acids are caustic agents that destroy tissue. They are not indicated in the treatment of BP. Cryotherapy of bullous lesions is not recommended, especially as the bullae tend to resolve spontaneously in approximately 1 week, with normal healing expected. Cryotherapy is not appropriate. Dermabrasion is a popular skin procedure for aging skin. It consists of mechanical removal of surface skin, producing a similar effect to chemical peels. Neither dermabrasion nor chemical peels are good treatments for BP. Topical acyclovir is an antiviral, which is useful in the treatment of herpes conditions. Herpes family infections could be on the differential, especially with a blister-like appearance, but herpes vesicles are typically much smaller, and none of the herpes infections fit the presentation and distribution of this patient's "rash". BP is an autoimmune condition, and an antiviral medication would not be appropriate. Topical imiquimod is also a treatment for condyloma acuminata (genital warts). It has no role in treating BP.

Case A 9-year-old girl presents with itching. 10 days ago, she attended a slumber party; 5 of the 7 girls who attended are now having similar symptoms. The mother states that the patient is constantly itching her scalp as well as the back of her neck. When asked why, she states it's because she feels as if something is crawling on her. Upon physical examination you note multiple pin-sized yellow, brown, and tan colored nits on individual shafts of the patient's hair. Question Based on the most likely diagnosis, what is the recommended clinical intervention at this time?

Correct answer: Permethrin (1%) Explanation The correct response is Permethrin (1%). The patient described in the above scenario is displaying classic signs and symptoms of an active head lice infestation, which is also known as pediculosis capitis. In the US, head lice manifestation is most common among preschool- and elementary school-aged patients. Transmission of actual lice in most cases is by direct contact with the head of an infested patient. Indirect contact may also lead to infestation; this occurs through contact with the infected person's personal belongings (combs, brushes, hats). Patients will more commonly complain of severe itching of the head as well as even the nape of the neck, a sensation of something crawling on them, visually seeing the louse and/or eggs (termed nits), and in very rare or severe cases, cervical adenopathy or impetigo. The gold standard for diagnosing head lice is finding live louse on the head; however, it is more commonly done by visualizing the tiny eggs that they lay. These are usually located within 1 centimeter of the scalp, and they can be more easily seen at the nape of the neck or behind the ears. Eggs are attached to the shaft of the hair and cannot be removed easily. A practitioner also has to be mindful that many other objects have and can be mistaken for lice nits, including dandruff, hair casts, scabs, dirt, hair spray droplets, or even other insects. Treatment should not be initiated unless there is clear diagnosis of head lice. Permethrin 1% is the most studied pediculicide in the US and is the least toxic to humans. It is considered first-line treatment and is the correct answer. Malathion 0.5% has also been used as a treatment for head lice, but it is not the first choice. This product has been taken off the market 2 times, most recently due to long application time, a strong odor, and flammability potential. It contains an extremely high alcohol contact (78% isopropyl alcohol); therefore, very strict instructions must be given to the patient and their parents to avoid any situations of extreme heat or fire (hair dryers, curling irons, smoking, etc.). Lindane 1% is available for treatment of lice and scabies; however, there is a known side effect of central nervous system toxicity in humans; reported seizures have occurred in children who have used this product. Benzyl alcohol 5% is also an acceptable treatment; however, it would not be the preferred treatment in the patient above due to the fact that common side effects seen include pruritus, erythema, pyoderma, and even ocular irritation. Crotamiton 10% is not indicated for treatment of head lice, but it is considered a treatment option for scabies. Studies have suggested potential use in patients with active head lice; however, no conclusion has been made on the safety of its use in children, adults, or pregnant women.

Case A 30-year-old man presents with a facial outbreak. He indicates that it comes and goes, and he says that he was not concerned until he thought it was getting worse and spreading. The outbreak is localized to his jaw area and upper neck. He does not shave every day and the outbreak seems to get worse when his beard grows back. On physical examination, you note papules and pustules that are located to the side of the hair follicles. Question What is your initial diagnosis?

Correct answer: Pseudofolliculitis Explanation The clinical picture is suggestive of pseudofolliculitis. Pseudofolliculitis is caused by ingrown hairs, particularly in men with curly beard hair. As the hairs grow back, the sharp tips of the shaved hairs penetrate the skin and cause inflammation. Papules and pustules are usually located to the side of the hair follicle, but not in the hair follicle. Pseudomonas folliculitis, which is also known as "hot tub" folliculitis, usually occurs on the trunk after immersion in spa water. Nonbacterial folliculitis is usually caused by oils that irritate the hair follicle. It is common in certain workers such as machinists. It can also be caused by the application of certain cosmetic oils (e.g., cocoa butter). Acne vulgaris is an inflammation of the pilosebaceous units of certain body areas; it often manifests itself during puberty (age of onset is usually 14-19 years old in boys). The lesions appear as closed or open comedones and papulopustules, or nodules and cysts. Demodicidosis is parasitic infection caused by the Demodex mite (Demodex folliculorum) that normally lives harmlessly in hair follicles, especially on the facial area. Lesions appear as papules and pustules with a background of erythema on the face.

Case A 53-year-old Caucasian man presents due to a bleeding mole on his face. The mole is located on his left cheek; it has been present for the past several years, but in the last 3 months, it has started to spontaneously bleed. The patient denies any other moles with the same characteristics, and he just wants it taken care of, so it is not as bothersome. The patient denies weight loss, night sweats, or fevers; he has no recent changes in his appetite or sleeping issues. He is a farmer, and he owns over 100 acres that he plants and harvests yearly; he has done so for the 25 years. On physical examination, you find a 4 cm macule that has irregular borders; there are at least 3 different shades of color to it, and there is a small ulcer in the middle of it. Question Considering the most likely diagnosis for this patient, what diagnostic study is most crucial to confirm this pathology?

Correct answer: Punch biopsy Explanation The correct response is punch biopsy. Melanoma is the most malignant tumor of the skin. Melanoma evolves from the malignant transformation of melanocytes, typically located at the dermal-epidermal junction. Melanoma has many categories of characteristics but is typically described as a flat or raised pigmented lesion. The "ABCDE Rule" created specifically for evaluating lesions for melanoma is what is used to help screen suspicious lesions: Asymmetry, Border irregularity, Color variegation, Diameter > 6 mm (at its longest axis) and now including the Elevation, or Evolution of the suspicious lesion. Not only do healthcare providers use this, it is being explained and described to all patients to help them detect questionable lesions that need further investigation. The lesion in question described above meets almost all of these criteria of the ABCDE rule. Any lesion that is suspicious to be malignant must be biopsied. Melanoma initially pathologically has a radial growth phase and then will progress to a vertical growth phase. For this reason, any lesion that one highly suspects of being malignant and is possibly melanoma must be biopsied using either a punch or excisional biopsy. A shave biopsy is contraindicated in a suspected case of melanoma due to the reason listed above; it will only give a partial specimen and therefore is not appropriate to determine diagnosis, prognosis and/or method of treatment. Cryotherapy is in fact a treatment regimen for more benign lesions, such as warts. Drawing a CA-125 level is not a regimen used in the case of malignant melanoma but very commonly is elevated in patients with ovarian cancer. Observation only would be inappropriate and blatantly harmful practice of medicine due to the fact of all the positive components met with the ABCDE rule.

Case A 55-year-old man states that he recently noticed that he has started to flush easily and his nose appears unusually large. Additionally, during an appointment with his eye specialist the month before, he was prescribed medication for blepharitis and keratitis. Question What is the most likely diagnosis?

Correct answer: Rosacea Explanation Rosacea is usually seen in men over the age of 40; there is erythema, telangiectasia, and hyperplasia of the tissues of the face and nose. The face has a rosy hue, and there may be associated migraine and eye problems. Perioral dermatitis and steroid rosacea are usually seen in younger women, mostly around the mouth area; they are not seen as flushing involving the nose and cheeks. There is no history of local steroid use. Pityriasis rosea is a pink and scaly dermatitis believed to be a reaction to human herpesvirus (HHV) infection primarily seen in young adults. This condition does not match the described dermatitis in the vignette. In the case of an after-shave allergy, there will be erythema, edema, and pruritus, along with use of the suspected agent. A patch test will be positive.

Case A 16-year-old boy presents with a general complaint of itching of his hands and wrists. He says it started a few weeks after he went to play in a high school soccer tournament down south. On physical exam, you note numerous wavy, skin-colored ridges on his hands and wrists with excoriations. You also note the intertriginous areas of the hands are involved. Question What is the most likely condition this patient is demonstrating?

Correct answer: Scabies Explanation The clinical picture is suggestive of scabies. Patients infected with scabies often present with generalized to severe pruritis of the infected area. The incubation period is from 2-6 weeks, but symptoms can occur earlier. The primary lesion with scabies is a burrow, often described as having gray or skin-colored ridges. These ridges can appear wavy or linear, often with a minute vesicle of papule at the end of the burrow. Ringworm is a generalized term used to describe numerous fungal infections on the skin. The description would be based on what area of the body that is affected. Tinea corporis is a type of fungal infection that infects the trunk, legs, arms, and/or neck, excluding the feet, hands, and groin. The skin lesions can have multiple descriptions: small to large scaling, sharply marginated plaques with or without pustules or vesicles, usually at margins peripheral enlargement and central clearing, producing annular configuration with concentric rings or arcuate lesions; fusion of lesions produces gyrate patterns single and, occasionally, scattered multiple lesions. These lesions are not seen in this patient. With a spider bite, the description would be based on the type of spider that bit the patient. Spiders rarely bite more than once and are not described as wavy, skin colored lesions. Body lice are usually associated with poor socioeconomic conditions. Body lice live in the seams of clothes and can affect the entire body. The lice grabs on to the body hairs to feed. Lesions can be described as eczema, excoriations, lichen simplex, and secondary impetiginization.

Case A 45-year-old Caucasian woman presents because she is worried she may have skin cancer. While interviewing the patient you note the patient has an extremely fair complexion. When you examine the lesions that the patient is concerned about, you note the following description in your documentation: "5 papules that vary in diameter from 0.3-0.6 centimeters dispersed on the skin around the sternal angle. Lesions vary in color from flesh-toned to slightly hyperpigmented, and when palpated have a sandpaper texture." Question Based on the most likely diagnosis, what would be the most appropriate pharmaceutical regimen at this time?

Correct answer: 5-fluorouracil cream Explanation Actinic keratosis is the diagnosis for the lesions being described in this case scenario. These are small (0.2-0.6 cm) macules or papules that can possibly be flesh-colored, pink, or hyper-pigmented in color. They possess a distinguishable sandpaper texture and potentially are tender to palpation when a finger is drawn over them. Commonly these lesions will occur in sun-exposed areas of patients who have fair complexions. Actinic keratosis should indeed be viewed as premalignant lesions; approximately 1 out of 1000 of these lesions will progress to squamous cell carcinoma. Topical treatment for AK usually will be with either 5-fluorouracil (5-FU), imiquimod cream, or diclofenac gel, with 5-FU cream being the mainstay of topical treatment for the past several decades. Benzoyl peroxide and clindamycin topical gel are both utilized most commonly in acne vulgaris patients. Hydroquinone cream is utilized as a type of skin-bleaching agent to help lighten freckles, melasma, or trauma. Ketoconazole is a treatment for tinea or seborrhea infections.

Case A 55-year-old woman presents with the skin of her armpits and groin 'getting darker and darker'. Physical examination demonstrates velvety-brown and warty skin in the axilla and groin. Biopsy of these lesions shows a variable hyperplastic epidermis with many sharp peaks and valleys. Question Aside from cosmetic considerations, what primary medical significance do these lesions signify?

Correct answer: A sign of visceral carcinoma Explanation The correct response is a sign of visceral carcinoma. The lesions are acanthosis nigricans; they resemble a mole or wart, but are actually due to epidermal hyperplasia. Acanthosis nigricans can be seen in obesity, diabetes, and in patients with underlying cancers (often adenocarcinomas of the chest or abdomen). The lesions are not characteristic of immunosuppression and are not easily superinfected. They are also not malignant or premalignant.

Case A 22-year-old woman is evaluated for a chronic pruritic lesion. She reveals that it began on her posterior scalp, but it has now spread to her elbows, knees, and gluteal cleft. The lesions appear to wax and wane, but as of late, they have been spreading further. A picture of her lesions is shown. She is extremely distressed by her condition; she is particularly worried because her mother succumbed to squamous cell carcinoma. Question A biopsy of the lesion is likely to demonstrate what histological finding?

Correct answer: Acanthosis with intracorneal pustule formation and lymphocytic infiltration Explanation Acanthosis with intracorneal pustule formation and lymphocytic infiltration is the correct answer. This is characteristic of psoriasis, which is what the vignette describes. Psoriasis results in symmetrical itchy scaling lesions, which predominate over extensor surfaces, the scalp, and folds of skin. Microscopically, the hallmark is acanthosis (thickening of epidermis), parakeratosis (cell nuclei within stratum corneum), and dermal/epidermal infiltration by lymphocytes (Munro's microabscesses) Basal cell liquefaction with saw tooth appearance of rete pegs is an incorrect response. This is characteristic of lichen planus. Basal cell dysplasia with palisading invasion of dermis is an incorrect response. This describes basal cell carcinoma. Large intraepidermal cleft in suprabasal region is an incorrect response. This suggests Pemphigus vulgaris. Granulomas, lymphocytes, and Langerhans' cells surrounding dermal neurovascular structures is an incorrect response. This is characteristic of Tuberculoid leprosy.

Case A woman presents with her 17-year-old son due to concerns about his face. She states that he seems to break out in a rash on his face and it has been getting worse over the last 3 years. On physical examination, you note multiple closed comedones and papulopustules covering his cheeks, forehead, and chin. Question What is the most likely diagnosis?

Correct answer: Acne vulgaris Explanation The clinical picture is suggestive of acne vulgaris. Acne is an inflammation of the pilosebaceous units of certain body areas and often manifests itself during puberty (age of onset is usually 14-19 in male patients). The lesions appear as closed or open comedones as well as papulopustules or nodules and cysts. Herpes simplex type 1 appears as grouped small vesicles on an erythematous base. Superficial lesions rupture early with erosion formation. Over time, the lesions will crust and heal. Secondary syphilis lesions usually appear as target-like lesions; they appear primarily on the palms of the hands and soles of the feet. Perioral dermatitis appears as 1-2 mm erythematous papulopustules on an erythematous background and may have tiny scales predominantly around the mouth. Steven-Johnson syndrome is a mucocutaneous drug-induced or idiopathic skin reaction. Patterns are characterized by skin tenderness and erythema of skin and mucosa, followed by extensive cutaneous and mucosal epidermal necrosis and sloughing.

Case An 65-year-old man is being treated at the dermatologist for a lesion on his face for the past 5 weeks that will not go away. Additionally, the patient has a history of staying out in the sun, as he lives in Florida. Upon physical exam, a 0.6 cm macule that appears flesh-colored and slightly hyperpigmented is present on his right cheek. Upon palpation, the macule feels like sandpaper and is tender to the patient. All labs are within normal limits. Question What is the most likely diagnosis?

Correct answer: Actinic keratoses Explanation The clinical picture is suggestive of actinic keratoses. The patient presents with a flesh-colored hyperpigmented macule that is present on a sun-exposed part of the body. Additionally, the sandpaper appearance is a common description of actinic keratoses. Bowen Disease lesions are not described as sandpaper appearance. Lesions present as red-colored plaques with an irregular border. Furthermore, it is an early skin cancer that is diagnosed, and this patient is older than when it would normally be diagnosed. Exfoliative dermatitis is described as a peeling or shedding of the skin due to a medical condition. It is not described as a sandpaper appearance or lesion, and therefore the incorrect answer. Intertrigo is not the correct answer because it is a rash that can be red or itchy; it is not a lesion that can become hyperpigemented as described in the stem of the problem. Furthermore, it is usually found in skin folds. Mycosis fungoides is a T cell lymphoma and can appear as a rash and skin lesions. It is not described as a hyperpigmented lesion that appears sandpaper-like, and is therefore an incorrect answer.

Case A 55-year-old man presents with lesions on the top of his head. He noticed them about 6 months ago and did not think much about them until more of them appeared. Physical examination of the scalp shows that he is slightly balding and has scattered multiple lesions that appear to be <1 cm in size. The lesions are yellow-brown, dry, and scaly. Upon palpation, the lesions have a rough, coarse texture and are tender. Question What is your initial diagnosis?

Correct answer: Actinic keratosis Explanation The clinical picture is suggestive of actinic keratosis, which consists of single or multiple, discrete, dry, rough, adherent, scaly lesions that occur on the habitually sun-exposed skin of adults. They have a rough sandpaper-like texture to them, and they are usually described as "better felt than seen." Squamous cell carcinoma appears as a sharply demarcated, scaling, or hyperkeratotic macule, papule, or plaque. Solitary or multiple lesions are pink or red in color; they have slightly scaling surfaces, small erosions, and can be crusted. Solar lentigo lesions are strictly macular and 1-3 cm in diameter, but they can be as large as 5 cm. They appear as light yellow, light brown, or dark brown; they are uniformly mixed in color. Seborrheic keratosis consists of benign plaques that are 3-20 mm in diameter; their color can range from beige to brown or even black. They feel greasy and have a "stuck on" appearance. Onset is rare before age 30. Dermatofibromas are benign skin growths; they are typically found on the legs and range in size from about 0.5-2 cm. They can be described as hard papules that may appear in a variety of colors, but they are usually brownish to tan.

Case A 22-year-old man presents with a 1-day history of painful blisters at the right angle of his mouth. He has had tingling and burning sensations at the site for 3-4 days. He has no significant past history. On examination, his temperature is 99.5°F, but other vitals are normal. Multiple small vesicles are seen at the right angle of the mouth. There are no such lesions elsewhere. Question What is the treatment of choice for this condition?

Correct answer: Acyclovir Explanation From the history and examination, it appears that the client has a herpes simplex infection (herpes labialis). The treatment of choice for this condition is acyclovir. It is usually caused by herpes simplex virus (HSV) 1. Other lesions caused by this virus include pharyngotonsillitis and genital herpes. The diagnosis can be made clinically, but a Tzanck smear may be used to demonstrate the cell changes brought about by the virus. Treatment is with an antiviral drug such as acyclovir, which is a nucleoside analog with a high affinity for virus-affected cells. A dose of 200 mg is given 5 times daily for a minimum period of 1 week. Other drugs that can be used are valacyclovir, penciclovir, and famciclovir. Ribavirin is a synthetic nucleoside analog that inhibits a wide range of RNA and DNA viruses, but its in vivo effectiveness is sufficiently less than its in vitro effectiveness. It has been used to treat hepatitis C and respiratory syncytial virus. Interferons are proteins that are secreted in response to foreign material (bacteria, viruses, parasites, etc). Today, they are manufactured by the recombinant DNA technique. Zidovudine is an antiretroviral drug that inhibits nucleoside reverse transcriptase and causes chain termination. It is used to treat HIV. Amantadine is an antiviral drug used against influenza A. It has also been found to be beneficial in Parkinson's disease.

Case An 11-year-old girl with hair loss, presents to the clinic with her mom. The girl noticed her hair thinning and a bald spot on her scalp. She reports having atopy, denies feeling recently stressed, and does not report any underlying disorders. Upon physical exam, her hair does appear thin. The bald spot is smooth and round. Around its border there are short, broken off hairs which appear narrower closer to the scalp. Her mom states that she also had an issue with hair thinning when she was her daughter's age. All laboratory finding are within normal limits. Question What is the most likely diagnosis?

Correct answer: Alopecia areata Explanation The clinical picture is suggestive of alopecia areata because the patient presented with hair loss and balding in the absence of other causes. Additionally, there appears to be a family history, which is found in alopecia areata. It has also been found that atopy is associated with alopecia areata. Anagen effluvium is not correct because the patient did not undergo any cancer treatment or and was not exposed to toxic chemicals, which is associated with anagen effluvium. Hypothyroidism is not correct because in the stem of the problem it states that all laboratory findings are normal. In hypothyroidism, T3 and T4 would be low, which is not present in the patient. Telogen effluvium is not correct because the patient would present describing stress or a stressful event, which is associated with the stem of this problem. Traction alopecia is not correct because traction alopecia is associated with localized trauma to the hair follicles from tight hairstyles, which is not described in the history.

Case You have been asked to do a house-call on an 88-year-old woman who is bed bound and lives at home with her private home health aide. She has had no medical follow up for the past year. Approximately three weeks ago, she appeared to the aide to be having "headaches." Two weeks ago, she developed a rash on her left forehead, that over time developed into "little blisters that popped and crusted over." She has a history of coronary artery disease and was diagnosed with 'senile dementia' six years ago. As per the home health aide, she is occasionally combative, and resistant to care. On exam, she is awake, mumbles several words, but is not responsive to verbal commands. BP is 118/68 mm Hg, P 84/min R 20/min. Skin exam is as shown (see image), and there are no other significant lesions noted on the body. Question Of the options below, which is the most appropriate treatment at this time?

Correct answer: Aluminum acetate solution (Burow's) Explanation Herpes zoster, also known as 'shingles', is a skin eruption caused by reactivation of latent varicella virus. When primary varicella eruption, which usually occurs in childhood (as chicken-pox), resolves, the virus may remain dormant in the dorsal root ganglia. Immunosuppressed persons are at higher risk for developing zoster, but it often occurs in otherwise healthy people, as well. The incidence of herpes zoster increases markedly with age. There are 1-2 cases per thousand in teenagers and young adults, which doubles between 50 and 60, and again, between 80 and 90 years of age. The rash usually involves one dermatome, and pain may occur before a rash erupts. It begins as a macular erythematous rash, that progresses to clusters of vesicles over the next 12-24 hours, and new vesicles can appear over the next two days. Vesicle fluid often becomes purulent looking, then dries out. Crust forms by 10 -12 days, and falls off in 2-3 weeks. Oral or IV treatment with anti-virals, such as Acyclovir, are most useful when started within 72 hours of rash eruption. Pain should be adequately relieved with analgesics. Prednisolone, along with Acyclovir, have been shown to reduce pain in H zoster. But it has unfavorable risk-benefit ratio. For mild-moderate pain, NSAIDs may be sufficient, but in severe pain a narcotic is needed. Intrathecal methylprednisolone can be used in patients with persistent pain. (Ref: 4 and 5) Local treatment of zoster rash should consist of cleaning with soap and water. Application of an astringent, like aluminum acetate solution (Burow's or Domeboro's ) can be helpful in relieving local symptoms of pain and itching. Although Acyclovir, orally or IV, is useful in decreasing acute pain and time for healing, Acyclovir ointment is not useful in treating herpes zoster. Indications for Acyclovir ointment include herpes labialis in immune compromised patients, and limited use in genital herpes. Permethrim cream is used in treating scabies, caused by infestation by the Scarcoptes scabiei mite. Burrows, linear ridges on the skin cause by the female laying eggs in the stratum corneum, is the typical lesion. Scabies can cause vesicles, nodules and excoriation due to hypersensitivity to the mites. However, lesions are usually in inter-digital web spaces, axilla, wrists, genitalia and umbilicus, and not confined to a dermatome. Capsaicin cream relieves pain by depleting substance P in nerve endings. It can be useful in treating post herpetic neuralgia which can occur, usually in elderly patients, after herpes zoster infection. Use is limited by local reactions of stinging and burning and the need for frequent application. Capsaicin and other topical treatments for post herpetic neuralgia should not be applied until all skin lesions are healed and the skin completely closed. Metronidazole as a topical treatment is useful in acne rosacea, an idiopathic inflammatory disease of the face, usually seen in middle aged and elderly fair skinned persons. Erythema, papules and pustules can be seen in rosacea, but the rash is usually in the center of the face and not confined to a dermatome. Telangiectasias and facial flushing are common. Other uses for topical metronidazole include bacterial vaginosis and infected decubitus ulcers.

Case An 8-year-old girl presents with an itchy rash. The mother says she has observed the child itch off and on over the last 4 months. "She'd be fine if I could just get her to stop itching," the mother says. The child is on no medications; she is well positioned on the growth curve, and she is otherwise healthy except for occasional bouts of exercise-induced asthma. Physical exam reveals generally dry skin and areas of mild erythema and excoriation, largely confined to the antecubital and popliteal fossae. As the doctor prepares to step outside, the mother asks if the fact that she was treated for syphilis prior to her pregnancy with this child could have anything to do with this. Question What condition does the girl most likely have?

Correct answer: Atopic dermatitis Explanation Atopic dermatitis is the skin's reaction to allergy (food, asthma, animal dander, etc.). In children and adolescents, it is frequently localized to the flexural skin creases of the antecubital and popliteal fossae. Skin injury is more often a result of scratching than of the atopic process itself, and these individuals have a higher incidence of Staphylococcus aureus skin infections than do unaffected patients. Patients with atopic dermatitis are advised to avoid irritants and to keep skin moist (which includes avoiding hot showers and profuse scrubbing). Treatment often includes the judicious use of low-dose topical glucocorticoids and conservative administration of antihistamines to reduce the itching. Psoriasis is a very common chronic inflammatory condition of the skin. The lesions are variably pruritic and are characterized by sharply demarcated papules and rounded plaques. A silvery scale is frequently observed covering the erythematous plaques. Depending on the distribution and character of the lesions, psoriasis can be further categorized into several sub-groupings. The most common subtype (plaque type) of psoriasis is usually distributed on the elbows, knees, sacral area/gluteal cleft, and scalp, but there is also a less common variety known as inverse psoriasis. This is where the plaque lesions form in intertriginous regions in addition to scalp, palms, and soles. Because of the moisture in the intertriginous areas, scales may not be evident. Other forms of psoriasis include eruptive or guttate psoriasis as well as some variants where the lesions are more pustular in character. The etiology of psoriasis is not well defined, but there may be some association with certain medications, such as lithium and beta-blockers. Intertrigo is not one of the listed answer choices, but discussion here may be helpful. Stedman's Medical Dictionary (26th edition) defines intertrigo as: "irritant dermatitis occurring between folds or juxtaposed surfaces of the skin, as between the buttocks, between the scrotum and the thigh, beneath pendulous breasts, etc., caused by friction, sweat retention, moisture, warmth, and concomitant overgrowth of resident microorganisms, and occurring in young children and obese adults." As such, a patient with intertrigo could indeed have candidiasis, but a patient with candidiasis does not necessarily have intertrigo. Candidiasis occurs because of overgrowth of this group of yeasts in particular areas of the skin, those that are chronically wet, and especially in the intertriginous zones of the groin and beneath pendulous breasts. Patients with diabetes, chronic intertrigo, and cellular immune deficiency (as in HIV) are particularly susceptible. Obese patients have more intertriginous areas and are thus at risk. The rash is said to be macerated (displaying a waterlogged or soaked appearance characteristic of the dead surface skin). It is also described as erythematous, with marked inflammation, and it may be in a "satellite lesion" distribution, i.e. smaller lesions a few cm out from the larger one, which the "satellites" appear to orbit. The lesions may show scaling, and skin scrapings will show pseudohyphae and yeast forms. Lichen planus is a condition that produces primary lesions described as pruritic, polygonal, flat-topped violaceous papules. Lesions may show thin grey lines (Wickham's striae), and they have a tendency to occur on wrists and shins, but can occur anywhere on the skin. Mucous membranes including the buccal mucosa can be involved. The etiology is not completely understood, and the course is variable, but the lesions usually disappear spontaneously within several months to 2 years. Treatment may include topical glucocorticoids. The rash of secondary syphilis can indeed manifest as moist pink lesions in the intertriginous regions. This occurs in about 10% of patients with secondary syphilis and the lesions are called condylomata lata. Secondary syphilis is not the best answer, however. Syphilis is notorious for masquerading as other diseases, but there are certain clues to look for that are more typical of syphilis. The rash, which may be subtle, usually manifests as symmetric mucocutaneous lesions with some truncal distribution. Furthermore, nontender generalized lymphadenopathy is usually present, and the primary chancre of syphilis will still be present in about 15% of patients with secondary syphilis. Finally, RPR should be positive in a patient with secondary syphilis. Other factors to consider are the patient's obesity, diabetes, reports of vulval pruritus consistent with candidal vulvovaginitis and positive HIV status. In summary, secondary syphilis would be an appropriate consideration in this patient, but it would not be the best answer. Of parenthetical note, IV drug users have a high (upwards of 25%) false positive rate by reagin-type tests such as RPR. Here is a list of each of the possible answer choices together with a simplified association: Atopic dermatitis—allergic, flexural creases Candidiasis—diabetic, chronic intertrigo Lichen planus—pruritic polygons Psoriasis—silvery scaled papules and plaques Secondary syphilis—the masquerader

Case A 20-year-old African-American woman presents with very itchy rashes on her arms. She reports that they have been recurrent for as long as she can remember. She is an asthmatic and on ventolin (salbutamol) inhaler. On examination, you note erythematous maculopapular lesions with hyperpigmentation and lichenification in the antecubital fossae bilaterally. Question What is the most likely diagnosis?

Correct answer: Atopic dermatitis Explanation This patient most probably has atopic dermatitis. It is characterized by chronic pruritic lesions marked by exacerbations and remissions. There is a personal or family history of asthma, allergic rhinitis, food allergies, or eczema. In the acute phase, they may present with erythematous patches, weeping, and crusted plaques. In the chronic phase, they present with hyperpigmented lichenified lesions. It commonly affects the face, neck, wrists, antecubital, and popliteal fossae. Hand eczema is characterized by erythema, dryness, cracking, and edema of the skin of the hands. It is a chronic condition, which is aggravated by frequent immersion of hands into water and detergents. Seborrheic dermatitis is characterized by greasy scales over the scalp. It may also be visible along the hairline, on the eyebrows, in the external auditory canals, in the nasolabial folds, and over the sternum. There may be some pruritus. Nummular dermatitis is characterized by chronic coin-shaped, crusted lesions, which are usually pruritic. They commonly appear on the trunk and extensor surfaces of the extremities, especially the pretibial areas.

Question A 23-year-old African American man, with a past medical history positive for Addison's disease, now complains of white patches on his knees and elbows. There is no pruritus or discomfort. On questioning he reveals that the first patch appeared over the right elbow a few months ago, and has been progressive since then. An examination confirms the presence of flat, well-demarcated areas of depigmentation on his elbows, knees and skin folds. Most hairs within these patches are also white. Vitiligo observed in patients with Addison's disease is due to which one of the following?

Correct answer: Autoimmune destruction of melanocytes Explanation Refer to the chart. Vitiligo is an acquired condition, affecting 15 of all races in which circumscribed depigmented patches develop. Vitiligo observed in patients with Addison's disease is due to autoimmune destruction of melanocytes and is seen in 10 to 20% of these patients. It is not due to decreased gluconeogenesis, decreased blood volume, infection, or diarrhea. There is complete loss of melanocytes from affected patches. Generalized vitiligo is associated with a positive family history and with other autoimmune diseases such as diabetes, thyroid and adrenal disorders as well as pernicious anemia. Trauma and sunburn may precipitate the appearance of vitiligo. The patches are sharply demarcated and have normal sensation in them.

Case A 59-year-old fair-skinned Caucasian woman presents with a lesion on her eyelid that has been growing slowly for the past 6 months. She reports that she has been an avid gardener most of her life. On examination, you find a nontender nodule with a pearly border on her right lower eyelid. Question What is the most likely diagnosis?

Correct answer: Basal cell carcinoma Explanation Basal cell carcinomas (BCC) arise from epidermal basal layer cells. Patients can present with a shiny pearly nodule (noduloulcerative BCC), some of which have more melanin (pigmented BCC), or may present with erythematous, scaling plaques (superficial BCC), or with a solitary flat yellowish plaque (morpheaform BCC). BCC are slow-growing locally invasive tumors that rarely metastasize. They can develop on the chest, back, extremities, head, and neck. Treatment depends on the histology, size, and location of the tumor and includes surgical excision, curettage and electrodesiccation, cryosurgery, and Mohs surgery. There are several types of malignant melanomas, including: Superficial spreading melanomas—plaques with blue-black spots Nodular melanomas—rapidly growing dark papules Lentigo maligna melanomas—brown macules with scattered darker spots on sun-exposed areas, such as the face Acral lentiginous melanomas—most common form in African American patients; they develop on the palms, sole, subungual skin, and mucous membranes Patients with squamous cell carcinomas (SCC) may present with a red papule or crusted plaque on the lips, ears, or hands. There are several forms of Kaposi's sarcoma (KS), including: Classic KS—purple plaques or nodules on the lower limbs of elderly men of Mediterranean origin without AIDS AIDS-related form—widely disseminated purple papules or plaques on the skin, mucous membranes, and viscera Lymphadenopathic form—involves lymph nodes and skin In Paget's disease of the breast, patients present with a unilateral, sharply marginated, red, scaly rash affecting the nipple and areola. There may also be an underlying mammary duct carcinoma.

Case A 28-year-old man presents with a 2-week history of a non-painful, non-pruritic rash. He is negative for any other rashes, dysuria, urinary frequency, penile discharge, erectile dysfunction, diarrhea, constipation, change in stool, nausea, or vomiting. He does recall having had a penile "scab" approximately 4 weeks ago that healed; he never sought medical attention. He is not aware of having been exposed to anyone with any illnesses in the past few months. Social history is positive for unprotected anal sex with multiple male partners in the past 6 months, with the last sexual encounter occurring 4 days ago. He states that he has not engaged in any illicit drug use or cigarette smoking. Skin exam reveals a pink-red papulosquamous eruption with scattered discrete coppery papules on the palms of his hands. Question What treatment is the best choice for the patient's condition?

Correct answer: Benzathine penicillin G 2.4 million units IM once Explanation Based on the clinical scenario, the most likely diagnosis is secondary syphilis. Secondary syphilis without CNS or ocular involvement is treated similarly to primary syphilis, with the preferred treatment of 2.4 million units of benzathine penicillin G. There is limited data on the use of ceftriaxone, and treatment specifics have not been defined. Azithromycin may be used as an alternative treatment, but there has recently been increased resistance questioning its efficacy. Doxycycline is a good alternative for penicillin-allergic patients, but it must be given for 2 weeks. Ciprofloxacin is not a recommended treatment for primary or secondary syphilis.

Case A 7-year-old girl was playing at a local playground near her home. She was playing kickball and the ball went into the bushes and she retrieved it. After several minutes, she noticed a red lesion on her right forearm. She rushed home and told her mom. The mother looked at the lesion and noticed a small red area and sprayed the area with a local antiseptic that she had in her medicine cabin. The child only complained that it burned. A few hours later, the mother looked at the area and noticed that a white area appeared. She thought nothing of it and placed a band-aid over the bite. Before the child went to bed that night, she removed the band-aid and notice that the area had darkened. She then took the child to the ER for further evaluation. Question Which of the following insects inflicts this envenomation?

Correct answer: Brown recluse spider Explanation The clinical picture is suggestive of a brown recluse spider bite. The bite usually produces localized burning, followed by a white area in 3 to 4 hours due to vasoconstriction, which will progress to local tissue necrosis after a few hours to days. Black widow spider bites cause generalized muscular pain, muscle spasms, and rigidity, which are not seen in this patient. Stings caused by most scorpions in the US cause only local pain. Centipede bites typically produces pain, redness, swelling, and possibly numbness at the site, which is not seen in this patient. Dermacentor andersoni tick or dog tick bites cause Rocky Mountain Spotted fever (RMSF). Sign and symptoms include flu-like symptoms followed by chills, fever, headache, sore muscles, restlessness, and physical exhaustion. A red macular rash appears first on wrists and ankles. It then spreads centrally, which is not seen in this patient.

Case A 66-year-old woman presents with a rash that has developed over the last 3 weeks; it has been getting worse. She states that her legs began to itch and became red with blisters, she noted that the same thing was occurring on her abdomen about 1 week ago. On physical examination, you note urticarial, inflammatory plaques, as well as papules with blister formation. On her abdomen, you note inflammatory plaques surmounted by tense blisters. Question What is your initial diagnosis?

Correct answer: Bullous pemphigoid Explanation The clinical picture is suggestive of bullous pemphigoid. Bullous pemphigoid is an autoimmune disorder that affects the structural protein of the skin. Common occurrence is age 60-80, and it often appears on the lower legs. Initial symptoms include moderate to intense pruritus. The lesions are usually urticarial, papular, and erythematous; they evolve in weeks to months into large, tense bullae. Toxic epidermal necrolysis and Steven-Johnson syndrome are mucocutaneous drug-induced or idiopathic skin reactions. Initial sore throat, fever, and headache may be present in both conditions. Skin patterns are characterized by skin tenderness and erythema of skin and mucosa, followed by extensive cutaneous and mucosal epidermal necrosis and sloughing. Diagnosis is based on the percent of epidermal skin detachment. Lichen planus appears as pruritic, polygonal, purple papules. Angioedema is a localized, rapid swelling of the face, hands, or genitalia.

Case A 15-year-old Caucasian boy presents with a 2-week history of recurrent break outs on his face. His mother states that the condition is worse during football season. He currently uses over the counter products; there has not been any improvement. Examination of the face reveals a combination of inflammatory papules, pustules, and comedones on the face; there is a concentration on the forehead and temples. Question In addition to prescription medication, what instructions should be given to the patient to manage his acne?

Correct answer: Clean his football helmet after use Explanation Since the acne is worse during the football season and the concentration of acne is worse on the forehead and temples the best advice to give patient is to clean his helmet after every use. The bacteria mixed in with the sweat after playing football accumulates on the helmet, and gets in contact with the skin; this worsens inflammation, forming more papules and pustules. Aggressively squeezing bumps could worsen acne and cause scarring. Washing the face at least 3 times a day is excessive and unnecessary. Gentle cleaning of the face as needed is enough to remove excess oil and bacteria. The use of ointments should be avoided; it can occlude pores worsening acne. Avoiding the washing of the face can cause excessive accumulation of bacteria and oils on the face; they can worsen acne.

Case A 26-year-old man presents with a 2-week history of an itchy rash. He says the rash is getting worse with the summer months; after jogging, it gets extremely itchy. The rash is scaly, red, and a map-like lesion; it is extending from the groin to the inner thighs on both sides. Question What is the best initial treatment for this condition?

Correct answer: Clotrimazole Explanation This man has typical presentation of Tinea cruris, dermatophytoses (ringworm) of the groin area. Topical therapy is usually successful and the initial choice for treating dermatophytoses infections. The only topical antifungal here is clotrimazole; all of the others cannot be used topically. The other antifungal topicals, which can also be used, are tolnaftate, miconazole, and ketoconazole. For resistant cases or those with very widespread involvement, systemic therapy is indicated. Griseofulvin is a widely used systemic antifungal agent and is effective in treating tinea capitis, corporis, pedis, cruris, and unguium. Tinea unguium requires long-term treatment, and fluconazole and itraconazole are also used successfully. Tinea vesicular can be treated with ketoconazole; griseofulvin is not effective in treating tinea vesicular. Amphotericin B is a parenteral antifungal; it is used for systemic, subcutaneous, deep mycoses, and opportunistic infections in immunocompromised patients. Praziquantel is an antiparasitic drug.

Case A 23-year-old woman comes to the office for a gynecologic examination. This is her first visit, and she has no complaints. She tells you that she has not had a Pap smear in several years. Her menarche was at 12 years, and she has had regular cycles since then. She has had several sexual partners in the past, but has been with her current partner in a monogamous relationship for 1 year. She reports that she had a chlamydial infection that was treated several years ago, but she denies a history of other sexually transmitted diseases. She has never been pregnant. On physical examination, her cervix appears friable with a slight area of ulceration. There are several perineal and vaginal lesions, which appear as small cauliflower-like projections. The results of the Pap smear, which return in 1 week, show a low-grade squamous intraepithelial lesion (mild dysplasia, CIN I). Question What factor in this patient's case is most closely correlated with the abnormal finding on the Pap test?

Correct answer: Condyloma acuminata Explanation A Pap smear should be obtained routinely during the pelvic examination at the initial visit. The Pap smear as a screening test has been extremely successful for the prevention of cervical cancer. Detection of dysplasia allows for successful treatment and prevention of cancer. Risk factors for cervical cancer include: Early age of first coitus Multiple sexual partners Immune suppression Low socioeconomic status Lack of Pap smear screening This patient has more than 1 risk factor at this young age. Condyloma acuminata are soft, fleshy warts that are caused by the HPV. Cervical dysplasia and carcinoma in situ are likely caused by types 16, 18, 31, 33, and 34. It is sometimes difficult to differentiate from squamous cell carcinoma. It is possible to treat these warts with a topical application if they are small in size. If they are large warts, cryotherapy may be required.

Case A 40-year-old man presents with a 2-day history of itchy lesions on his face; the onset came after using a new aftershave. On examination, you find erythematous, sharply demarcated, weeping lesions on his cheeks and chin. Question What is the most likely diagnosis?

Correct answer: Contact dermatitis Explanation Contact dermatitis in the acute phase is characterized by sharply demarcated, weeping, edematous, eczematous plaques. There is usually a history of contact with an allergen. Management includes avoidance of irritants and the application of topical glucocorticoids. Asteatotic eczema is characterized by fine cracks that may be erythematous. It is usually found on the pretibial surfaces of elderly people during winter; it may also be pruritic. Management includes application of topical emollients and avoidance of irritants. Stasis dermatitis is characterized by erythematous, pruritic scales with oozing patches; it is usually found over the medial aspect of the ankles. Hyperpigmentation and stasis ulcers may also be present. It develops on the lower limbs due to chronic venous incompetence. Management includes using topical glucocorticoids and emollients, as well as avoiding irritants. Lichen simplex chronicus consists of a well-circumscribed, scaly plaque with lichenification and hyperpigmentation due to chronic scratching or rubbing (which can also occur during sleep). It commonly involves the occiput, back of the neck, arms, dorsum of the feet, and ankles. Lichen planus is characterized by pruritic, polygonal papules; they are flat-topped and violaceous in color and have a network of gray lines (Wickham's striae) on the surface. They commonly develop on the wrists, legs, and genitalia. Management includes using topical glucocorticoids, though many patients have spontaneous remissions.

Case A 5-year-old girl presents with a rash. The girl's mother states that, over the previous weekend, she took the child to an urgent care center; the patient was diagnosed with a urinary tract infection. The child was started on a 7-day course of sulfamethoxazole/trimethoprim and currently only has one more dose to take. The mother states the child has never taken this type of medication before. The rash in question was first noticeable 2 days ago; it has spread, worsened, and intensified, prompting the mother to bring the child in to see you today. She denies any recent fevers, irritability, itching, or other significant symptoms. On physical examination, you note scattered lesions on the child. Each lesion appears to have 3 concentric circles of color change. Question Ico-delete Highlights Making note of the characteristic target lesions this patient is displaying, what treatment would you recommend at this time?

Correct answer: Discontinue sulfamethoxazole/trimethoprim and begin symptomatic treatment Explanation This patient scenario is describing a classic example of uncomplicated erythema multiforme (EM). EM typically begins as papules that later develop a dark center and then evolve into lesions with central bluish discoloration and/or blisters. They then will have the characteristic target lesions (sometimes referred to as iris lesions) which appear with 3 concentric circles of color change. Causes may include herpes simplex virus, various drugs (most commonly sulfonamides), and Mycoplasma infections. The patient is otherwise asymptomatic at this time. For these reasons, discontinuation of the sulfamethoxazole/trimethoprim as well as supportive symptomatic care is the appropriate treatment. Because there is no evidence of a recent herpes simplex infection, an antiviral such as acyclovir would not be an appropriate choice. Oral histamines are typically used to help relieve pruritic symptoms, if needed. This patient has not displayed any significant problems. Oral steroids have not been demonstrated to be effective. Because this is not an infectious bacterial process, azithromycin would not be the acceptable choice.

Case You see a 35-year-old African-American man for a painless tumor on his left hand. He also complains about malaise, fever, headache, and nausea. He works at a textile mill, which processes imported goat hair. He reports having cut himself about a week earlier at work. The lesion is a red-brown papule with peripheral edema, vesiculation, and induration. It shows an ulcer and formation of a black eschar in the center. Question Ico-delete Highlights What is the most likely diagnosis?

Correct answer: Cutaneous anthrax Explanation The symptoms described above are highly suspicious of cutaneus anthrax, caused by bacillus anthracis. The gram-positive, encapsulated, facultatively anaerobic rod is transmitted to humans by contact with infected animals and their products. It is also known as woolsorter's disease. The incubation period varies from 12 hours to 5 days. Molluscum contagiosum is a poxvirus infection, characterized by skin-colored papules with a central depression and is highly contagious. Furuncles (perifollicular inflammations) caused by staphylococci, are most painful when they occur in skin that is closely attached to underlying structures. Melanoma, a malignant tumor of the skin can vary in size, shape and color but usually does not grow fast on the surface. Impetigo is a contagious and autoinoculable infection of the skin caused by staphylococci or streptococci. The itching lesions can consist of macules, vesicles, pustules, and honey-colored gummy crusts.

Case A 35-year-old woman notices a change in the appearance of a mole on her neck. Physical examination reveals that the lesion is an irregular nodular superficial mass with a variegated appearance. Biopsy demonstrates a primary malignant tumor. Question What factor is most predictive of the patient's long-term prognosis?

Correct answer: Depth of lesion Explanation The lesion is a malignant melanoma. Melanomas can develop either de novo or in an existing mole. Sunlight exposure is a significant risk factor, and fair-skinned individuals are at increased risk of developing melanoma. The most significant factor for long-term prognosis is the depth of the lesion since the superficial dermis lies about 1 mm under the skin surface; penetration to this depth is associated with a much higher incidence of metastasis than is seen with a more superficial location. The circumference of the lesion is much less important than the depth since one form of melanoma (superficial spreading) can still have a good prognosis—despite large size—if it has not extended to the depth of the superficial dermal lymphatic bed. The darkness or degree of variation in color does not have prognostic significance once melanoma is diagnosed. Irregularity (or fuzziness) at the border of a mole-like lesion is a good clue to potential malignancy, but it does not affect prognosis once a melanoma is diagnosed.

Case A 65-year-old woman initiates carbamazepine for a head trauma-related seizure disorder. A few weeks later, she develops a facial rash and arthritis. She has no new paresthesias or other neurological symptoms, and she has not had any additional seizures. No ulcers, pleuritis, chest pain, bruising, hematuria, or gastrointestinal symptoms are noted. On physical examination, her temperature is 98.6°F; her blood pressure is 120/80 mm Hg, and her pulse is 80 beats per minute. She has a malar rash, and the remainder of her examination is normal. Labs are as follows: Anti-double-stranded antibodies Negative Antihistone antibodies Positive Hemoglobin 13.5 g/dL Serum creatinine 0.7 mg/dL Urinalysis No protein, no blood, no cells, no glucose, no casts White blood cell count 5 x 103 cells/mm3 Question What is the most likely diagnosis?

Correct answer: Drug-induced lupus Explanation This patient most likely has drug-induced lupus. Drug-induced lupus follows a medication exposure; cutaneously, it may look like SLE. Unlike systemic lupus erythematosus, drug-induced lupus resolves on its own if the offending drug is discontinued. Additionally, the systemic involvement of the kidneys and nervous system does not usually occur in drug-induced lupus. If vasculitis-type symptoms do occur, a drug-induced variant may be distinguished from vasculitis by the presence of anti-myeloperoxidase anti-neutrophil cytoplasmic antibodies. A variety of drugs, such as procainamide, hydralazine, carbamazepine, methyldopa, and sulfasalazine, etc., may cause an immunologic reaction that causes a lupus-like picture. Patients present with fever, rash, myalgias, arthralgias, arthritis, and serositis. Anti-double-stranded DNA antibodies and hypocomplementemia are unusual, although anti-histone antibodies are seen in most patients with drug-induced lupus, and they may also be noted in SLE. SLE is a systemic inflammatory disease that causes a variety of signs and symptoms affecting many organs. Renal involvement is frequent in SLE and may include a variety of glomerular diseases causing hematuria, proteinuria, and urinary casts. Wegener's granulomatosis is a small vessel vasculitis of unknown pathogenesis; it causes inflammation and necrotizing granulomas in the upper and lower respiratory tract and kidneys; it also causes a rash. Palpable reddish-purple lesions on the lower extremities and positive anti-neutrophil cytoplasmic antibody (ANCA) should raise suspicion of vasculitis. Renal involvement is very common; necrotizing glomerulonephritis with negative immune staining may be seen on renal biopsy. Henoch-Schönlein purpura (HSP) is a vasculitic syndrome seen more often in children than adults; it may cause petechiae, purpura, gastrointestinal bleeding, and hematuria/glomerulonephritis. Complement counts and anti-double-stranded DNA are normal; elevated IgA antibody levels may be seen. Rheumatoid arthritis (RA) is inflammatory arthritis that causes joint stiffness and swelling that takes a long time to loosen up after prolonged sitting or sleeping. Symmetrically swollen joints and rheumatoid factor may be noted, but vasculitis is uncommon in RA (Harrison's chapter 314).

Case Ico-delete Highlights A 33-year-old woman presents with a 1-week history of lesions on her face and neck. She works as a cook and dishwasher, and she denies taking any medications. The lesions occurred with the presentation of a sore throat, diarrhea, and fever. On clinical exam, the lesions are well-rounded, red macules, with an iris appearance. A few lesions are also apparent on the buccal mucosa. Question What is the most likely diagnosis?

Correct answer: Erythema multiforme Explanation Erythema multiforme is characterized by macules, papules, vesicles, and bullae. It occurs secondary to a toxic influence. There may or may not be a prodromal period with a sore throat, diarrhea, and fever. The lesions are red and are seen on the sides of the neck, face, legs, genitalia, and mucosa membranes. A typical lesion is a "target" or "iris" lesion. The course is generally 3 - 4 weeks. Lichen simplex is rare in children; it presents with lesions that are circumscribed, scaling patches resulting from chronic scratching. The margins appear well defined. The most common occurrences are on the ankles, anterior tibia, and nuchal areas. Erythema ab igne, which is often called toasted skin syndrome, occurs secondary to exposure to heat from flames or heating appliances. The course is generally benign, but it may have a potential for malignant changes. The lesions are red, mottled skin with hypo- or hyperpigmentation. After many years of constant exposure, hyperkeratotic papules, plaques, and ulcers may occur. Contact dermatitis is a common inflammation due to external agents that can appear anywhere on the body. The lesions can be papules, blisters, or pustules and appear red, scaly, and dry. Treatment includes a detailed history to remove the causative agent, antihistamines, cold compresses, and or topical/systemic steroids. Psoriasis presents with a well-defined border that is bright red surrounding a silver white scaly lesion. Generally, psoriasis is limited to elbows, knees, and the scalp. Nail involvement is common. Auspitz sign's (pin point bleeding with the removal of the scales) is characteristic.

Case An adolescent girl who is established to your practice has come in to see you because of "blue areas on her skin." Over the past 5 days, the patient has noticed small red bumps that then formed areas with blue centers. Some even have blisters in the center. The patient states that she feels like she looks like a target. Examination of the patient reveals multiple lesions with central bluish discoloration as well as 3 concentric circles of color change in each lesion. While completing the physical examination, the patient states she was started on Bactrim by an urgent care physician 6 days ago to treat a urinary tract infection. Question What is the most likely diagnosis?

Correct answer: Erythema multiforme Explanation The scenario above is describing a patient with erythema multiforme (EM). This is a common, usually acute, and potentially recurrent inflammatory disease. Causes may include herpes simplex, upper respiratory tract infections, contact allergens, medications, and even connective tissue disorders to list a few etiologies. The patient's treatment course of Bactrim is highly likely to be the cause of her EM. Typically, the appearance of EM is numerous lesions that are described as target lesions; a confirmed diagnosis of EM should only be made if these lesions are seen. Target lesions, or "iris" lesions, originate from a centrifugal spread of the red maculopapular to a circumference of 1-3 cm in a span of 24-48 hours. The center can appear dark red, purpuric, or a dusky blue hue color. Lesions will appear in crops, and they typically resolve in a 1-2 week span without evidence of scarring, although it can take up to 1 month to completely resolve; reoccurrence is also possible. Erythema nodosum (EN) presents as pink-to-dusky-red firm nodules with indistinct edges. These nodules occur symmetrically, specifically on the pretibial surfaces; they may also be seen on the head, neck, torso, arms, and other areas. Both ankle edema and leg pain are also commonly seen with EN. Pityriasis rosea is a dermatologic disorder characterized initially by a herald patch on the trunk, followed by numerous smaller lesions of the same characteristics. A herald patch is described as an oval plaque that is usually 1-2 cm in diameter and has a thin collarette of residual scale inside its border. These lesions are salmon-colored in appearance. Psoriasis is usually described as red, sharply defined, scaling papules that coalesce to form stable round-to-oval plaques. It has a deep rich red color to it and the scale is adherent to this area, having a silvery-white appearance; when the scale is removed, it results in bleeding points (known as Auspitz sign). Areas of seborrheic dermatitis are moist, transparent (to almost yellow in color), greasy, and scaling. There are coalescing red patches and plaques.

Case A 16-year-old girl presents with ongoing issues with acne. She has had breakouts since she was 10 years old, and both she and her mother have noted the breakouts have been significantly worsening with each passing year. Although she does note a worsening of her breakouts around her menses each month, she states she has multiple lesions consistently throughout the month. She has a history of using diet modification, topical retinoids, benzoyl peroxide, and even 2 types of oral antibiotics without improvement. Physical examination of the patient today reveals extensive open and closed comedomes on the forehead, cheeks, and chin. Cystic, painful lesions are present throughout. Isotretinoin is now considered. Question How often will this patient need to have a serum pregnancy test once the treatment regimen has begun?

Correct answer: Every 4 weeks Explanation Our patient above is suffering from significant acne vulgaris, which could easily be classified as moderate/severe, along with cystic acne. The presence of comedomes as well as cysts requires a different approach to treat each of these conditions. No matter the classification, this patient has tried multiple entities in the more conservative means of treatment (diet, topical retinoids, benzoyl peroxide) along with somewhat more invasive means (oral antibiotics). Despite usage of these multiple means, she has failed to note any improvement in her symptoms. For this reason, the next treatment to consider would be isotretinoin. Isotretinoin is a vitamin A analog that is used for treatment of severe cystic acne that has not responded to regular conventional therapy. The use of isotretinoin should be offered to patients to decrease the incidence of severe scarring. This medication is known to be extremely teratogenic; therefore, pregnancy during the usage of isotretinoin is an absolute contraindication. Prior to beginning treatment, female patients are required to have 2 serum pregnancy tests. While on Isotretinoin therapy, female patients are required to obtain serum pregnancy tests every 4 weeks. If sexually active, female patients are also instructed to utilize 2 forms of contraception to prevent pregnancy. A stringent monitoring program is in place along with the initial consent form to ensure that these strict precautions are followed (iPledge). Weekly, every 3, 6, or 8 weeks are incorrect choices in this scenario.

Question A 46-year-old HIV-positive man presents with skin changes. You find a painful ulcer on his face and in his mouth and bluish-red and purple bumps on the skin of his thighs, arms, and hands. What is the cause of the disease in this patient?

Correct answer: HHV8 infection Explanation Your patient most probably has Kaposi's sarcoma. Kaposi sarcoma is a cancer of lymphatic endothelium. It forms vascular channels containing blood cells, thus giving the tumor its characteristic bruise-like appearance. It is the most common malignancy seen in HIV-infected patients. Patients develop Kaposi's sarcoma because of co-infection with HHV8. Sometimes it is the presenting symptom of AIDS; your patient most probably has AIDS and Kaposi's sarcoma. The drop in CD4 count in AIDS results in the development of opportunistic infections and neoplasms. In the United States, Kaposi sarcoma is an AIDS-defining illness in HIV-infected homosexual men. The pattern of opportunistic infections reflects the most common pathogens in that area (e.g., in developing countries, there will be tuberculosis, and in the United States, there will be Pneumocystis and Candida). HIV infection is not the cause of Kaposi sarcoma; rather, human herpesvirus 8 is. Worldwide, the predominant virus is HIV1, but Kaposi sarcoma will develop in both HIV1 and HIV 2 positive patients when immune defense is weak, as can be confirmed by CD4 count. HIV 2 type is concentrated in West Africa and is rarely found elsewhere. It is not the cause of Kaposi sarcoma. Kaposi sarcoma will develop when immune defense is damaged enough. Human herpesvirus 4 (HHV4), known also as Epstein-Barr virus (EBV), causes infectious mononucleosis and is associated with Hodgkin's lymphoma, Burkitt's lymphoma, nasopharyngeal carcinoma, and central nervous system lymphomas associated with HIV. EBV is also associated with a risk of certain autoimmune diseases (dermatomyositis, systemic lupus erythematosus, rheumatoid arthritis, Sjögren's syndrome, and multiple sclerosis). Although parvovirus B19 causes a rash called fifth disease, erythema infectiosum, or slapped cheek syndrome in children, parvovirus B19 infection causes chronic anemia, not skin lesions, in AIDS. It can also trigger an inflammatory reaction in AIDS patients who have just begun antiretroviral therapy.

Case A 4-year-old girl presents with her mother to discuss treatment of her atopic dermatitis. She was diagnosed as an infant, but her case appears to be getting worse despite frequent lubrication with thick emollient creams and medium potency topical corticosteroid use. Mother states that the patient is itching a lot more, especially during the night. Large, single patches of erythematous scaly excoriations, measuring about 3 cm by 4 cm, are present in the flexor surfaces of both the right and left elbows. Question What second line treatment is a reasonable choice to replace her current regimen now that the disease has become more severe?

Correct answer: High potency topical corticosteroids Explanation 'High potency topical corticosteroids' is the correct answer. Since a medium potency corticosteroid topical has been attempted, it would be most efficient to now try a step stronger in regards to topical steroids. 'Oral antihistamines' is not the correct answer. Oral antihistamines are given to patients with atopic dermatitis if they have pruritus that requires treatment, but do not actually treat the condition itself. Medications such as hydroxyzine can be given at bedtime as needed. 'Systemic anti-Staphylococcal antibiotics' is not the correct answer. It is possible for patients with atopic dermatitis to become superinfected with bacteria, commonly Staphylococcus aureus. In addition to treatment for the atopic dermatitis, patients can be placed on systemic antibiotics for 10-14 days. 'Ketoconazole cream' is not the correct answer. Ketoconazole cream is an antifungal cream used to treat various other skin disorders, but not atopic dermatitis. Seborrheic dermatitis typically has yeast on the skin surface and can be treated with ketoconazole. However, atopic dermatitis does not have a yeast or fungal component, and therefore is not treated with ketoconazole.

Case A 30-year-old woman and her 10-year-old son present with a vesicular outbreak on the skin (arms, legs, and face). There is no history of insect bites/infestation, exposure to chemicals, or use of medications. Vital signs are normal for both patients. The vesicles are described as itchy and red. The appearance of the condition seemed to coincide with a heat wave that's been impacting the area for the past 4 days. Vesicle material was obtained for Gram stain and culture. The Gram stain was positive for the presence of Gram-positive cocci in clusters and moderate white blood cells (refer to the image). Antibiotics were prescribed along with instructions to take care of the infected skin areas. The culture was significant for 4+ Gram-positive aerobic cocci that were beta-hemolytic on blood agar media, catalase-positive, and coagulase-positive. Question What is the most likely diagnosis?

Correct answer: Impetigo Explanation Impetigo is a vesicular superficial infection of the skin that later becomes crusted. The predominant etiological agents that cause impetigo are Streptococcus pyogenes and Staphylococcus aureus. Impetigo infections are most common during the summer when it is hot and humid. The infection is highly communicable. The infection is spread among family members and preschool children, especially in conditions of overcrowding and poor hygiene. Exposed areas are the most common sites of lesions. Pruritus is common, and scratching of the lesions can cause infections to spread. If material from a vesicle is obtained for Gram stain, Gram-positive cocci are usually seen. Initial vesicular lesions may appear like early varicella, tinea circinata, and Herpes simplex infections. Folliculitis is a pyoderma that is located within the hair follicles and apocrine regions. It can be caused by Staphylococcus aureus, Pseudomonas aeruginosa, Enterobacteriaceae, and Candida species. Staphylococcus aureus is the most common cause of folliculitis. Lesions are small (2-5 mm) and erythematous, with a central pustule topping the papules. Folliculitis due to Pseudomonas aeruginosa is usually acquired from swimming pools or whirlpools that are contaminated and are not adequately chlorinated. Pseudomonas aeruginosa produces pruritic papulo-urticarial lesions, which appear within 48 hours of exposure and go on to pustule formation. The sites of predilection are the hips, buttocks, and axillae, with a sparing of the soles and palms. When contracted in a whirlpool, the folliculitis is limited to the trunk below the upper chest or neck, depending on the water level.Carbuncle is a large and deep inflammatory nodule that is extensive and extends into the subcutaneous fat of areas covered by thick, inelastic skin. It is usually located at the nape of the neck, on the back, or in the thighs. The patient presents with fever and malaise. Occasionally, the patient will present acutely ill. Drainage occurs externally along the course of multiple hair follicles. In cases where the carbuncle produces a large amount of undrained pus, cellulitis, or bacteremia, there is a leukocytosis present. Osteomyelitis, endocarditis, or other metastatic foci can occur due to unpredictable blood stream invasion. Blood stream invasion can also be promoted by the manipulation of the lesions. Staphylococcus aureus is the most common cause. Furuncle is a deep inflammatory nodule that usually develops due to a preceding folliculitis. It is similar to a carbuncle, except that it is not as extensive. As with the carbuncle, Staphylococcus aureus is the most common cause. Furuncles appear as red nodules that are firm yet tender and become painful as the infection progresses. Furuncles will be found in areas of the body where there is friction, perspiration, and hair follicles. These areas include the neck, axillae, face, and buttocks. Predisposing factors include diabetes, obesity, neutrophil function defects, and treatment with corticosteroids. Ecthyma is similar to impetigo in that the lesions that appear are small vesicles on exposed areas of the body. The difference with ecthyma is that the lesions will penetrate through the epidermis. Streptococcus pyogenes is the causative agent. The lesions are most commonly found on the lower extremities (especially in the elderly and children). The lesions appear as "punched-out" ulcers that are covered with greenish-yellow crusts that extend deep in the dermis. They are surrounded by raised violaceous margins. Streptococcus pyogenes can be seen on Gram stains of lesions, and it is an aerobic Gram-positive cocci that will occur in short to long chains. It is beta-hemolytic on blood agar, catalase-negative, and has a Lancefield grouping of "A".

Case A 39-year-old woman presents with a rash. The rash started 5 days ago; it is pruritic and located primarily on her arms and legs, with a few sores in her mouth. Her husband described the rash as like a "bulls-eye". She has felt mildly "flu-like", but she denies fevers. She denies any changes in soaps, detergents, or diet. She has not been around anyone with a similar condition, and she denies travel. Otherwise, she reports feeling better than usual, with more improved control of her migraines since her neurologist started her on topiramate about 3 weeks ago. She has not needed to use her sumatriptan for over 1 month. Her review of systems is negative. She suffers from migraines, but she has no other chronic health conditions. Her current medications are topiramate daily, with sumatriptan as needed. She is allergic to amoxicillin. She has regular menses; she had a tubal ligation as contraceptive. On physical exam, a few small oral lesions are noted. The lesions on the extremities are primarily on the dorsal surfaces, with a ringed appearance, similar to a target. The remainder of her exam is normal, including vital signs. Question What is the most important and appropriate intervention in this case?

Correct answer: Immediate discontinuation of the new medication Explanation This patient is presenting with a target-lesion rash; it is typical of erythema multiforme, which is a relatively common type IV hypersensitivity reaction. In about 50% of cases of erythema multiforme, no cause is identified. Other causes are infection and medications, including anticonvulsants. In this patient's case, her rash occurrence suspiciously followed the initiation of a new medication. Erythema multiforme is an acute, self-limited condition; it can range in severity from a rash, mild malaise, and pruritus (erythema multiforme minor) to a much more severe condition, involving mucosal surfaces and possible desquamation with erythema multiforme major. The most important treatment of erythema multiforme, when a medication is attributed as the cause, is immediate discontinuation of the new medication. This patient's condition would not be expected to progress much more than her current state. The lesions often persist 1 - 2 weeks, then resolve. The decision to admit to the burn unit for skin management should be made for individuals with extensive and severe erythema multiforme major, Stephens-Johnson syndrome, and toxic epidermal necrolysis (TEN). High-dose oral antihistamines (or normal-dose antihistamines) can help with the pruritus associated with the erythema multiforme rash, and they are often used. However, until the offending causative medication is stopped, the antihistamine will have limited benefit. High-dose oral steroids would be indicated in cases of severe erythema multiforme. This patient could potentially be offered oral steroids if her discomfort and pruritus was quite severe. However, discontinuation of the topiramate remains the 1st step of successful resolution of the erythema multiforme. Topical steroid cream is another commonly used adjunct treatment for the erythema multiforme rash/pruritus. As with the other treatments above, it alone is not an appropriate treatment; topiramate must be discontinued.

Case A 28-year-old man presents on a cold winter night. He missed his last bus on his way back home and was out in the snow for a prolonged period without his winter gloves. He has numbness of his right middle and index finger. On examination of his right hand, the color of the affected fingers appears erythematous. You make a diagnosis of superficial frostbite. Question What is the initial step in the treatment of this condition?

Correct answer: Immersion of affected part in circulating water (99 - 104F) Explanation Treatment of frostbite is aimed at rewarming and preventing tissue ischemia caused by vascular thromboses. Initial treatment of superficial frostbite is immersion of the affected extremity in circulating water (99 - 104 F)1. Frozen tissue should be thawed rapidly and completely by immersion in warm water1. Rapid re-warming often produces initial hyperemia. A common error is premature termination of thawing, since reestablishment of circulation is intensely painful1. The use of intravenous narcotics is incorrect; they have no role in restoring blood flow to the affected extremities. In addition, they are not usually necessary to treat the pain associated with superficial frostbite1. Intravenous Ca2+ channel blockers, hyperbaric oxygen, and prostaglandin inhibitors are incorrect; there is no conclusive evidence that these agents promote tissue salvage1.

Case A 5-year-old boy has 3 honey-colored crusted lesions with surrounding erythema on his legs. The swabs taken from the lesions were sent to the microbiology laboratory. The results show yellow colonies grown on blood agar with hemolysis. The colonies are coagulase positive and mannitol positive. Question What is the most likely diagnosis?

Correct answer: Impetigo Explanation The case described here is impetigo. It is a superficial skin infection that is common in children. The arms, legs, and face are more susceptible to impetigo than unexposed areas. Lesions vary from pea-sized vesicopustules to large ringworm-like lesions. They can start as maculopapules and rapidly progress to vesicopustules or bullae to exudative and then honey-colored crusted lesions. The most frequent cause is Staphylococcus aureus, and the second is Streptococcus pyogenes. Staphylococcus aureus produces yellow-white colonies on blood agar with hemolysis and S. aureus is coagulase and mannitol positive, and these tests are used in the lab to differentiate S. aureus from other staphylococci. Streptococcus pyogenes is not coagulase and mannitol positive. Staphylococcus pyogenes produces white colonies, and it is coagulase and mannitol negative. Streptococcus viridans and Streptococcus agalactiae do not cause impetigo.

Case A 45-year old white woman complains of an oval growth on her forehead. She noticed the growth 2 weeks back, and it has gradually grown in size and now bleeds occasionally. She works as a secretary at a dentist's office. She often visits tanning parlors to get a "tanned" look. She has also had several cosmetic procedures done, including liposuctions in the past to treat her obesity. She gives a history of cocaine abuse in her 20s. She is a hypertensive on treatment with thiazides. On exam, there is an erythematous papule about 3x3 cm in size, which bleeds on touch. Her BMI is 30. Biopsy of the lesion reveals squamous cell carcinoma (SCC) of the skin. Question What was a risk factor for SCC in this woman?

Correct answer: Indoor tanning Explanation Indoor tanning is a risk factor for squamous cell carcinoma of the skin. Indoor tanning has been shown to increase the risk of malignant melanoma and SCC, but not basal cell carcinoma. The woman's indoor occupation as a secretary is not a risk factor for SCC, as there is minimal sun exposure. Increased UV radiation exposure from the sun, which occurs in outdoor workers such as farmers, etc., is a risk factor for SCC. Smoking, not cocaine abuse, is a risk factor for SCC skin. Obesity and cosmetic procedures are not known to be associated with SCC skin.

Case A 4-year-old boy presents with skin eruptions, fever, and diarrhea. Skin eruptions developed 1 week ago; they developed after exposure to multiple mosquito bites that left weepy, crusted areas. Over the past 2 days, the boy has become quiet, sleepy, and febrile; he has had a few loose stools. His past medical history is non-contributory, and his immunizations are up to date. On examination, you find a child in a mild distress; his temperature is 39 C. Heart rate is 100/min, and respirations are 22/min. On the skin of the arms and trunk, you notice multiple excoriations; there are a few fragile thin-roofed, flaccid, and transparent bullae, with a clear, yellow fluid that turns cloudy and dark yellow. Several bullae are ruptured, leaving behind rims of scales around erythematous moist bases, but no crusts. You also notice patches of skin of brown-lacquered appearance, with collarettes of scale and peripheral tube-like rims. Question For this condition, what is the primary mechanism of action of the antibiotic that represents the best treatment option?

Correct answer: Inhibition of peptidoglycans synthesis Explanation The correction response is inhibition of peptidoglycans synthesis. Your patient most probably has bullous impetigo. The diagnosis of impetigo is usually made on the basis of the history and physical examination. Patients with soft-tissue infection accompanied by signs and symptoms of systemic toxicity should be given an antibiotic that covers against both Staphylococcus aureus and Streptococcus pyogenes. Empirical treatment is semisynthetic penicillin, or first- or second-generation oral cephalosporins; these inhibit cell wall synthesis by inhibiting the synthesis of peptidoglycans, thereby inhibiting the synthesis of the bacterial cell wall. Antibiotics that bind to ribosomes in order to inhibit protein synthesis (aminoglycosides, tetracyclines, chloramphenicol, erythromycin, clindamycin, oxazolidinones, telithromycin etc.) are rarely the drugs of choice for either a particular infection or for a particular bacterial species. They are generally not recommended for use in children. Rifampin is an example of a bactericidal antibiotic that inhibits RNA transcription by inhibiting RNA polymerase. It is active against Staphylococcus and Mycobacterium. Rifampin monotherapy is not recommended because of the high likelihood for the development of resistance; instead, rifampin is added to other antibiotics for the treatment of streptococcal or staphylococcal infections (usually in combination with vancomycin) or Haemophilus influenzaetype b infection. Antibiotics that disrupt cytoplasmic membrane are used as topical agents (e.g., bacitracin). DNA synthesis Inhibitors include fluoroquinolones and Metronidazole. The use of fluoroquinolones to treat infections in children in most countries is approved only under certain circumstances. Metronidazole could be useful foranaerobic and certain parasitic infections.

Case A young woman presents with her daughter, who appears to be about 6 years old. You note that the child's eyes are downcast and she is scratching her scalp. On closer inspection, you note a tiny white wingless insect and tiny eggs attached to the hair shafts. The child's mother also notes that the same insects are present in the girl's eyelashes. Question What is the most likely diagnosis?

Correct answer: Lice Explanation The gold standard for diagnosing lice is finding a live louse on the head. Lice feed by biting the skin and sucking the blood of the individual on which they are living. The bites cause itching, so the infected person tends to scratch. Non-prescription medications can be used to treat the infestation. Scabies are tiny mites that burrow under the skin and lay eggs. Itching is the most common symptom of scabies. The itch of scabies is insidious and relentless. Treatment can include Elimite, which is considered the drug of choice. Thrush is a yeast infection that occurs in the mouth. Oral thrush causes creamy white lesions on the tongue or inner cheeks. Amphotericin B may be used to treat thrush. Ringworm is a fungal infection that grows on the outer layer of skin, hair, or nails. On the skin, ringworm appears as patches with clear centers and blister-like bumps. The treatment is an over-the-counter lotion. Dandruff occurs when the skin cells of the scalp flake off. The flake mixes with oil and dust to form dandruff. One can treat flaking and dryness with over-the-counter dandruff or medicated shampoos.

Case A 77-year-old man presents with a rash. The patient states that the rash began a few months ago as several small reddish-brown spots on his shoulders. Over time, the spots became bigger, with some of them growing together to form large well-defined patches. The surrounding skin is not affected. He denies any fever, and he states that he has never had this before. His past medical history is significant for hypertension and obesity. He is otherwise healthy, and he denies any other symptoms. On physical exam, he is a well-developed obese man in no acute distress. He has large areas of well-defined patches that have various shades of brown. Examination of skin scrapings under a microscope shows hyphae. Question This condition is usually caused by what organism?

Correct answer: Malassezia Explanation Tinea versicolor is a fungal infection common in adults and adolescents. The most commonly affected areas include the chest, back, and shoulders. Occasionally, it can be found on the face. It causes the affected skin to change color and become either lighter or darker. It was believed to be caused by a yeast called Malassezia furfur, but recent evidence points at Malassezia globosa as the cause. Tinea versicolor is not contagious and is very common. It can recur, so treatment may need to be repeated. The affected skin becomes reddish-brown to brown or may be light in color. Initially, the lesions are well-defined round-to-oval scaly macules. Over time, they tend to coalesce and form patches with various amounts of shading. The colors can be darker or lighter than the unaffected skin. Darker patches may disappear shortly after treatment is started, but lighter patches may take longer to go away. The skin discoloration is not permanent, and the color will eventually return to normal. Diagnosis can be confirmed by using an ultraviolet light (Wood light). The affected areas usually fluoresce and appear to be orange in color. If they do not fluoresce, the skin will appear darker than normal skin. A scraping of the skin will show the presence of hyphae in a characteristic "spaghetti and meatballs" appearance when exposed to potassium hydroxide. Diagnosis can also be confirmed by microscopic analysis. A scraping of the area placed in potassium hydroxide solution will show hyphae if a fungal infection is present. Tinea versicolor can be treated with several preparations that are applied to the skin. Over-the-counter preparations usually contain miconazole, ketoconazole, or clotrimazole; they can be found in shampoo or cream form. There are also prescription strength versions of these preparations. Oral medications (e.g., itraconazole or ketoconazole) also exist. Pseudomonas aeruginosa, Staphylococcus epidermidis, and Staphylococcus aureus do not show hyphae under the microscope. Trichophyton rubrum is a common fungus that causes ringworm, athlete's foot, and jock itch.

Case A 65-year-old man presents for a retirement physical. The physical examination is unremarkable except for a lesion on his upper back. It is a flat, blue-gray, pigmented lesion; it measures 1 x 2 cm and has irregular borders. The patient tells you that it has been there "for a while" and bleeds occasionally. Question What is the most likely diagnosis?

Correct answer: Malignant melanoma Explanation The clinical picture is suggestive of malignant melanoma. Clinical features of pigmented lesions suspected for melanoma are asymmetry, irregular border, colors that include pink, blue, gray, white, and black, as well as color variegation. The diameter is usually >6 mm. Bowen's disease is a form of squamous cell carcinoma. The lesions typically appear as small, about 1-3 cm in size, well-demarcated, pink to red in color, slightly raised, scaly plaques that may resemble psoriasis or actinic keratosis. Solar lentigo lesions are strictly macular and typically 1-3 cm in diameter; however, they can be as large as 5 cm. They appear as light yellow, light brown, or dark brown, and they are uniformly mixed in color. Tuberous xanthomas are flat-topped, yellow, firm nodules that are primarily located on the elbows and knees. Pilar cysts (trichilemmal cysts) are cutaneous cysts that are most often seen on the scalp of middle-aged individuals. They appear as firm, dome-shaped nodules that are typically 0.5-5 cm in size.

Case A 28-year-old woman has a history of obesity and diabetes. Upon physical exam, she has velvety hyperpigmented plaques over the back of her neck, groin, axilla, and breast area. The area has a dirty appearance with a rough texture. Question What treatment can be used to help treat this condition?

Correct answer: Metformin Explanation Metformin is the correct answer because, while there is no specific treatment for acanthosis nigricans, this skin condition is usually a sign of hyperinsulinemia and insulin resistance, which can be treated by weight loss and the use of metformin. Antibiotics treat bacterial infections, and acanthosis nigricans is not a bacteria. Topical steroids commonly treat eczema and dermatitis through anti-inflammatory properties that will not help acanthosis nigricans. Acanthosis nigricans is not a fungus. Antifungals help kill the fungal cells by allowing the fungal cells to leak out and the cell to die. Acanthosis nigricans would not improve with anticoagulants. An anticoagulant helps prolong clotting time and prevent blood coagulation.

Case A 3-year-old girl presents with a 2-week history of a pruritic, erythematous, excoriated rash. Her past medical history is unremarkable. Her only medications include a topical steroid ointment and oral diphenhydramine. She has no known allergies. She lives on a farm on the outskirts of town. She does not attend daycare and lives with 3 older siblings and her parents. There are cats and dogs in the house, which appear to be in good health. Her physical exam is significant for small, red papules in her interdigital spaces, wrist flexors, anterior axillary folds, and forearms. Scattered red-brown nodules are found in her axillary region as well. Question What is the most appropriate diagnostic study?

Correct answer: Microscopic examination of skin scrapings Explanation Scabies is caused by the burrowing and release of toxic substances by the adult female mite Sarcoptes scabiei var. hominis. Clinical manifestations include an intensely pruritic papular rash, characterized by thread-like burrows. The papules may be excoriated, scaly, or crusted. Infants with scabies may not manifest with the classic burrows but may present with pustules or bullae. Older children, adolescents, and adults usually present with papules and burrows in the interdigital areas, wrist flexors, axillary regions, buttocks, ankles, and groin. Palms and soles are generally spared. Occasionally nodules are found, most notably in the axilla, groin, and genitalia. The diagnosis can be made clinically but can be confirmed with an examination of skin scrapings, which can reveal mites or ova in the epidermal scrapings. The Wood lamp is a light that transmits ultraviolet light in a wavelength of 365 nm. It may be helpful in the diagnosis of certain fungal infections of the scalp. Infected hair shafts give off a blue-green fluorescence at their bases with ectothrix and some endothrix infections. The lesions of tinea corporis do not fluoresce, but macules of tinea versicolor will give off a golden fluorescence. Areas of altered pigmentation can be visualized more clearly when using a Wood lamp, such as those of tuberous sclerosis, incontinentia pigmenti, and atopic dermatitis. Skin biopsy is not usually indicated in the diagnosis of scabies, though mites might be inadvertently found in the biopsy sample. Culture and potassium hydroxide preparation of skin scrapings would aid in the diagnosis of candidal and tinea infections.

Case An 83-year-old woman is presently in a nursing home following a short hospitalization for a CVA. 2 weeks prior, she experienced a thromboembolic stroke, which resulted in right hemiparesis and dysphagia. Other medical problems include congestive heart failure, atrial fibrillation, osteoarthritis, and depression. The nursing staff contacts you to reports a sacral pressure ulcer measuring 3 X 2 cm. On physical examination, there appears to be interruption of the epidermis with an abrasion. The lesion is clean; there is no cellulitis. Question After wound cleansing, what is the most appropriate management step?

Correct answer: Moist dressing Explanation According to conventional pressure ulcer staging criteria (National Pressure Ulcer Advisory Panel or NPUAP), the patient in this case scenario has a Stage II pressure ulcers. Stage II pressure ulcers are characterized by partial thickness skin loss of the epidermis and/or dermis presenting as abrasion, blister, or shallow crater. Treatment of stage 2 pressure ulcer includes: Keeping the wound clean. Wash the lesion with saline solution to keep it clean. Use gauze dressings moistened with saline to cover the skin, keep it clean, and retain the wound's natural fluids. Dry dressings or bandages can slow the healing process or make the sore worse. Using prescribed ointments or creams, such as those that contain enzymes that may help speed the healing process. Maintenance of a nutritional diet with adequate protein, in order to promote healing and healthy skin. Removing dead skin or tissue (debridement). In evaluating and managing pressure wounds, it is important to note the difference between wound infection and contamination (colonization). The basic difference between the 2 conditions lies in the concentration of organisms in the wound. An infected wound contains a larger number of microorganisms than a contaminated wound. According to the Agency for Health Care Policy and Research (AHCPR), stage 2, 3, and 4 pressure ulcers should all be considered as colonized with bacteria. Because all stage 2, 3, and 4 ulcers are invariably colonized by bacteria, debridement and topical or systemic antibiotics are helpful. According to this patient's examination, there is no evidence of necrosis, excessive drainage, or infection at this time. Proper wound cleansing and debridement should prevent bacterial colonization from proceeding to the point of clinical infection. A contaminated wound will heal, but an infected wound will not. The use of antibiotics in the management of pressure ulcers is reserved for those pressure ulcer lesions complicated by infection. Infectious complications include bacteremia and sepsis, cellulitis, endocarditis, meningitis, osteomyelitis, septic arthritis, and sinus tracts or abscesses.

Case Ico-delete Highlights A 4-year-old boy presents with skin eruptions, fever, and diarrhea. Skin eruptions developed 1 week ago; he was in the Caribbean with his parents, and he was exposed to multiple mosquito bites. It was at the sites of mosquito bites that his parents initially noticed weepy, honey-colored crusted areas; the boy notes itching. Over the past 2 days, he has become quiet, sleepy, and febrile; he has had a few loose stools. His past medical history is non-contributory, and his immunizations are up to date. On examination, you find a child in a mild distress; his temperature is 39°C. Heart rate is 100/min, and respirations are 22/min. On the skin of the arms and trunk, you notice multiple excoriations, vesicles, and 'collarette' of scale surrounding the blister roof at the periphery of ruptured lesions; also seen are bullae, with erythematous bases between and adjacent to the lesions. Question What topical medication should you prescribe in addition to oral medications?

Correct answer: Mupirocin Explanation The correct response is mupirocin. Your patient most probably has bullous impetigo; a pathognomonic finding of this condition is a "collarette" of scale surrounding the blister roof at the periphery of ruptured lesions. It is caused by Staphylococcus aureus, producing exfoliative toxins that cause epidermo-dermal separation, which is macroscopically presented as vesicles. Vesicles rapidly enlarge and form the bullae. Itching, lymphadenopathy, fever, and diarrhea may accompany skin lesions. Antibiotic creams are the preferred topical treatment, and mupirocin is considered the most effective. Neosporin, fusidic acid, and chloramphenicol are other options. Terbinafine is an antifungal medication. It is useless in cases of bacterial infection. Benzoyl peroxide is used in the treatment of acne. Acyclovir ointment is used for the treatment of the symptoms of the first episode of genital herpes and limited non-life-threatening herpes simplex viral infections of the skin, lips, and genitals in patients with weakened immune systems Triamcinolone is a medium-strength corticosteroid used to treat the itching, redness, dryness, crusting, scaling, inflammation, and discomfort of various skin conditions. It is contraindicated in this case; it may help rapid spread of bacterial infection.

Case A 21-year-old woman presents with a 3-month history of a black mole on her right calf. She tells you that the lesion is enlarging and expanding. It began to itch about 3 weeks ago, and it has bled 2 times. She thinks that there may have been a mole near the same spot previously, but she is not certain. Her general health is good; there is no history of chronic illness, hospitalizations, or surgeries. She works as a professional model for a large advertisement agency. She does not take any prescription medication; she does not smoke tobacco, and she does not drink alcohol or use illicit drugs. Although she has dark-haired, she has a fair skin, and she says that she usually burns with even short sun exposure. Nevertheless, she does occassionally use a tanning booth prior to modeling events and vacations. There is no family history of skin cancer. The image depicts the lesion. VS stable, she looks anxious, but she is otherwise well. There is a dark brown-black nodule on the right calf; it measures 1 cm in diameter. On the surface of the nodule, there is a tiny area of crusting. There are no hairs. The nodule is asymmetrical, and its border is sharply demarcated; the color is uniform, and the elevation is regular. There is a narrow (1 - 2 mm) rim of erythema around most of the nodule. She has a sprinkling (about 25 - 30 in all) of melanocytic nevi on her trunk and legs. There is no significant local or distal lymphadenopathy. The liver is not palpable. The remainder of the physical examination is unremarkable. Question What is the most likely diagnosis in this case?

Correct answer: Nodular melanoma Explanation The sudden appearance of a black lesion on a Caucasian woman's leg, in addition to its rapid growth, itch, and bleeding, is highly suggestive of melanoma. This diagnosis makes sense considering her fair skin, history of sunbathing, and the fact she gets a sunburn easily. A diagnosis of malignant melanoma should be considered initially in the differential diagnosis until it is excluded by excisional biopsy. Multiple benign melanocytic nevi (as this patient has) are a risk factor, and so is her skin type. In this case, the major warning sign of malignant melanoma (as well as a clue to the melanoma type) is the history of sudden and undoubted change in a pigmented skin lesion. There are 4 basic types of melanoma; they differ in frequency as well as the location on the body where they manifest. 1) Nodular melanoma represents 15% to 30% of all melanomas and is the most likely type of melanoma in this case. It grows deeper more quickly than other types of melanoma, and is found most often on the trunk or head and neck. The melanoma usually appears as a blue-black, dome-shaped nodule. Nodular melanoma is more common in men than women. Nodular melanoma (NM) is the most aggressive form of melanoma. It tends to grow more rapidly in thickness (penetrate the skin) than in diameter. Instead of arising from a pre-existing mole, it may appear in a spot where a lesion did not previously exist. Since NM tends to grow in depth more quickly than it does in width, and because it can occur in a place that did not have a previous lesion, the prognosis is often worse because it takes longer for a person to be aware of the changes. NM is most often darkly pigmented; however, some NM lesions can be light brown, multicolored, or even colorless (non-pigmented). A light-colored or non-pigmented NM lesion may escape detection because the appearance is not alarming; however, an ulcerated and/or bleeding lesion is common. 2) Superficial spreading melanoma is the most common type of melanoma, representing about 70% of all cases. It spreads along the epidermis for a period of months to years before penetrating more deeply into the skin. The earliest sign of a new superficial spreading melanoma is darkening in 1 part of a pre-existing mole or the appearance of a new mole on unaffected normal skin. 3) Lentigo maligna melanoma arises from a pre-existing lentigo, rather than a mole. Lentigo maligna melanoma accounts for approximately 5% of all melanoma cases. This type of melanoma typically takes many years to develop. It occurs most often in older adults, and it is usually seen on the face and other chronically sun-exposed areas 4) Acral lentiginous melanoma accounts for less than 5% of all melanomas but is the most common melanoma in African-Americans and Asians; however, it may also occur in light-skinned (Caucasian) individuals. Acral comes from the Greek word akron, which means extremity, and the disease typically appears on the palms, soles, or under the nails. A Spitz nevus (previously called juvenile melanoma) is benign lesion and reasonable consideration; however, it usually presents as a pink or red nodule, and it is absent of the aggressive features describe in this case, making it less likely. It is important to note that 1 type of melanoma is not inherently more dangerous than another. All 4 types pose the same level of risk based on the following 5 factors: Tumor depth (Breslow's depth) Mitotic index The presence or absence of ulceration The number of regional lymph nodes containing melanoma The extent of spread in the regional lymph nodes

Case An 87-year-old woman in a nursing home has dementia, malnutrition, and incontinence and is bed bound. The visiting physician notes a deep pressure injury on the sacrum with visible bone in the base of the wound. The wound is also draining large amounts of purulent material. Question What is the best solution for daily wound cleansing?

Correct answer: Normal saline Explanation Normal saline is the correct response. Pressure ulcers occur in 20% of nursing home patients and 10% of all hospital patients. Among the elderly admitted to the intensive care units and those admitted for hip fractures, the incidence has been reported as high as 50%. They are caused by pressures on the skin that results in loss of local blood flow with subsequent tissue risk for necrosis. It can be caused by shear forces on the skin/traction on the skin (e.g., when the head of the bed is raised the person tends to slide down). Pressure ulcers occur predominantly in bony prominences (e.g., occiput, spine, sacrum, heels, trochanters, knees, and ankles). The risk factors include: Immobility (paralysis, weakness, stroke, splints, casts, etc). Malnutrition (protein and calories, iron deficiency anemia, dehydration, vitamin, and mineral deficiency) Moisture (urine and fecal incontinence, perspiration) Sensory neuropathy (dementia, neuropathy) Patients should be evaluated for the presence of immobility, malnutrition, moisture, and sensory impairment. The Braden and Norton pressure ulcer risk assessment scales may be used for this. Staging and assessment of pressure sores can be divided as: Stage 1: Nonblanchable erythema of intact skin and the heralding lesion of skin ulceration. This stage may be difficult to identify in darkly pigmented skin. Stage 2: Partial thickness skin loss involving the epidermis and/or dermis; superficial sore presents as an abrasion, blister, or bulla Stage 3: Full thickness skin loss involving damage or necrosis of subcutaneous tissue that may extend down to, but not through, the underlying fascia; sores appear as deep crater with or without undermining of adjacent tissue. Stage 4: Full-thickness loss with extensive destruction, tissue necrosis, or damage to muscle, bone, or supporting structures. Stages 2, 3, and 4 may be difficult to stage if the ulcers are covered with eschar or scab; therefore, they would need to be debrided. Treatment of pressure ulcers is divided into controlling pressure, infection, moisture, and nutrition: Control of pressure: This may be achieved with pressure reducing devices, turning and repositioning the patient often, using lifting devices, using donut cushions, using massage, not dragging patients across surfaces, and not raising the head of the bed. Control of infection: This begins with surgical or chemical debridement (the latter with collagenase) that breaks down the eschar. Debridement is done for less than 2 weeks because it will break down normal tissue. Systemic antibiotics should be reserved for those patients with cellulitis, osteomyelitis, or sepsis. Cultures of the wound are always mixed flora and essentially useless unless taken deeply from biopsy. Superficial swabs are generally not taken. Topical anesthetics and antibiotics are often toxic to healthy tissue. Povidone iodine, soaps, hydrogen peroxide, and acetate should not be used. In the final analysis, only sterile water or saline should be used for irrigation. Control of moisture: Early on, too much moisture breeds infection. Once granulation tissue develops, the process of epithelial cell migration requires moisture and ambient oxygen. With too much moisture, tissues become macerated and swollen and bacterial overgrowth enhanced. With too little moisture, the wound becomes dry; eschar forms, and delayed healing results. Occlusive dressings that are impermeable to oxygen should be avoided because they promote anaerobic infections. Semiocclusive dressings such as DuoDerm should be reserved for relatively clean and shallow wounds in which bacterial overgrowth is less likely. There are a variety of products for deeper wounds to help maintain moisture and retard bacterial growth. Most of these contain hydrocolloids or calcium alginates that absorb moisture from draining wounds and release moisture when the wound begins to dry out. They create a hyperosmotic environment that retards bacterial growth. Typically, an alginate is applied to the wound and removed every 2 or 3 days. Deep wounds and those with sinuses should be packed lightly with gauze to promote exposure to air and drainage. Nutritional support: Malnutrition or inadequate intake of food, water, vitamins, and minerals can retard healing. Canned supplemental diets, tube feedings, and hyperalimentation should be considered. There is mixed evidence about vitamin C and zinc supplements.

Case A 50-year-old Caucasian man presents with an itchy lesion on his left leg. He reports that it has been recurrent for several years, always appearing at the same site. On examination, a coin-shaped, crusted lesion is noted on his left pretibial surface. Question What is the most likely diagnosis?

Correct answer: Nummular dermatitis Explanation This patient most probably has nummular dermatitis; it is characterized by chronic, coin-shaped, crusted lesions which are usually pruritic. They commonly appear on the trunk and extensor surfaces of the extremities, especially the pretibial areas. Atopic dermatitis is characterized by chronic, pruritic lesions marked by exacerbations and remissions. There is a personal or family history of asthma, allergic rhinitis, food allergies, or eczema. In the acute phase, the lesions may present with erythematous patches, weeping, and crusted plaques. In the chronic phase, they present with hyperpigmentated, lichenified lesions. It commonly affects the face, neck, wrists, antecubital, and popliteal fossae. Seborrheic dermatitis is characterized by greasy scales over the scalp. It may also be visible along the hairline, on the eyebrows, in the external auditory canals, in the nasolabial folds, and over the sternum. There may be some pruritus. Hand eczema is characterized by erythema, dryness, cracking, and edema of the skin of the hands. It is a chronic condition which is aggravated by frequent immersion of hands in water and detergents. Pompholyx is characterized by scaling and deep-seated pruritic vesicles on the palms, fingers, and soles. There may also be erythema and scaling.

Question A 15-year-old boy presents for follow-up for acne vulgaris. He has been using benzoyl peroxide and retinoic acid for the past 4 months. He returns for a re-evaluation, as his acne appears to have worsened. He states his diet has not changed significantly, and his past medical history is unremarkable. His mother had significant acne as an adolescent. Physical examination reveals large papules and pustules on the forehead, cheeks, chin, and upper back. No nodulocystic cystic lesions are noted. The most appropriate next step in management is to prescribe what?

Correct answer: Oral doxycycline Explanation Acne vulgaris may occur in as many as 85% of children with the onset between 8 and 10 years in 40% of patients. It is characterized by follicular occlusion and inflammation. The various types of lesions seen with the disorder include open and closed comedones, papules, pustules, nodules, and cysts. Acne is classified into 4 categories: Grade I: noninflammatory acne with comedones Grade II: moderate acne with inflammation, comedones, papules, and occasional pustules Grade III: severe localized inflammatory acne Grade IV: severe, generalized inflammatory acne with pustules, nodules, and cysts Acne is a multifactorial process involving pathogens such as Propionibacterium acnes, abnormal keratinization of the follicular epithelium, and increased production of sebum by the sebaceous glands. There is no solid evidence that certain foods trigger acne or its flare-ups. Cleansing the skin may be helpful by increasing drying and peeling the surface skin, but over cleansing may irritate and chap the skin. Treatment of mild to moderate acne consists of the topical preparations benzoyl peroxide and retinoic acid in varying strengths. Topical antibiotics may be helpful as adjuncts in decreasing the growth of P. acnes. They are most effective if used in conjunction with the other topical preparations. Patients who do not tolerate or who have not responded to topical medications are prescribed oral antibiotics, including doxycycline, tetracycline, and clindamycin. These patients typically have moderate to severe acne with papules, pustules, nodules, and cysts. Patients with severe nodulocystic acne with or without scarring are given isotretinoin for up to 4 months. This medication is teratogenic and is absolutely contraindicated in pregnancy. Individual painful nodulocystic lesions may be treated with intralesional injection of steroids, such as triamcinolone.

Case A 2-year-old boy is brought to your office by his mother after she noticed that he often scratches his head. She also notes patchy loss of hair on the top of his head. She has been sending him to a daycare center for the past 2 months. On examination, you note patchy loss of hair in the right parietal area and another area of "black dot" alopecia about 4 cm lateral to it. The area of hair loss shows a grayish ring-shaped scaly lesion. A KOH preparation demonstrates branching hyphae and spores. Question What is the best treatment for this condition?

Correct answer: Oral griseofulvin Explanation The correct diagnosis is tinea capitis, best treated with oral griseofulvin. Tinea capitis is a fungal infection of the scalp, most common in children living in poor hygiene and overcrowded conditions. The typical lesion is an itchy, scaly, and ring-like lesion, causing irregular or well-demarcated hair loss. Fracture of hair a few millimeters from the scalp results in "black dot" alopecia. Occipital or cervical lymphadenopathy may be present. The most common causative organism in the US is the Trichophyton tonsurans, accounting for 90 to 95% of the cases. It may be demonstrated by KOH preparation; however, it does not fluoresce under Wood's lamp. Oral therapy is necessary for tinea capitis, as topical therapy is ineffective. Griseofulvin is considered the gold standard, although it requires 6 to 12 weeks of therapy and relapse may occur. Other options are oral ketoconazole (risk of hepatotoxicity), itraconazole, fluconazole, and terbinafine. Adjunctive therapy with selenium sulphide or ketoconazole shampoo may reduce viable fungal shedding and spores. Topical or oral steroids are not indicated in uncomplicated tinea capitis.

Question Refer to the image. The 26-year-old man seen in the image is HIV positive with a CD4 count of less than 200. While he was in the hospital for the treatment of his miliary Tb, he developed smooth, skin-colored, umbilicated papules on his face. The lesions are asymptomatic, but spreading gradually to the other parts of the body and causing cosmetic problems to the patient. Which one of the following organism is the most likely cause of this skin infection?

Correct answer: Poxvirus Explanation This patient has typical lesions of Molluscum contagiosum. Molluscum contagiosum is a poxvirus infection, characterized by skin-colored, smooth, waxy, umbilicated papules 2 to 10mm in diameter. Transmission can be direct or venereal. Its contagiousness to others varies. The lesions may be seen anywhere on the skin, face, body or genital area. They are usually asymptomatic unless secondarily infected. This disease is quite common in children, and the lesions can be widespread in patients with reduced cellular immunity. Human papilloma virus 1-4 causes skin and plantar warts. Parvovirus B19 is the cause of erythema infectiosum (slapped-cheek disease, fifth disease). Herpes simplex virus type I causes acute gingivostomatitis, recurrent herpes labialis (cold sores), keratoconjunctivitis, and encephalitis. Ebstein-Barr virus is the cause of infectious mononucleosis.

Question An 18-year-old man presents with multiple painful vesicles on an erythematous base on the right side of his lower lip. He experienced similar symptoms a month before this occurrence with an associated sore throat. He has an oral temperature of 101°F and positive tender cervical lymphadenopathy. What is the most appropriate clinical intervention at this time?

Correct answer: Oral valacyclovir Explanation Mucocutaneous lesions of HSV can be treated with oral valacyclovir, famciclovir, or acyclovir to help clear the lesions. HSV is a viral infection and therefore not susceptible to antibiotic therapy. Punch biopsy is not the diagnostic test in HSV. Herpes simplex virus (HSV) that recurs on the lips can most frequently be diagnosed clinically. Recurrences on the labial areas are the most common site. Tissue culture or a check for HSV antigens in lesion scrapings is required for definitive diagnosis. Intravenous acyclovir is the treatment for HSV encephalitis. Most clinicians are not skilled in performing Tzanck preparation, and this test does not differentiate between HSV and varicella-zoster virus infection.

Case A 4-year-old boy is seen with a 3-week history of generalized rash. Pruritus has been minimal. There has been no fever. The mother noted that the rash began as a single lesion on the abdomen that grew in size as the rash progressed elsewhere; the original lesion is now the largest present. Examination revealed an otherwise well, afebrile child with a generalized rash consisting of scaly, reddish-pink plaques concentrated on the trunk in a Christmas tree pattern with a single lesion at the abdomen greatest in size. Question What area does this condition least commonly affect?

Correct answer: Palms and soles Explanation Pityriasis rosea most commonly presents at the trunk, including both the abdomen and back. Less commonly, it is seen on the extremities, although an inverse form mainly affecting the extremities but sparing the trunk is seen. If the palms and soles are involved, especially in a sexually active patient, secondary syphilis must be excluded.

Case Ico-delete Highlights A 10-year-old girl is brought to you by her mother with a complaint of "really bad dandruff that itches like crazy." The child states that all of her friends at school have it too. Upon physical examination of the hair and scalp, you note numerous oval grayish-white sesame seed capsules, 1 to 2 mm in size deposited on the hair shaft near the scalp. The scalp presents with small red bumps and sores, which appears to be due to the patient scratching her scalp from the itching. Question What is the most likely condition?

Correct answer: Pediculosis Explanation The clinical picture is suggestive of pediculosis capitis. The oval grayish-white sesame seed capsules, 1 to 2 mm in size, deposited on the hair shaft near the scalp represent the eggs that are laid by the parasite, Pediculosis humanis capitis. This is commonly seen in girls more than boys, ages 3 to 11 years old. Psoriasis is described as sharply marginated erythematous papules with silvery-white scales. This is not seen in this patient. Seborrheic dermatitis is described as redness and scaling, and occurs in regions where the sebaceous glands are most active (scalp, gluteal crease, neck line, breast, face). On the scalp, there is mostly marked scaling, which is not seen in this patient. Trichorrhexis nodosa is a defect in the hair shaft, characterized by thickening or weak points (nodes) that cause the hair to break off easily. This is not described in this patient. Atopic dermatitis is an acute, subacute, or chronic relapsing skin disorder that usually begins in infancy (rarely adult), and is characterized principally by dry skin and pruritis. This is not seen in this patient.

Case A 4-year-old boy presents with a 2-week history of severe itching all over the body. It is more severe at night. Physical examination reveals grey thread-like serpentine lines with papules at the ends. There is a generalized papular and papulovesicular rash with few pustules. The lesions are more confluent between the webs of the fingers and toes and on the flexor surface of the wrists, axillae, genitalia, feet, and buttocks. Scrapings from the skin show an arthropod with 4 pairs of legs, hemispheric body, and brown spines and bristles on the dorsal surface. Question What is the treatment of choice for this child?

Correct answer: Permethrin 5% Explanation Permethrin 5% cream is the treatment of choice for a child with scabies. It is the most effective and most convenient drug for scabies. Toxicity of permethrin is very low and reported to be 40-100 times lower than lindane (Gamma benzene hexachloride). Nearly 100% cure rates have been obtained in scabies. It has been found to be more effective than lindane, benzyl benzoate, and crotamiton. Overnight single application of 5% permethrin is required. All close contacts of the patient, even if asymptomatic, should be treated. If required, it may be reapplied after 1 week. Benzyl benzoate 25% emulsion has been used for treatment of scabies. It achieves 76-100% cure in scabies. Emulsion is applied all over, except on the face and the neck, after a bath. Second coat is applied the next day and washed after 24 hours. Though absorption through skin is minimal, neurological symptoms have been reported in children. Skin irritation is also common in children following the application of benzyl benzoate. Crotamiton 10% is effective in treatment of scabies, but it has produced lower cure rates (60-88%) in scabies. Better results are obtained by an application extended 5 days in children. It is a second choice drug for scabies. Invermectin is an anthelmintic drug that is found to be effective in scabies. It is administered orally in a single dose of 0.2 mg/kg (12 mg in adults) and gives a cure rate of 91-100% in scabies. However, it is not to be given to children less than 5 years of age or to pregnant and lactating mothers. Malathion 0.5% in isopropanol for local application is the treatment of choice for head lice and is not used for the treatment of scabies.

Case A 39-year-old woman presents with a rash. The rash started 5 days ago; it is pruritic and located primarily on her arms and legs, with a few sores in her mouth. Her husband described the rash as like a "bullseye". She has felt mildly "flu-like", but she denies fevers. She denies any changes in soaps, detergents, or diet. She has not been around anyone with a similar condition, and she denies travel. Otherwise, she reports feeling better than usual, with more improved control of her migraines since her neurologist started her on topiramate about 3 weeks ago. She has not needed to use her sumatriptan for over a month. Her review of systems is negative. She suffers from migraines, but she has no other chronic health conditions. Her current medications are topiramate daily, with sumatriptan as needed. She is allergic to amoxicillin. She has regular menses; she had a tubal ligation as contraceptive. On physical exam, a few small oral lesions are noted. The lesions on the extremities are primarily on the dorsal surfaces, with a ringed appearance, similar to a target. The remainder of her exam is normal, including vital signs. Question What medication is a recommended treatment for this patient's condition?

Correct answer: Prednisone Explanation The correct response is prednisone. This patient is presenting with a target-lesion rash, typical of erythema multiforme, which is a relatively common type IV hypersensitivity reaction. In about 50% of cases of erythema multiforme, no cause is identified. Other causes are infection and medications, including anticonvulsants. In this patient's case, her rash occurrence suspiciously followed the initiation of a new medication. Erythema multiforme is an acute, self-limited condition; it can range in severity from a rash and mild malaise and pruritus (erythema multiforme minor) to a much more severe condition, involving mucosal surfaces and possible desquamation with erythema multiforme major. The most important treatment of erythema multiforme, when a medication is attributed as the cause, is immediate discontinuation of the new medication. Other recommended treatments vary with the severity of the condition. Antihistamines, topical and oral steroids, and analgesics (if needed) are all appropriate treatments for erythema multiforme. Acyclovir is an antiviral, which is helpful in erythema multiforme associated with herpes simplex virus (HSV) infection. Nothing in this patient's history suggests a recent herpes infection. Doxycycline is a tetracycline antibiotic. It is useful for several bacterial infections and several dermatologic conditions, ranging from acne to anthrax. It does not have a role in the treatment or prevention of erythema multiforme. Methotrexate is an immunosuppressant medication used for a variety of conditions, such as rheumatoid arthritis and severe psoriasis. This patient's rash is not suggestive of psoriasis, and methotrexate would not be appropriate. Vancomycin is an antibiotic; it is used for severe bacterial infections. It has no role in treating erythema multiforme, but its use could cause various skin rashes, such as Stevens-Johnson syndrome and toxic epidermal necrolysis.

Case A 4-year-old boy presents with a rash on his feet, ankles, wrists, and gluteal areas; the rash is accompanied by severe itching, particularly at night. His 6-month-old sister has a similar rash on her neck and head. On examination, you find pruritic erythematous papular and papulopustular skin changes between the web spaces of the fingers; changes are also seen on the flexor aspects of the wrists and in the genital and gluteal areas. On superficial epidermis, you find several short, elevated, red, tortuous lines; they have a small vesicle at the tip. Question What treatment you will suggest?

Correct answer: Permethrin cream Explanation Both your patient and his sibling most probably have scabies: the pathognomonic sign is a short elevated pink or gray, straight or tortuous track in the superficial epidermis; there is also a small vesicle at the tip (burrow). Topical permethrin is the drug of choice. Low-strength hydrocortisone is used for the temporary relief of irritations, itching, and rashes caused by allergic reactions. The presence of similar symptoms in siblings should make you think about the infection or infestation; in such cases, the treatment should be causative. Clindamycin lotion is used as a topical antibiotic for acne. Selenium sulfide is sometimes used to treat tinea versicolor of the skin. Local tar soap is a traditional treatment of scabies; it has been replaced by permethrin, which is far more efficient.

Case A 12-year-old girl presents with a 1-week history of a rash on her trunk. The patient has not been ill or exposed to anyone ill. On examination, there are scattered lesions on her trunk; they look like they form a Christmas tree. The girl states that the lesions are itchy at times, but they are generally not bothersome. Question What diagnosis is suspected?

Correct answer: Pityriasis rosea Explanation Pityriasis rosea is a rash that typically occurs in girls/women and is located truncally. The lesions will generally form in the shape of a Christmas tree. Tinea corporis would be causing an annular pruritic lesion. Seborrheic dermatitis would most likely have scaly patches associated with the lesions. Tinea versicolor would yield hypopigmented or hyperpigmented lesions. The patient has not had any sick contacts or been ill herself, so a viral exanthem can be ruled out.

Case A 17-year-old girl presents with a rash. She states she noted a single oval patch several days before a more generalized rash erupted. She indicates that the rash mildly itches. On physical examination, the initial lesion appears as an erythematous (salmon-colored) plaque with a collarette on the trailing edge of the advancing border. You note a fawn-colored rash that follows the cleavage lines on the posterior trunk. This rash is most prevalent on the trunk, and the proximal upper and lower extremities. Question What is the most likely diagnosis?

Correct answer: Pityriasis rosea Explanation This is a clinical picture of pityriasis rosea. Initially, a single (primary or "herald") plaque lesion develops, usually on the trunk. 1 or 2 weeks later, a generalized secondary eruption develops in a typical distribution pattern. The "herald" patch occurs in 80% of patients. Lesions are usually confined to the trunk and proximal aspects of the arms and legs. Atopic dermatitis is an acute, subacute, or chronic relapsing skin disorder that usually begins in infancy (rarely adult), and is characterized principally by dry skin and pruritus, usually on the flexor surfaces of the skin. It is known as "the itch that rashes;" usually, the patient has a family history of asthma or allergies (IgE-mediated). Psoriasis is described as sharply demarcated and erythematous, as well as surmounted by silvery scales, usually found on the extensor surfaces. Lichen planus is described by the 4 P's - "papule, purple, polygonal, pruritic". Tinea corporis is described as small to large scaling, sharply marginated plaques with or without pustules or vesicles, usually at margins. Peripheral enlargement and central clearing produces annular configuration with concentric rings or arcuate lesions; fusion of lesions produces gyrate patterns.

Case A 33-year-old Caucasian man presents with redness, itching, and burning on the back of his hands. He gives a history of clearing 'weeds' on his farm yesterday. You ask him to describe the weeds, and he states that they had a cluster of 3 leaves. There are several tiny blisters with a linear distribution on both his hands. Question What is the most likely diagnosis?

Correct answer: Poison ivy dermatitis Explanation The most likely diagnosis is contact dermatitis induced by poison ivy. Poison ivy, poison oak, and poison sumac release an oil, ursodiol, when damaged. Urshodiol induces an allergic reaction in susceptible persons called contact dermatitis. An itchy rash and blisters in a linear distribution (where the plant brushes against the skin) are produced. About 80 to 90% of adults are allergic to poison ivy. Those with outdoor occupations (e.g., farming, forestry, and construction) are at greater risk of exposure. Poison ivy can be identified by the cluster of 3 leaves. Poison sumac has a cluster of 7 - 13 leaves and may also cause contact dermatitis. Although an insect bite is a possibility, the linear distribution of the blisters and the patient's history of clearing plants makes dermatitis due to poison ivy more likely. Sunburn affects all exposed parts of the skin, not just the back of the hands. A nickel allergy would affect the palms (where the tools are held).

Case An adolescent patient known to your practice has come in to see you because of her "annoying skin". She states that around the Spring and Fall season changes, areas of her skin will become significantly irritated, red, itchy, and dry; sometimes, the areas will even drain clear fluid. She has had these problems with her skin for many years, but she has noticed recently the symptoms increasing in severity. Areas affected are around the inside of her elbows and behind both knees. This patient has a history of asthma, which is maintained successfully with a daily inhaled corticosteroid and an as-needed short acting beta-agonist. Pertinent family history includes her mother having asthma and her father having very similar dermatologic problems. Question What instruction would you initially give her in regards to treatment to help better control the patient's symptoms?

Correct answer: Prescription topical corticosteroids Explanation The correct response is prescription topical corticosteroids. The patient in this clinical scenario is most likely suffering from atopic dermatitis, which is otherwise known as eczematous dermatitis. The diagnosis can be deduced from several components: symptoms occur seasonally; the patient has diagnosed components of the atopy triad; and there is family history of atopic issues on both her mother and father's side. A major cause of eczematous dermatitis is seasonal and perennial airborne allergies, which if severe enough, may cause significant eczematous issues. Other causes that can lead to outbreaks include food allergies, substances that come into contact with the skin, changes in temperature and humidity, and even psychological stress. Whatever is causing eczematous dermatitis, lifestyle modifications (avoiding factors that trigger the flare ups) as well as medications is needed. Topical corticosteroid creams are considered the mainstay of treatment of eczematous dermatitis. Topical corticosteroids help reduce the inflammatory reaction of eczema and decrease the patient's urge to itch. These creams are found in mild-, medium-, or high-potency forms depending on the severity of the pathology. Lifestyle modifications are also stressed in patients. Use of topical and oral antihistamines can be utilized to relieve pruritus or even cause sedation. Oral corticosteroids may be prescribed in significantly severe acute exacerbations. Antibiotics are only prescribed if an active microbial infection is evident in the affected areas. Allergen immunotherapy has been widely used in patients with eczematous dermatitis caused by atopy with positive results. A meta-analysis of 8 randomized trials, which included 385 patients, found that those receiving allergen specific immunotherapy (versus placebo) were more likely to experience success, and it improved the course of their eczematous dermatitis. Many variables can affect the success of this course of treatment, including (but not limited to) preparations of allergens, treatment schedules, the patients' age, and severity of symptoms.

Case A 20-year-old African-American woman presents with itchy rashes on her hands. She reports that they have been persistent for the past 2 years. On examination, deep-seated vesicles with scaling on her palms are noted bilaterally. Question Ico-delete Highlights What is the most likely diagnosis?

Correct answer: Pompholyx Explanation This patient most probably has pompholyx. It is characterized by scaling and deep-seated vesicles on the palms, fingers, and soles; the vesicles are pruritic. There may also be erythema and scaling. Atopic dermatitis is characterized by chronic pruritic lesions marked by exacerbations and remissions. There is a personal or family history of asthma, allergic rhinitis, food allergies, or eczema. In the acute phase, the dermatitis may present with erythematous patches, weeping, and crusted plaques. In the chronic phase, it presents with hyperpigmented lichenified lesions. It commonly affects the face, neck, wrists, antecubital, and popliteal fossae. Seborrheic dermatitis is characterized by greasy scales over the scalp. It may also be visible along the hairline, on the eyebrows, in the external auditory canals, in the nasolabial folds, and over the sternum. There may be some pruritus. Hand eczema is characterized by erythema, dryness, cracking, and edema of the skin of the hands. It is a chronic condition which is aggravated by frequent immersion of hands in water and detergents. Nummular dermatitis is characterized by chronic coin-shaped, crusted lesions which are usually pruritic. They commonly appear on the trunk and extensor surfaces of the extremities, especially the pretibial areas.

Case A 14-year-old African American boy presents to your clinic with a several-week-long history of the appearance of spots on his face; they produce pus when he squeezes them. Over the past 1 week, he has also noticed similar spots on his back. He recently began smoking cigarettes. His 17-year-old brother also has similar spots on his face. Question What is not a pathogenic factor associated with his condition?

Correct answer: Poor hygiene Explanation The clinical presentation is that of acne vulgaris. Acne is a common skin condition that affects the pilosebaceous units. It usually begins in adolescence; however, it may also affect adults. The main pathogenic factors that contribute to the development of acne are: Excessive sebum production due to androgen hormones and sebaceous gland hyperplasia. Follicular epidermal hyperproliferation, which prevents normal keratinocytes shedding, resulting in follicular canal obstruction and the formation of microcomedo. Cellular debris and lipids collect within the blocked follicle and form noninflammatory follicular papules called closed comedones ("whiteheads") and open comedones ("blackheads"). Propionibacterium acne's colonization of the blocked follicle triggers an immunological reaction with the release of proinflammatory mediators. Follicular rupture and spilling of the cellular debris, lipids, and bacteria into the dermis further increases the inflammation. This results in the inflammatory papules, pustules, and nodules; it may lead to fibrosis and scarring. Acne may be aggravated by several factors, which include: Cosmetics containing comedogenic agents Mechanical obstruction and friction by shirt collars and helmet chin straps Medications such as phenytoin, isoniazid, lithium, systemic corticosteroids, and bromine and iodine-containing medications. Dietary factors and stress may contribute to the development of acne; however, these factors remain controversial in their role in acne. There is no evidence that poor hygiene triggers acne. The diagnosis of acne is based on the history and physical examination. The face is the most commonly affected area; however, the neck, chest, and back may also be affected. In addition to the acne lesions, there may also be scars and postinflammatory hyperpigmentation.

Question A 16-year-old girl has moderate facial acne vulgaris which you plan to treat with tretinoin-containing creams. What potential condition should you rule out before treating the patient with retinoic acid cream?

Correct answer: Pregnancy Explanation Tretinoin creams contain a retinoid that is a metabolite of naturally occurring vitamin A; used topically, they are an effective treatments for acne vulgaris. Unfortunately, retinoids are also potent teratogens; they have been shown to have effects on bone development in laboratory animals. Their use is contraindicated in pregnant women; therefore, a pregnancy test using a sensitive assay for the beta subunit of human chorionic gonadotropin (β-hCG) must be used to rule out pregnancy before it is safe to begin treatment. Graves' disease manifests with changes in skin, hair, and nails, but it is not exacerbated by retinoid treatment. Psoriasis is a chronic inflammatory skin condition; approximately 2% of the United States population has this condition. Retinoids would not have an adverse effect on this condition. Contact dermatitis and dermatitis caused by poison ivy are both common minor skin irritations; neither are incompatible with retinoid treatment of acne vulgaris.

Question A 16-year-old girl presents with a history of severe nodulocystic acne. She has used topical benzoyl peroxide and topical tretinoin and has undergone numerous courses of oral tetracycline. The acne has not responded to any treatment thus far. You are considering using isotretinoin (Accutane) to treat her condition. Before beginning treatment of this patient, what must be obtained?

Correct answer: Pregnancy test Explanation Isotretinoin (Accutane) is used in the treatment of moderate to severe nodulocystic acne that has not responded to conventional treatment, scarring acne such as acne fulminans, and acne associated with psychological disturbance. It is generally taken for 16-20 weeks and produces a cure in 30% of patients. An additional 35% of patients still need further conventional therapy, and the remainder suffers relapses and often requires a second course of isotretinoin. Isotretinoin causes a decrease in sebum production, decreases the inflammatory response, and decreases the population of Propionibacterium acnes. It has many side effects. The most important is its teratogenicity. It is absolutely contraindicated in pregnancy. Pregnancy should be avoided up to a month after discontinuation of therapy. 2 or 3 forms of birth control should be used. Pregnancy tests should be performed prior to starting therapy and monthly thereafter. Increased serum triglyceride and cholesterol levels may occur; preexisting liver disease should be ruled out prior to initiating treatment. Other side effects include cheilitis, epistaxis, xerosis, blepharoconjunctivitis, arthralgias, myalgias, depression, paronychia, colonization of the skin with Staphylococcus aureus, and temporary thinning of the hair.

Case A 35-year-old man presents with a skin eruption that has been developing slowly for several months. The lesions are symmetric, involve primarily the elbows, knees, and scalp, and appear as erythematous, sharply demarcated plaques covered by a silvery scale. Question What is the most likely diagnosis?

Correct answer: Psoriasis Explanation Psoriasis is a chronic inflammatory dermatitis that has very characteristic manifestations. The skin eruption tends to be symmetric and to involve primarily the elbows, knees, and scalp, although multiple sites may be affected in advanced disease. The lesions are well-demarcated, erythematous plaques, with a characteristic scale that appears silvery in color. The distribution and appearance of psoriatic skin lesions are not found in erythema multiforme, lichen simplex chronicus, fixed drug eruption, or eczematoid dermatitis.

Case A 35-year-old obese man presents with a 6-month history of itchy rash. He was referred to you by his primary care physician, who treated him for tinea cruris over a period of 4 months with limited success. A brief history reveals the patient is bipolar on lithium, does not smoke, and takes no other medications aside from the terbinafine (Lamisil) tablets prescribed by the primary physician. On physical examination, the rash is located primarily in the intertriginous areas of the groin, but it can also be seen in the axillary and sacral regions. The morphology is variable, but it is largely consistent with that of erythematous plaques. Scaling is widely observed, except for in the groin, where the plaques are moist and more confluent. The dermatologist does a KOH preparation of the scrapings and finds no hyphae. Question What is the most likely diagnosis?

Correct answer: Psoriasis Explanation Psoriasis is a very common chronic inflammatory condition of the skin. The lesions are variably pruritic and are characterized by sharply demarcated papules and rounded plaques. A silvery scale is frequently observed covering the erythematous plaques. Depending on the distribution and character of the lesions, psoriasis can be further categorized into several sub-groupings. The most common subtype (plaque type) of psoriasis is usually distributed on the elbows, knees, sacral area/gluteal cleft, and scalp. However, there is also a less common variety known as inverse psoriasis. This is where the plaque lesions form in intertriginous regions in addition to scalp, palms, and soles. Because of the moisture in the intertriginous areas, scales may not be evident. Other forms of psoriasis include eruptive or guttate psoriasis as well as some variants where the lesions are more pustular in character. The etiology of psoriasis is not well defined, but there may be some association with certain medications, such as lithium and beta-blockers. Atopic dermatitis is the skin's reaction to allergy, be it food, asthma, or animal dander, etc. In children and adolescents, it is frequently localized to the flexural skin creases of the antecubital and popliteal fossae. Skin injury is more often a result of scratching than of the atopic process itself, and these individuals have a higher incidence of Staphylococcus aureus skin infections than do unaffected patients. Patients with atopic dermatitis are advised to avoid irritants and to keep skin moist (which includes avoiding hot showers and profuse scrubbing). Treatment often includes the judicious use of low-dose topical glucocorticoids and conservative administration of antihistamines to reduce the itching. Intertrigo is not an answer choice, but discussion here may be helpful. Stedman's Medical Dictionary (26th edition) defines intertrigo as: "irritant dermatitis occurring between folds or juxtaposed surfaces of the skin, as between the buttocks, between the scrotum and the thigh, beneath pendulous breasts, etc., caused by friction, sweat retention, moisture, warmth, and concomitant overgrowth of resident microorganisms, and occurring in young children and obese adults." Therefore, a patient with intertrigo could indeed have candidiasis, but a patient with candidiasis does not necessarily have intertrigo. Candidiasis occurs because of overgrowth of this group of yeasts in particular areas of the skin, those that are chronically wet, and especially in the intertriginous zones of the groin and beneath pendulous breasts. Patients with diabetes, chronic intertrigo, and cellular immune deficiency (as in HIV) are particularly susceptible. Obese patients have more intertriginous areas and are thus at risk. The rash is said to be macerated (i.e., displaying a waterlogged or soaked appearance characteristic of the dead surface skin). It is also described as erythematous, with marked inflammation, and it may be in a "satellite lesion" distribution. That is, smaller lesions a couple of cm out from the main larger one, which the "satellites" appear to orbit. The lesions may show scaling and skin scrapings will show pseudohyphae and yeast forms. Lichen planus is a condition that produces primary lesions described as pruritic, polygonal, flat-topped violaceous papules. Lesions may show thin grey lines (Wickham's striae), and they have a tendency to occur on wrists and shins, but can occur anywhere on the skin. Mucous membranes including the buccal mucosa can be involved. The etiology is not completely understood, and the course is variable, but the lesions usually disappear spontaneously within several months to 2 years. Treatment may include topical glucocorticoids. The rash of secondary syphilis can indeed manifest as moist pink lesions in the intertriginous regions. This occurs in about 10% of patients with secondary syphilis, and the lesions are called condylomata lata. However, secondary syphilis is not the best answer. While syphilis is notorious for masquerading as other diseases, there are certain clues to look for that are more typical of syphilis. The rash, which may be subtle, usually manifests as symmetric mucocutaneous lesions with some truncal distribution. Furthermore, nontender generalized lymphadenopathy is usually present and the primary chancre of syphilis will still be present in about 15% of patients with secondary syphilis. Finally, RPR should be positive in a patient with secondary syphilis. Other factors to consider are the patient's obesity, diabetes, complaints of vulval pruritus consistent with candidal vulvovaginitis, and positive HIV status. In summary, secondary syphilis would be an appropriate consideration in this patient, but it would not be the best answer in this case. Of parenthetical note, IV drug users have a high (upwards of 25%) false positive rate by reagin-type tests, such as RPR. Here is a list of each of the possible answer choices together with an oversimplified "lightning flash" association: Atopic dermatitis...allergic, flexural creases Candidiasis...diabetic, chronic intertrigo Lichen planus...pruritic polygons Psoriasis...silvery scaled papules and plaques Secondary syphilis...the masquerader

Question A 50-year-old woman has had a facial rash and a 10-year history of reddening of her face that comes and goes. On physical exam, you note erythema, telangiectasia, red papules, and tiny pustules on both cheeks. What is the most likely diagnosis?

Correct answer: Rosacea Explanation The clinical picture is suggestive of rosacea. Rosacea is a chronic inflammatory condition of the facial pilosebaceous units. Peak incident occurs age 40-50; it predominantly affects women. Lesions develop in stages with a history of episodic reddening of the face. Skin lesions can appear as tiny papules and papulopustules, persistent erythema, and telangiectasia. Distribution is symmetric and primarily affects the face. Perioral dermatitis presents as 1-2 mm erythematous papulopustules on an erythematous background; they may have tiny scales around the mouth. This disorder is primarily seen in women age 16-45; facial creams, soap, and steroids are typical causes. Seborrheic dermatitis can appear as orange-red or gray-white skin, as "greasy" or white dry scaling macules, and as papules of varying size that are sharply marginated on the scalp, face, and trunk. Atopic dermatitis is an acute or chronic relapsing skin condition characterized by dry skin and itching. Lichen simplex is a localized form of lichenification resulting from repetitive rubbing and scratching. The constant scratching causes thick leathery brownish skin.

Case A 2-day-old infant still in hospital care is found to have erythematous patches with 1-2 mm yellow pustules with a surrounding irregular erythematous base, giving a "flea-bitten" appearance scattered all over the body. The infant was full-term, delivered via spontaneous vaginal delivery without complication. The mother received regular prenatal care. The patient is feeding well and has had several wet diapers and 2 meconium stools. Vital signs have been normal since delivery. On exam, patient is alert and awake. No sign of jaundice, no murmur, and lungs are clear. Question What should be done at this time?

Correct answer: Routine monitoring and parental education Explanation The correct response is routine monitoring with parental education. Erythema toxicum neonatorum is a benign transient condition occurring in 48-72% of full-term newborns. Appearance is as described in the patient scenario. Additionally, the appearance may be only splotchy erythema. The differential diagnosis is extensive, but the most common differential diagnoses include: staphylococcus folliculitis acne neonatorum congenital candidiasis Treatment begins with education of the parents of the benign nature, duration of the condition and assurance that the condition will resolve without any sequelae. Biopsy would yield a characteristic infiltrate of eosinophils with the absence of organisms. No treatment is necessary, and the rash disappears within a few days. The differential includes candidosis, herpes simplex, transient neonatal pustular melanosis, and miliaria. Candidosis and herpes would warrant immediate transfer to the NICU with full diagnostic workup and further treatment.

Question A 23-year-old man presents with unbearable itching in his genital area. The itching increases in intensity at night. He admits to several recent sexual encounters with different people in the past month. Skin exam reveals multiple excoriated papules and burrows with surrounding inflammation. What is the most likely diagnosis?

Correct answer: Scabies Explanation The correct answer is scabies because intense pruritis, especially at night, with excoriated papules and characteristic burrows or raised tunnels under the skin are a typical presentation. Tinea cruris is a fungal infection and can also be intensely pruritic but does not cause the dermatologic manifestations noted above. Pediculosis pubis, an arthropod, is another cause of pruritis but does not cause the dermatologic manifestations noted above, and typically the lice and their nits are seen on examination. Molluscum contagiosum and HPV are caused by viruses, are not pruritic lesions, and do not present as excoriated papules with burrows.

Case A 26-year-old African-American man presents with a rash on his back. He first noticed the rash 3 weeks prior to presentation. He describes small, whitish upper back lesions that are not painful and do not itch. He has no significant past medical history; he has no known drug or food allergies, and he is not taking any medications. On physical examination, there are several small, hypopigmented macules that coalesce on the upper 1/3 of his back. Fine scaling is produced on scratching. The remainder of the physical examination is unremarkable. Question Ico-delete Highlights How should you proceed in the management of this patient?

Correct answer: Scrape lesions and KOH stain Explanation This patient's presentation is consistent with tinea versicolor. Tinea (or pityriasis) versicolor is a fungal infection caused by Malassezia furfur. The lesions present as small, discrete macules or papules that tend to be darker than the surrounding skin in light-skinned patients and hypopigmented in patients with dark skin. They often coalesce to form large patches of various colors ranging from white to tan; thus the name 'versicolor'. Tinea versicolor most commonly involves the upper trunk; however, the arms, axillae, abdomen, and groin may also be affected. To confirm the diagnosis, a KOH preparation of scrapings from the lesions should be done, which can demonstrate pseudohyphae and spores resembling 'spaghetti and meatballs'. Treatment of tinea versicolor involves applying selenium sulfide shampoo topically. An alternative is the use of topical azoles (anti-fungal agents), such as ketoconazole, miconazole, and clotrimazole. For patients who have difficulty using topical agents, oral ketoconazole or fluconazole may be attempted, although the use of topical agents is preferred. 'Obtain a fungal culture from the lesion' is an incorrect response. Fungal cultures of skin lesions are appropriate when certain infections (e.g., blastomycosis)?are suspected. The infection is diagnosed by identifying the fungus in a culture taken from a skin lesion. Doing so usually requires a skin biopsy. 'Perform a skin biopsy' is an incorrect response. Skin biopsy had no place in the diagnosis or treatment of superficial fungal infection of the skin. Terbinafine and griseofulvin are anti-fungal agents, but they are not effective against Malassezia furfur.

Case A 42-year-old man presents with a 3-month history of a persistent, slowly worsening rash. He changed soaps and shampoos without any effect, and he got some relief with OTC 0.5% topical hydrocortisone. He experienced the same symptoms a year ago from February to March, but it was not as severe as this time, and it resolved without treatment after 6 weeks. The rest of his past medical history is unremarkable, and he does not take any medications. On examination, you see diffuse, yellowish, greasy scaling of the scalp, the forehead, upper chest, and groin area, with scaling papules along the hairline, behind the ears, at the nasolabial folds, and in the external auditory canals. Question What is the most likely diagnosis?

Correct answer: Seborrheic dermatitis Explanation This is the typical picture of seborrheic dermatitis. In addition to the dandruff with variable itching, marginal blepharitis and conjunctival irritation may be present. In neonates it is known as cradle cap, and it can be associated with stubborn diaper rash. It more likely occurs in fall and winter. Pityrosporum ovale seems to play a role in the development of the disease; therefore, ketoconazole in addition to topical steroids is used for treatment. Nummular dermatitis is a chronic skin disease characterized by inflamed, coin-shaped, vesicular, crusted scaling as well as pruritic lesions. The efflorescences are often more prominent on buttocks and extensor areas of the extremities. There is no uniformly effective treatment. Antibiotics, steroids, UV-B radiation, or UV-A radiation combined with psoralens may be tried. Atopic dermatitis is a chronic, itching, superficial inflammation of the skin; it is often associated with a family history of allergies. It can start at any age, but most commonly begins during the initial months of life with red, weeping, crusted lesions on face, scalp, diaper area, and extremities, typically at the flexor side of the joints and the earlobe. The disease can resolve, but it can also exacerbate during childhood or adolescence. Corticosteroid application should be alternated with hydrophilic petrolatum and hydrogenated vegetable oil. Bathing should be minimized and soap use avoided. Lichen simplex (or localized scratch dermatitis) is a chronic, pruritic, superficial skin inflammation distinguished by dry, scaling, sharply demarcated, circumscribed, hyperpigmented, lichenified patches. It is common in Asian and American Indians. It starts between the ages of 20 and 50, mostly in women. It affects the face, nuchal region, extremities, genital, and perianal region. Topical steroids are effective in controlling the itching, but the patients also need to be aware that the skin changes are due to their rubbing and scratching. Exfoliate dermatitis is a widespread, scaly erythematous skin disease that is often associated with loss of hair and nails, pruritus, as well as hyperkeratosis of the palms and soles. The etiology is unknown; sometimes, there is a connection to other skin diseases, mycosis fungoides, lymphoma, or systemic drug intake. It is important to determine the cause because this can be a life-threatening disease. Topical steroids give temporary relief.

Case A 20-year-old Caucasian woman presents to the hospital with a 2-month history of itchiness of her scalp. On examination you note greasy yellowish scales on her scalp and eyebrows. Question What is the most likely diagnosis?

Correct answer: Seborrheic dermatitis Explanation This patient most probably has seborrheic dermatitis. Itis characterized by greasy scales over the scalp. It may also be visible along the hairline, on the eyebrows, in the external auditory canals, in the nasolabial folds, and over the sternum. There may be some pruritus. Hand eczema is characterized by erythema, dryness, cracking, and edema of the skin of the hands. It is a chronic condition, which is aggravated by frequent immersion of hands in water and detergents. Atopic dermatitis is characterized by chronic pruritic lesions marked by exacerbations and remissions. There is a personal or family history of asthma, allergic rhinitis, food allergies, or eczema. In the acute phase, they may present with erythematous patches, weeping, and crusted plaques. In the chronic phase, they present with hyperpigmented lichenified lesions. It commonly affects the face, neck, wrists, antecubital, and popliteal fossae. Nummular dermatitis is characterized by chronic coin-shaped, crusted lesions, which are usually pruritic. They commonly appear on the trunk and extensor surfaces of the extremities, especially the pretibial areas. Pompholyx is characterized by scaling and deep-seated vesicles on the palms, fingers, and soles, which are pruritic. There may also be erythema and scaling.

Case A 28-year-old man presents with a rash. The lesions, which are mildly pruritic, are located on his arms and legs. They have been present for about 3 days, without change or resolution. He has tried over-the-counter anti-itch creams, but they have been ineffective. The patient reports that he was seen approximately 1 week ago for some blister-like lesions on his penis. He was given an antiviral medicine, and those lesions resolved. He is wondering if he was misdiagnosed and if the 2 rashes are related. He admits to feeling some malaise over the last 2 weeks, but he is otherwise healthy. He denies fevers, unusual travel, medication use (except for as listed above), and known allergies. He has no known chronic conditions. On physical exam, vitals are normal; the patient is in no apparent distress. A pink-to-red papular rash is observed on the backs of the hands and feet and extensor surfaces of the arms and legs. The individual lesions are quite distinct; they have a red center, and they are surrounded by a pale ring and then another outer ring of red, inflamed tissue. The remainder of his physical exam is normal. Question What test will confirm the suspected diagnosis?

Correct answer: Skin biopsy Explanation This patient is presenting with a target-lesion rash; it is typical of erythema multiforme, which is a relatively common type IV hypersensitivity reaction. The confirmatory test is skin biopsy. In about 50% of cases of erythema multiforme, no cause is identified. Other causes are infection and medications. When a cause is identified, recent herpes simplex virus (HSV) infection is the most common. In this patient's case, he likely was appropriately diagnosed and treated for his HSV infection, then developed the erythema multiforme rash. Erythema multiforme is an acute, self-limited condition and can range in severity from a rash and mild malaise and pruritus (erythema multiforme minor) to a much more severe condition, involving mucosal surfaces and possible desquamation with erythema multiforme major. A complete blood count (CBC) with differential is a useful test in many conditions in which infection is on the differential. However, it is rarely confirmatory for dermatologic conditions. This patient may have a slight leukocytosis, but the differential may be variable, with possible monocytosis and/or neutrophilia. A fungal culture would not be useful for the diagnosis. Ringworm (tinea circinata) may come to mind with ring-shaped lesions, but these fungal lesions do not have the characteristic red center with target appearance. Ringworm lesions more typically feature central clearing with a red outer ring. A herpes culture would have been useful in confirming the diagnosis of the initial rash, which the patient described as blister-like on his penis. Culturing the current erythema multiforme target lesions would not yield HSV. A heterophile antibody is a test for infectious mononucleosis. A mild rash may occur with mononucleosis, but there are typically several other key symptoms and findings, such as pharyngitis, lymphadenopathy, and profound fatigue. A skin scraping with potassium hydroxide (KOH) is another useful test for fungal causes of skin conditions. The skin is scraped with a microscope slide and examined under a microscope for fungal hyphae. This test is not useful for erythema multiforme.

Case A 7-year-old boy presents with a 3-day history of intense pruritus on his wrists, fingers, and antecubital fossae. It began on his fingers and has moved proximally. According to his mother, they are raised red eruptions and some have scabbed over. She states he has never had this before and it is keeping him up at night. He recently started at a new school. Question What procedure would be most helpful in confirming your diagnosis?

Correct answer: Skin scraping with immersion oil Explanation Skin scraping with immersion oil is the confirmatory test for scabies, which is the most likely diagnosis. Wood lamp inspection consists of shining ultraviolet light on the skin, looking for fluorescence indicative of such diseases as erythrasma, fungal infections, and sclerosis. Skin scraping with KOH is helpful in diagnosing fungal infections of the skin. Punch biopsies are more invasive, and they are used for deeper skin lesions. Skin culture and Gram stain are not helpful in the diagnosis of scabies; they are used for suspected bacterial or fungal infections.

Case A mother brings her 5-year-old son in due to papular and pustular lesions on his face. A serous honey-colored fluid exudes from the lesions. You suspect impetigo. A Gram stain reveals spherical Gram-positive arrangements in irregular grape-like clusters. Question What is the most likely organism causing this patient's condition?

Correct answer: Staphylococcus aureus Explanation The history and lab findings suggest Staphylococcus aureus as the causative organism. The most common causes of impetigo are usually more than likely Staphylococcus aureus, but also at times may be beta-hemolytic streptococcus group A. S. aureus causes inflammatory and toxin-mediated diseases. When a gram stain has been completed, S. aureus will appear as spherical Gram-positive cocci arranged in irregular grape-like clusters, such as what is described in this patient. Inflammatory diseases that can be caused by an active S. aureus infection include the following: skin infections, including impetigo, furuncles, carbuncles, and cellulitis, surgical wound infections, eyelid infections, and postpartum breast infections; septicemia (sepsis), which can originate from any localized lesion, especially wound infection, or as a result of intravenous drug abuse; endocarditis on normal or prosthetic heart valves; osteomyelitis and arthritis, either hematogenous or traumatic; pneumonia in postoperative patients or following viral respiratory infection, especially influenza; and abscesses (metastatic) in any organ. Streptococcus pneumoniae are Gram-positive lancet-shaped cocci arranged in pairs (diplococci) or short chains. On blood agar they produce alpha-hemolysis. Virulence factors of pneumococci are polysaccharide capsules. Pneumococci cause pneumonia, bacteremia, meningitis, and infections of the upper respiratory tract such as otitis and sinusitis. Mortality rate is high in elderly, immunocompromised (especially splenectomized), or debilitated patients. They should be immunized with the polyvalent polysaccharide vaccine. Peptostreptococci are anaerobic Gram-positive cocci. It grows under anaerobic or microaerophilic conditions and produces variable hemolysis. Peptostreptococcus is a member of the normal flora of the gut and female genital tract, and participates in mixed anaerobic infections of the abdomen, pelvis, and brain. Staphylococcus epidermidis is part of normal human flora on the skin and mucous membranes, but can cause infections of intravenous catheters and prosthetic implants. This organism is particularly infectious in the inpatient hospital setting in patients with compromised immune systems. Gram stain of this bacterium reveals Gram-positive cocci arranged in grape-like clusters. Although the Gram stain characteristics are similar to the correct answer, the patient scenario is inappropriate. Haemophilus influenzae is the incorrect choice in this scenario due to the fact that on Gram stain characteristics seen include Gram-negative and rod-shaped.

A 40-year-old man presents with fever, weakness and pain in his right hand. Fever and weakness started this morning. 2 days ago, he noticed painful itching blisters on his hand. He thinks that it might have been caused by a spider bite; he saw several while he was cutting a juniper bush. He has diabetes mellitus type II and was treated in the ER for acute alcohol intoxication 2 times over the last month. His records show that he received a tetanus booster dose 5 years ago. On examination, he appears very ill; his blood pressure is 85/55 mm Hg. HR is 120/minute, and respiratory rate is 22/min. His hand is swollen and red; there is a small deep ulcer surrounded by a blue halo, with preserved voluntary movements of the hand and fingers. You also notice a crepitus and the discharge of fluid that resembles 'dish water'. His laboratory findings are below: Findings Normal Units Urine glucose Positive 0 Urine myoglobin Positive 0 Glucose 250 140 mg/dl BUN 256 1.8 - 71 mmol/L Blood alcohol levels 0.02 0 % by vol. WBC 25000 4,500 - 10,000 per cubic mm Neutrophills 68 54 - 62 % Lypmphocytes 24 24 - 44 % CRP 1000.0 1.0 mg/L Serum creatinine 2.6 0.7 to 1.3 mg/Dl Serum potassium 6.0 3.5 - 5.2 mEq/L Serum sodium 137 135 - 147 mEq/L Question What is the next best step in management of this patient?

Correct answer: Surgical debridement Explanation Your patient most probably has necrotizing fasciitis. He has at least 2 risk factors (diabetes and alcoholism); there are also signs and symptoms of rapidly evolving infection of deep skin layers (of fascia and subcutaneous tissue). There is relative sparing of the muscles, and he has laboratory values pointing to a severe infection. Necrotizing fasciitis is a severe disease of sudden onset and must be treated immediately with extensive, early surgical debridement. A dopamine infusion is used in the management of shock; even if your patient is developing shock, dopamine will not change the final outcome if the cause is not removed. Many types of bacteria can cause necrotizing fasciitis, either in a polymicrobial or monomicrobial form. You should consider parenteral administration of broad-spectrum antibiotics; however, antibiotics without the removal of the infected tissue would not significantly change the outcome. Tetanus prophylaxis with immunoglobulin administration may be considered; however, doing so would not change the outcome of the necrotizing fasciitis. Systemic antihistamines are used in the treatment of histamine-mediated allergic conditions, including skin reactions, but your patient clearly has an infection, not an allergy.

Case A 52-year-old Caucasian man with a PMH of uncontrolled diabetes and hypertension presents with onychomycosis of the toenails. He states that it just keeps getting worse and the antifungal cream he purchases from the local pharmacy is not working. He is having difficulty trimming his nails and is starting to have some discomfort with ambulation. He has added discomfort trying to get his compression stockings over his feet, which he wears due to his venous insufficiency. The PA and patient both agree to start oral therapy, and his recent hepatic panel shows no elevation in his liver enzymes. Question What oral pharmacological therapy would be indicated at this time?

Correct answer: Terbinafine (Lamisil) Explanation Terbinafine is the correct answer choice. Terbinafine has the highest rate of success compared to Itraconazole and Griseofulvin. Therapy is usually about 3 months with monitoring liver enzymes after 6 weeks of use. Your patient should be treated this way, as fungal infection can increase the risk of a cellulitis due to uncontrolled diabetes and leg edema. Unfortunately, even with oral therapy, there is up to a 50 percent reoccurrence rate. Ciprofloxacin is a Quinolone antibiotic. Unless there is a secondary bacterial infection, an antibiotic would not be useful in this situation. Lamotrigine is an anticonvulsant. It would serve no purpose in this situation. Mebendazole is an antihelmintic. There is no mention of parasites in the case above. Metronidazole is also an antibiotic and an amebicid used to treat anaerobic infections and pseudomembranous colitis.

Case A 24-year-old woman presents with a rash. She has never had a rash like it prior to presentation. You ask about symptoms, and the only thing she states is that it mildly itches. On physical examination, you note scattered multiple lesions with marked vesicles, pustules, sharply marginated plaques with central clearing, and crusting at the margins that produces annular configurations; the rash appears only on the arms, neck, and trunk. Question What detail of the patient's history, if elicited, may denote the etiologic cause of this disorder?

Correct answer: The patient works with animals Explanation The clinical picture is suggestive of Tinea corporis. Patients who work with animals can contract this condition. Medications, allergies, or medication allergies do not cause this disorder. Tinea corporis is a fungal infection. A patient's history may be useful, but identifying the patient's occupation is most helpful in confirming your suspicions.

Case A 13-year-old boy presents for the treatment of acne vulgaris that did not respond to antibiotic therapy. On physical examination, the patient's torso and shoulders showed several ill-defined annular lesions, with fine scale and decreased pigmentation. Samples of the skin were obtained, and a potassium hydroxide preparation was positive for hyphae. A diagnosis of tinea versicolor is considered. Question What is true regarding the patient's condition?

Correct answer: There is a high recurrence rate. Explanation Recurrence of tinea versicolor is common because the fungus that causes it normally lives on the skin. It is more common in areas with a warmer climate. Tinea versicolor, also known as pityriasis versicolor, is a skin infection. It is very common, especially in young adults. Men and women are affected equally. Malassezia furfur, which is found normally on the skin of most adults, is responsible for tinea versicolor. The disease most commonly appears as fine, scaly, well-marginated macules that are round to oval in shape. They are usually found on the trunk or chest and occasionally on the abdomen, neck, and extremities. The macules may eventually combine to form patches of various colors, hence the name. Nails and mucous membranes are not affected. The diagnosis is usually made based on clinical examination findings; however, the diagnosis is easily confirmed by microscopic examination of scales soaked in 10-15% potassium hydroxide. An ultraviolet light may also be used to show the infection more clearly. Tinea vesicolor does not usually cause pain, but involved areas of the skin do not tan, which causes patches of skin that are lighter in color than surrounding skin. Tinea versicolor may be diagnosed by its appearance. Treatment is usually with topical antifungal cream. Other topical treatments include azole and allylamine antifungals, sodium sulfacetamide, and selenium sulfide. The skin may not regain its normal pigmentation for many months after the infection is treated.

Question Ico-delete Highlights An 8-year-old boy presents with a pruritic rash. The rash began as a few small, red bumps on the arms a few weeks ago, but it has now spread across his body, and the itching has gotten worse. The itching is worse at night, and he is having problems sleeping because of the pruritus. His mother and brother now have similar eruptions. Examination of his skin reveals diffuse excoriation, with scattered vesicles and erythematous papules. In a few areas, the lesions appear linear. What should you prescribe?

Correct answer: Topical scabicide for scabies Explanation The correct response is topical scabicide for scabies. Scabies is an infectious disease caused by the burrowing of a female mite into the stratum corneum of a patient's skin. The mite deposits eggs and fecal pellets in the burrow, which irritates the skin, causing pruritus (and secondary excoriation due to scratching). After about a month, enough mites have hatched and been spread by migration and by scratching, that patients often develop a wide spread, intensely pruritic eruption. Patients with scabies present complaining of a pruritic eruption, often with itching worse at night. The vesicular or papular lesions are usually found on the webs of fingers, the wrists, waist, ankles, axillae, extensor surface of elbows and knees, and the sides of the hands and feet. In infants, the scalp and face may be involved. Burrows are often seen in areas spared by itching, since scratching destroys the burrow. Patients usually have some secondary lesions from scratching, with erosions, pustules, nodules, or signs of eczematous inflammation. The diagnosis of scabies is suspected by the patient's history, characteristic symptoms, and distribution of lesions. A scraping of the burrow lesions with a curved #15-scalpel blade can isolate a mite, the eggs, or feces that can be easily identified by oil immersion under the microscope. Treatment includes the use of an effective scabicide, as well as preventative environmental measures to prevent re-infection. While lindane had been the preferred, most effective scabicide, the associated neurotoxicity of this agent, especially in infants, make it less appealing for use in scabies. Permethrin 5% cream is an effective, safe treatment, and is applied from the head to the feet. The cream is then washed off after 8 - 14 hours; some clinicians recommend a repeat treatment in 1 week. All intimate and family contacts should be treated as well, in case they are in the early stages of infestation. In order to prevent re-infection, patients should wash all clothing, bed linens, and towels that have touched the skin. Rhus dermatitis, an allergic contact dermatitis caused by exposure to poison ivy, poison oak, and poison sumac, presents with erythema, vesicles, and often has linear lesions (from the leaf or oleoresin oil streaking along the skin). This patient's history and intense pruritus are not consistent with Rhus dermatitis; in addition, they are not consistent with a bacterial or fungal infection.

Case A 32-year-old migrant worker presents because some of his toenails have changed color and have become brittle. You ask him to remove his shoes and socks; you note a yellow-brown discoloration of the first and fifth toenails. On closer exam, the toenails have become thickened and friable; there is debris under the nail distally. Question What is the most likely diagnosis?

Correct answer: Tinea unguium Explanation The clinical picture is suggestive of Tinea unguium. Typical findings include subungual hyperkeratosis and yellow-brown discoloration with debris under the nail distally and laterally. When lichen planus affects the nail, it causes destruction of the nail fold and nail bed with longitudinal splintering, which is not seen in this patient. Acute paronychia is an acute infection of the lateral or proximal nail fold. Findings include throbbing pain, erythema, swelling, and pain. There may or may not be an abscess. These findings are not present in this patient. Tinea pedis is incorrect. Tinea pedis is tinea infection of the foot, also known as athlete's foot, and is characterized by itchy lesions and scaling in the interdigital spaces and on the soles of the feet. In onychomycosis caused by Candida, the entire nail bed is thickened and dystrophic, and it most commonly occurs on the fingernails and is associated with immunocompromise.

Case An 18-year-old man presents with a rash. He states that the rash began a few weeks ago and it has worsened over the past 2 weeks. He denies fever, chills, nausea, vomiting, or weight loss. He also denies recent travel or illness. He has no significant past medical history and is otherwise very healthy. Physical exam reveals a well-developed, well-nourished man in no acute distress. He has areas of hyperpigmentation on his back and chest. A scraping taken from the back area shows orange fluorescence under UV light. Question What is the most likely cause of this patient's symptoms?

Correct answer: Tinea versicolor Explanation Tinea versicolor is a fungal infection common in adults and adolescents. The most commonly affected areas include the chest, back, and shoulders. Occasionally, it can be found on the face. It causes the affected skin to change color, becoming either lighter or darker. It was believed to be caused by a yeast called Malassezia furfur, but recent evidence points to Malassezia globosa as the cause. Tinea versicolor can recur, so treatment may need to be repeated in the future. The affected skin becomes reddish-brown to brown or may be light in color. Initially, the lesions are well-defined round-to-oval scaly macules. Over time, they tend to coalesce and form patches with various amounts of shading. The colors can be darker or lighter than the unaffected skin. Darker patches may disappear shortly after treatment is started, but lighter patches may take a long time to go away. The skin discoloration is not permanent; the color will eventually return to normal. Diagnosis can be confirmed by using an ultraviolet light (Wood light). The affected areas usually fluoresce and appear to be orange in color. If they do not fluoresce, the skin will appear darker than normal skin. A scraping of the skin will show the presence of hyphae in a characteristic "spaghetti and meatballs" appearance when exposed to potassium hydroxide. Diagnosis can also be confirmed through microscopic analysis. A scraping of the area placed in potassium hydroxide solution will show hyphae if a fungal infection is present. Tinea versicolor can be treated with several preparations that are applied to the skin. Over-the-counter preparations usually contain miconazole, ketoconazole, or clotrimazole; they can be found in shampoo or cream form. There are also prescription strength versions of these preparations. Oral medications (e.g., itraconazole or ketoconazole) also exist. Vitiligo causes a loss of pigmentation. Psoriasis causes raised scaly patches that do not fluoresce under UV light. Tinea unguium is a fungal infection of the nails. Seborrheic dermatitis causes patches of thick, scaly, crusty skin; it occurs mainly on the scalp. It can also appear in areas with oil glands (e.g., the folds of skin as in those of the face, groin, and armpits).

Case A 16-year-old Caucasian girl presents with a 6-month history of blackheads and whiteheads on her face. On examination, there are a few papules and pustules on her cheeks; there are no nodules. Her mother reports that she had similar spots on her face before she got married. Question What is an appropriate first-line medication for this patient?

Correct answer: Topical tretinoin Explanation Topical retinoids (e.g., tretinoin, adapalene, and tazarotene) are effective first-line treatments for mild acne. Tretinoin is a comedolytic which inhibits follicular canal obstruction by normalizing keratinocyte shedding, thereby inhibiting microcomedone formation. It also has anti-inflammatory properties. The clinical presentation is that of acne vulgaris. Acne is a common skin condition that affects the pilosebaceous units. It usually begins in adolescence although it also affects adults, especially women. Acne can be classified as mild, moderate, or severe. In mild acne, there are comedones, a few papules, and pustules without any nodules. In cases of moderate acne, patients have comedones, several papules and pustules, and a few nodules. In cases of severe acne, there are numerous comedones, papules, pustules, and nodules. Other topical agents (e.g., benzoyl peroxide, topical sulfacetamide, and topical azelaic acid) are also effective first-line treatments for mild acne. Topical antibiotics (e.g., erythromycin and clindamycin) are used in mild-to-moderate inflammatory acne because they have anti-inflammatory properties in addition to being effective against P. acnes. Benzoyl peroxide and topical antibiotic combinations reduce antibiotic resistance. Isotretinoin, a vitamin A derivative, is taken orally and is reserved for the treatment of severe nodulocystic acne unresponsive to other treatment regimens. It has anti-inflammatory activity, normalizes epidermal cell differentiation, and reduces sebum excretion. It has multiple side effects which include cheilitis, xerosis, pruritus, arthralgias, night blindness, and pseudotumor cerebri; it is also a potent teratogen. Moderate to severe inflammatory acne requires systemic antibiotics (e.g., oral tetracycline, doxycycline, minocycline, and erythromycin). They have anti-inflammatory effects in addition to battling the P. acnes colonization. They can increase resistance to erythromycin and cause possible GI disturbances; tetracycline should be avoided during pregnancy and in children under 9 years of age. Intralesional corticosteroid injections reduce the inflammation in nodulocystic acne. Triamcinolone is injected into the cyst; care should be taken because it can cause atrophy and depressed scars.

Question Allergic contact dermatitis is one of the most common dermatologic conditions and is the result of contact with an external agent. These reactions can be seen anywhere on the body, and symptoms include localized erythema, vesicles, scale, and pruritus. Members of the Rhus family, including poison ivy, poison oak, and poison sumac, cause this type of allergic reaction. Allergic contact dermatitis is an example of

Correct answer: Type IV hypersensitivity Explanation The majority of occupation-related skin disease is contact dermatitis, and identification of a contact allergen can be a difficult and time-consuming task. Substances in the workplace and in the household can cause this reaction. The table shows some common allergens found in these sites. Allergenic Substances Encountered in the Workplace Chromates Formol Synthetic resins Antioxidants Nickel Allergenic Substances found in the Household Cosmetics Rubber Certain stain removers Some locally applied drugs Allergic contact dermatitis is an example of delayed-type hypersensitivity or type IV hypersensitivity reactions. These responses are caused by specific effector T cells. The prototype delayed-type hypersensitivity reaction is the tuberculin test, used to determine whether an individual has previously been infected with Mycobacterium tuberculosis. When small amounts of a protein from M. tuberculosis are injected subcutaneously, a T-cell mediated local inflammatory reaction evolves over 24-72 hours in individuals who have responded to this pathogen. Local mast cell activation in the skin as a result of an insult such as an insect sting leads to the appearance of a distinctive lesion called a wheal-and-flare-reaction. A disseminated form of the wheal-and-flare reaction, known as urticaria, or hives, sometimes appears when ingested allergens enter the bloodstream and reach the skin. Antibody-mediated destruction of red blood cells is an example of a type II hypersensitivity reaction. Type III hypersensitivity reactions can occur when the antigen is soluble. Antigen-antibody aggregates or immune complexes are deposited in certain tissue sites and may be deposited in blood vessel walls. Serum sickness is a systemic type III hypersensitivity reaction.

Case A 4-year-old boy presents with headache, anorexia, dull abdominal pain, and skin lesions. About 3 weeks ago, the mother noticed several small red bumps on his feet that had changed into blisters and crusts and spread to his legs and lower abdomen. The child says that "bumps" only itch and hurt a little bit. His past medical and family history are non-contributory. Nobody in the surroundings is sick, and nobody has similar problems. His immunizations are up to date. On examination, you find a pale boy with periorbital edema; he is in mild distress. He has multiple discrete yellowish-brown crusted lesions that are 0.5-1 cm in diameter and have an erythematous base. Some lesions are fragile vesicles, some are pustular with honey colored discharge, and some are crusted. Regional lymph nodes are swollen. The rest of the exam is within normal limits. Question What diagnostic step should be taken next?

Correct answer: Urinalysis Explanation Your patient has probably impetigo, which is the most common superficial bacterial skin infection in children; it is often caused by S.aureus, S.pyogenes, or both. In a child 2 - 6 years old, headache, anorexia, a dull back, abdominal pain, and edema 3 weeks after pyoderma is suggestive of acute glomerulonephritis. It is actually an acute nephritic syndrome in which inflammation of the glomerulus is secondary to an immunologic mechanism. It is caused by group A Beta hemolytic Streptococcus. Children also have hypertension, proteinuria, hematuria, and RBC casts in urine. Therefore, urinalysis is the most important diagnostic step. A skin biopsy is most often not necessary in a case of impetigo, and it will not contribute to the diagnosis of acute glomerulonephritis secondary to an immunologic mechanism. You should consider a complete blood count (CBC) in a pale child to get the information about red blood cells, white blood cells, and platelets; however, this test will not contribute to the diagnosis of edema and glomerulonephritis. You should order a peripheral smear to obtain the information about hematologic disorders, particularly in cases of cytopenic states, because peripheral smears evaluate the functional status of the bone marrow. In this case, bone marrow dysfunction will be low on the list of differential diagnosis. A rapid antigen detection test (RADT) or Rapid Strep Test (RST) is done to determine whether or not a patient has streptococcal pharyngitis (and possibly streptococcal infection of other parts of the body). However, the patient may have streptococcal infection without having glomerulonephritis.

Question A 15-year-old boy presents to the office with his mother with a complaint of constant itching and burning on his arms and thighs for 2 weeks. His mother admits to giving him acetaminophen without relief. The patient is taking no other medications. On clinical exam, there are multiple 2-cm wheals with a few small papules on his thighs and forearms. They are red and slightly raised. What is the most likely diagnosis?

Correct answer: Urticaria Explanation Urticaria is chronic or acute and is characterized by wheals and papules. Itching and prickling sensations are constant. Both sexes are affected equally, and it's often seen in childhood or teen years. The presenting areas are the arms, legs, thighs, and waist. The most common cause is an allergy to medications, foods, or physical agents. The symptoms usually disappear within 6 months (acute), but they can last longer (chronic). Erythema nodosa is an acute inflammatory condition characterized by painful nodules on the anterior aspect of the legs. It is often symptomatic of a bacterial, viral, or fungal disease or drug eruption. This occurs most often in women and between the ages of 20 - 30 years. Clinical features include acute fever, malaise, and joint pain. Lesions are nodular, painful, red, and shiny. The symptoms last 2 weeks, and the lesions heal without scarring. Erythema multiforme is characterized by macules, papules, vesicles, and bullae. It occurs secondary to a toxic influence. There may or may not be a prodromal period with a sore throat, diarrhea, and fever. The lesions are red macules or papules and are seen on the sides of the neck, face, legs, genitalia, and mucosa membranes. A typical lesion is a "target" or "iris" lesion. The course is generally 3 - 4 weeks. Erythema ab igne, often called toasted skin syndrome, occurs secondary to exposure to heat from flames or heating appliances. The course is generally benign, but may have a potential for malignant changes. The lesions are red, mottled skin with hypo- or hyperpigmentation. After many years of constant exposure, hyperkeratotic papules, plaques, and ulcers may occur. Nummular eczema has round, coin-like (nummular) lesions with a distribution on the extensor surface of the extremities as well as the posterior portion of the trunk, buttock, and legs. Purulent drainage is not uncommon. Treatment includes topical steroids, systemic antibiotics, and antihistamines; decreasing exposure to water is recommended, and drying agents are used when oozing is present.

Case An 18-month-old infant presents with a 1-day history of fever that is currently 101°F rectally. You symptomatically treat the patient and ask the mother to return if the condition worsens. 2 days later, the mother returns because the infant has developed small red spots that became bumps and are now blisters. The mother also noted the infant was scratching the lesions. The majority of these lesions are on the thorax. Each vesicle resides on its own erythematous base. Question Ico-delete Highlights What is the most likely diagnosis?

Correct answer: Varicella Explanation The clinical picture is suggestive of a varicella infection. Signs and symptoms include fever and malaise, which are mild in children. The rash usually begins on the face and trunk and then spreads to the extremities. The lesions are pruritic; the lesions appear as maculopapular and then will become vesicles, then pustular, and then will crust over. All forms of the lesions can be seen at the same time. A classic description of primary varicella lesions are "dewdrop on a rose petal," indicating that each vesicle resides on an erythematous base. Shingles is commonly found in adults, but cases have been reported in children. Skin lesions resemble varicella. Pain is often severe and then a rash will appear, usually following a dermatomal pattern on the skin. In most cases, a single dermatome is involved on the chest or thorax. Ramsey-Hunt syndrome is reactivated varicella, known as herpes zoster. Facial palsy is a key finding. Erythema infectiosum (or fifth disease) is caused by Parvovirus B19. It occurs commonly in children ages 6-15. The common "slapped-cheek" appearance is seen as an erythematous eruption on the face and then spreads to the trunk and extremities. Primary herpes simplex will appear as vesicles, will transition to ulcers over several days, and will then epithelialize over 1-2 weeks.

Question A 71-year-old man with a history of prostate cancer and unstable angina is admitted to a community hospital for facial cellulitis. Examination shows an indurated erythematous rash that extends across the face and zygomatic arch to the lip. In the oral cavity, vesicular lesions are present on the hard palate. Laboratory results are within normal ranges. The patient describes the lesions as very painful. A Tzanck smear of the vesicle showed multinucleated syncytial giant cells and intranuclear inclusion bodies. What is the most likely causative agent?

Correct answer: Varicella-zoster virus Explanation Vesicular lesions of the palate and skin are suggestive of herpes simplex virus infection. However, the patient's lesions in this case are within the region of trigeminal nerve and this dermatomal distribution strongly indicates varicella-zoster virus (VZV). Multinucleated giant cells on Tzanck smears are seen both in the case of Varicella and Herpes Simplex. In older or immunosuppressed patients, varicella (chickenpox) virus is reactivated after latency in the basal ganglia (shingles). The increase in anti-VZV antibody can be useful in differentiating shingles from infection with herpes simplex virus 1. NSAIDS and corticosteroids may be used to allay pain. Acyclovir helps in faster healing of the lesions. While VZV can be isolated in tissue culture, fungal agents such as Malassezia and Blastomyces and the bacterium Mycobacterium marinum, are more difficult to culture. However, VZV culture is not used in diagnosis because the viral growth rate is rather slow. A preferred test would be direct antigen detection, the results of which are made available in 2 hours. PCR is another reliable test. Infection with a bacterial pathogen, such as Staphylococcus, would likely lead to suppurative lesions, leukocytosis, and fever.

Case A 32-year-old woman is brought to the emergency room with sudden onset of fever, headache, sore throat, profuse watery diarrhea, vomiting, and lethargy; symptoms started in the morning. On physical examination, she is slightly confused; her temperature is 39°C (103°F), her blood pressure is 100/50 mmHg, and she has diffuse sunburn like an erythematous rash. When the emergency doctor is trying to find out if she is pregnant, she mentions that her period started 2 days ago. Question What is the most likely diagnosis?

orrect answer: Toxic shock syndrome Explanation This is a typical case presentation of toxic shock syndrome (TSS). The patient is a young woman; she is menstruating, and the disease had a sudden abrupt onset with a rapid deterioration. TSS is caused by toxic shock syndrome toxin-1 (TSST-1) produced by Staphylococcus aureus. TSST-1 is a pyrogenic exotoxin, which causes fever, multiple organ dysfunction, rash, hypotension, syncope, and shock. TSS can be seen in menstruating women who use tampons, individuals with wound infections, or patients with nasal packing to stop bleeding from the nose. Scarlet fever also presents with sore throat and a fine erythematous rash, but diarrhea and shock is unlikely. Meningococcemia is a life-threatening condition and should be suspected in every case presenting with rash, fever, and headache. Meningococcal rash is more petechial, and it progresses to gangrenous lesions if not treated. Rocky Mountain spotted fever also presents with fever, headache with a sudden onset, but the typical rash appears 2-6 days later. It initially starts on the hands and feet, then moves to the trunk. It is a rickettsial disease transmitted by ticks, from dogs, and rodents. Food poisoning can have acute onset with fever, but it is very unlikely to present with a rash


Kaugnay na mga set ng pag-aaral

ATG 268 Business Ethics Final Exam

View Set

Chapter 42: Drugs Used to Treat Glaucoma and Other Eye Disorders

View Set

Contingencies, Addenda, and Amendments +Financing Real Estate (Texas Pre-License - Law Of Contracts)

View Set

pre Class 11 - intro to muscle structure and function

View Set

Extra Test 1, Test 2, Test 3 pt.1, Test 3 pt.2, Test 4

View Set

Postpartum Physiological Assessments and Nursing Care

View Set